[obm-l] Questões legais - AVENTURA ????

2013-04-10 Por tôpico Robério Alves


 PROBLEMA1
Os números naturais de 1 até
1998 são escritos em um imenso quadro negro. Em seguida, um aluno apaga dois
quaisquer colocando no lugar sua diferença (não negativa). Depois de muitas
operações, um único número ficará escrito no quadro. É possível que esse número
seja zero?
 
 
PROBLEMA 2
Em uma ilha plana existem 11
cidades numeradas de 1 a 11. Estradas retas ligam 1 a 2, 2 a 3, 3 a 4, ..., 10
a 11 e 11 a 1. É possível que uma reta corte todas as estradas?


AVENTURA 
-- 
Esta mensagem foi verificada pelo sistema de antivírus e
 acredita-se estar livre de perigo.



[obm-l] Re: [obm-l] Questões legais - AVENTURA ????

2013-04-10 Por tôpico Rogerio Ponce
Ola' pessoal,

No problema1, observemos que:
- duas casas pares, quando combinadas, geram uma diferenca tambem par, e
a quantidade de numeros impares nao se altera.
- uma casa impar quando combinada com uma casa par, gera uma diferenca
impar, e a quantidade de impares nao se altera.
- duas casas impares, quando combinadas, geram uma diferenca par, e a
quantidade de impares diminui de 2.

Como existem 999 impares no quadro negro, e a quantidade de impares diminui
somente em pulos de 2, e' impossivel obtermos um numero par no final do
processo.


No problema2, existem 11 estradas.
Suponhamos que haja uma reta que corte todas as estradas.
Ao percorrermos todas as cidades, a partir da cidade 1 (e voltando para
ela), ao atravessarmos a tal reta, mudariamos de lado (em relacao a tal
reta) 11 vezes, ou seja, terminariamos o percurso do lado oposto ao lado da
partida, o que e' absurdo.
Logo nao existe a tal reta.

[]'s
Rogerio Ponce



2013/4/10 Robério Alves prof_robe...@yahoo.com.br


  *PROBLEMA1*
 Os números naturais de 1 até 1998 são escritos em um imenso quadro negro.
 Em seguida, um aluno apaga dois quaisquer colocando no lugar sua diferença
 (não negativa). Depois de muitas operações, um único número ficará escrito
 no quadro. É possível que esse número seja zero?
 ** **
 ** **
 *PROBLEMA 2*
 Em uma ilha plana existem 11 cidades numeradas de 1 a 11. Estradas retas
 ligam 1 a 2, 2 a 3, 3 a 4, ..., 10 a 11 e 11 a 1. É possível que uma reta
 corte todas as estradas?


 AVENTURA 






 --
 Esta mensagem foi verificada pelo sistema de antivírus e
 acredita-se estar livre de perigo.


-- 
Esta mensagem foi verificada pelo sistema de antivírus e
 acredita-se estar livre de perigo.



[obm-l] Re: [obm-l] Questões interessantes (na minha opinião)

2013-03-02 Por tôpico Bernardo Freitas Paulo da Costa
2013/3/1 Artur Costa Steiner steinerar...@gmail.com:
 1) suponhamos que exista uma função f tal que, para todo real x, tenhamos 
 f(f(x)) = ax^2 + bx + c, a não nulo, b e c reais. Mostre que (b +1)(b - 3) = 
 4ac.
Esse eu ainda tenho que pensar com cuidado. A primeira coisa é reduzir
a g(g(x)) = x^2 + c, mas eu ainda não sei fazer o caso c  0.

 2) seja (a_n) uma sequência de reais e (p_n) uma sequência de pesos 
 positivos. Seja (s_n) a sequência das médias ponderadas de (a_n) com relação 
 os pesos p_n. Mostre que, se Soma p_n divergir, então

 liminf s_n = liminf a_n = limsup a_n = limsup s_n

 É bem fácil mostrar que, se Soma p_n convergir, as desigualadas da direita e 
 da esquerda não têm que valer.
Curioso... Eu diria que s_n é uma combinação convexa dos a_n, logo s_N
= min(a_n, n=1..N) e portanto min(s_N, N=1..k) = min(min(a_n,
n=1..N), N=1..k) = min(a_n, n=1..k). Claro que tem que fazer do outro
lado (no infinito, não no 1) mas eu diria que liminf a_n = liminf
s_n. Mais tarde tento enviar uma prova dessa soma de Césaro.

 3) Seja f uma função definida em um intervalo I de R (suponhamos aberto, para 
 facilitar) e com valores em R. Suponhamos que, em cada ponto de I, as 4 
 derivadas de Dini de f existam e sejam finitas. Mostre que existe um 
 subintervalo de I no qual f é Lipschitz.
 Isto é mais fácil de mostrar se supusermos diferenciabilidade cheia em todo o 
 I. Mas, de fato, basta a existência das 4 derivadas de Dini.
Diferenciabilidade cheia = full differentiability = diferenciável no
sentido usual ? (Nunca fiz nada com derivadas de Dini)

-- 
Bernardo Freitas Paulo da Costa

-- 
Esta mensagem foi verificada pelo sistema de antivírus e
 acredita-se estar livre de perigo.


=
Instruções para entrar na lista, sair da lista e usar a lista em
http://www.mat.puc-rio.br/~obmlistas/obm-l.html
=


Re: [obm-l] Re: [obm-l] Questões interessantes (na minha opinião)

2013-03-02 Por tôpico Artur Costa Steiner
Para o caso da condição de Lipschitz, supondo que f seja diferenciável em I, me 
ocorreu uma vez o seguinte

1) f' é, conforme se sabe, o limite de uma sequência de funções contínuas.

2) Como R é um espaço de Baire, para toda sequência g_n de funções contínuas em 
um intervalo I  que convirja para uma função g, existe um subintervalo no qual 
as g_n são uniformemente limitadas por algum M  0. Logo, g é limitada por M 
neste subintervalo. 

3) de (2) segue-se haver um subintervalo de I no qual f' é limitada por algum M 
 0. Logo, neste subintervalo f' é Lipschitz e M é uma constante da condição de 
Lipschitz.


Uma vez eu mostrei esta prova para algumas pessoas e não gostaram. Paciência, 
não se pode agradar a todos. No caso, não agradei ninguém. Alguns disseram que 
estava errado, porque sabidamente diferenciabilidade não implica que a função 
seja localmente Lipschitz. Mas a condição que eu citei não é ser localmente 
Lipschitz, é mais fraca do que isso. 

Depois vim a saber que para, haver o subintervalo em que f seja Lipschitz, 
basta que em cada ponto de I as 4 derivadas de Dini de f sejam finitas. Me 
enrolei nesta prova, mas acho que tenho uma por contradição.

Aliás, nos complexos há uma conclusão interessante. Se f é inteira, então f é 
Lipschitz em todo conjunto limitado do plano complexo.

Abraços


Artur Costa Steiner

Em 02/03/2013, às 17:50, Bernardo Freitas Paulo da Costa 
bernardo...@gmail.com escreveu:

 2013/3/1 Artur Costa Steiner steinerar...@gmail.com:
 1) suponhamos que exista uma função f tal que, para todo real x, tenhamos 
 f(f(x)) = ax^2 + bx + c, a não nulo, b e c reais. Mostre que (b +1)(b - 3) 
 = 4ac.
 Esse eu ainda tenho que pensar com cuidado. A primeira coisa é reduzir
 a g(g(x)) = x^2 + c, mas eu ainda não sei fazer o caso c  0.
 
 2) seja (a_n) uma sequência de reais e (p_n) uma sequência de pesos 
 positivos. Seja (s_n) a sequência das médias ponderadas de (a_n) com relação 
 os pesos p_n. Mostre que, se Soma p_n divergir, então
 
 liminf s_n = liminf a_n = limsup a_n = limsup s_n
 
 É bem fácil mostrar que, se Soma p_n convergir, as desigualadas da direita e 
 da esquerda não têm que valer.
 Curioso... Eu diria que s_n é uma combinação convexa dos a_n, logo s_N
 = min(a_n, n=1..N) e portanto min(s_N, N=1..k) = min(min(a_n,
 n=1..N), N=1..k) = min(a_n, n=1..k). Claro que tem que fazer do outro
 lado (no infinito, não no 1) mas eu diria que liminf a_n = liminf
 s_n. Mais tarde tento enviar uma prova dessa soma de Césaro.
 
 3) Seja f uma função definida em um intervalo I de R (suponhamos aberto, 
 para facilitar) e com valores em R. Suponhamos que, em cada ponto de I, as 4 
 derivadas de Dini de f existam e sejam finitas. Mostre que existe um 
 subintervalo de I no qual f é Lipschitz.
 Isto é mais fácil de mostrar se supusermos diferenciabilidade cheia em todo 
 o I. Mas, de fato, basta a existência das 4 derivadas de Dini.
 Diferenciabilidade cheia = full differentiability = diferenciável no
 sentido usual ? (Nunca fiz nada com derivadas de Dini)
 
 -- 
 Bernardo Freitas Paulo da Costa
 
 -- 
 Esta mensagem foi verificada pelo sistema de antivírus e
 acredita-se estar livre de perigo.
 
 
 =
 Instruções para entrar na lista, sair da lista e usar a lista em
 http://www.mat.puc-rio.br/~obmlistas/obm-l.html
 =

-- 
Esta mensagem foi verificada pelo sistema de antiv�rus e
 acredita-se estar livre de perigo.


=
Instru��es para entrar na lista, sair da lista e usar a lista em
http://www.mat.puc-rio.br/~obmlistas/obm-l.html
=


[obm-l] Questões interessantes (na minha opinião)

2013-03-01 Por tôpico Artur Costa Steiner
Eu acho estes dois aqui interessantes. O primeiro acho que já enviei para a 
lista, ma não houve comentários. Tive muita dificuldade. O segundo também acho 
interessante.

1) suponhamos que exista uma função f tal que, para todo real x, tenhamos 
f(f(x)) = ax^2 + bx + c, a não nulo, b e c reais. Mostre que (b +1)(b - 3) = 
4ac.

2) seja (a_n) uma sequência de reais e (p_n) uma sequência de pesos positivos. 
Seja (s_n) a sequência das médias ponderadas de (a_n) com relação os pesos p_n. 
Mostre que, se Soma p_n divergir, então

liminf s_n = liminf a_n = limsup a_n = limsup s_n

É bem fácil mostrar que, se Soma p_n convergir, as desigualadas da direita e da 
esquerda não têm que valer.

A desigualdade do meio vale, é claro, para qualquer sequência de reais.

E eu também acho este interessante e pouco conhecido. Tive uma boa dificuldade.

3) Seja f uma função definida em um intervalo I de R (suponhamos aberto, para 
facilitar) e com valores em R. Suponhamos que, em cada ponto de I, as 4 
derivadas de Dini de f existam e sejam finitas. Mostre que existe um 
subintervalo de I no qual f é Lipschitz.
Isto é mais fácil de mostrar se supusermos diferenciabilidade cheia em todo o 
I. Mas, de fato, basta a existência das 4 derivadas de Dini.

Abraco a todos


Artur Costa Steiner
-- 
Esta mensagem foi verificada pelo sistema de antiv�rus e
 acredita-se estar livre de perigo.


=
Instru��es para entrar na lista, sair da lista e usar a lista em
http://www.mat.puc-rio.br/~obmlistas/obm-l.html
=


[obm-l] Re: [obm-l] Questões lógicas

2011-02-26 Por tôpico Marcelo Costa
o próximo é 200, todos os números começam com D
Abraços

Em 25 de fevereiro de 2011 11:00, Marco Bivar Jr.
marco.bi...@gmail.comescreveu:

 Duas questões lógicas para os colegas deleitarem-se:

 1. Qual o número X na sequência: 2, 10, 12, 16, 17, 18, 19, X, ...?

 2. Um fazendeiro decidiu doar sua fazenda para apenas um de seus dois
 filhos, que a teria logo após a sua morte. Ele decidiu que deveriam
 circundar a fazenda numa volta a cavalo, e que o dono do cavalo mais lento,
 este ficaria com a fazenda. Haviam dois cavalos pretos no estábulo, e cada
 um pertencia a um filho. Os filhos, então, pegaram um cavalo, e
 apressaram-se para completar a volta em primeiro. Por que?

 --
 Marco Bivar Jr.



[obm-l] Re: [obm-l] Re: [obm-l] Questões lógicas

2011-02-26 Por tôpico Marco Bivar Jr.
A questão 1 foi tirada da lousa em sala de aula na faculdade. A número 2 é
uma piada contada por um amigo. Portanto, não pensei 2 vezes ao postá-las na
lista. Quanto à questão 1, se eu falo português, então X=200. A questão 2
não merece atenção pois vocês viram quantas interpretações ela pode dar.
Deleitarem-se, Se deleitarem...acho que é questão de estética; ou seja,
você escolhe. Havia por Haviam...talvez você esteja certo.

--- Marco Bivar Jr.

Em 26 de fevereiro de 2011 07:17, Marcelo Costa mat.mo...@gmail.comescreveu:

 o próximo é 200, todos os números começam com D
 Abraços

 Em 25 de fevereiro de 2011 11:00, Marco Bivar Jr. 
 marco.bi...@gmail.comescreveu:

 Duas questões lógicas para os colegas deleitarem-se:

 1. Qual o número X na sequência: 2, 10, 12, 16, 17, 18, 19, X, ...?

 2. Um fazendeiro decidiu doar sua fazenda para apenas um de seus dois
 filhos, que a teria logo após a sua morte. Ele decidiu que deveriam
 circundar a fazenda numa volta a cavalo, e que o dono do cavalo mais lento,
 este ficaria com a fazenda. Haviam dois cavalos pretos no estábulo, e cada
 um pertencia a um filho. Os filhos, então, pegaram um cavalo, e
 apressaram-se para completar a volta em primeiro. Por que?


 --
 Marco Bivar Jr.





-- 
Marco Bivar Jr.


[obm-l] Questões lógicas

2011-02-25 Por tôpico Marco Bivar Jr.
Duas questões lógicas para os colegas deleitarem-se:

1. Qual o número X na sequência: 2, 10, 12, 16, 17, 18, 19, X, ...?

2. Um fazendeiro decidiu doar sua fazenda para apenas um de seus dois
filhos, que a teria logo após a sua morte. Ele decidiu que deveriam
circundar a fazenda numa volta a cavalo, e que o dono do cavalo mais lento,
este ficaria com a fazenda. Haviam dois cavalos pretos no estábulo, e cada
um pertencia a um filho. Os filhos, então, pegaram um cavalo, e
apressaram-se para completar a volta em primeiro. Por que?

-- 
Marco Bivar Jr.


[obm-l] RE: [obm-l] Questões lógicas

2011-02-25 Por tôpico Artur Steiner

1. Não está parecendo um problema matemático. Na realidade, nenhuma sequência 
fica definida conhecendo-se um número finito de seus termos. Há uma infinidade 
de possibilidades poara se determinar o próximo termo. Vc pode, por exemplo, 
ajustar um polinômio aos pontos dados e estimar os outros com base neste 
polinômio. Isto é possível para uma infinidade de polinômios de gruas maior ou 
igual ao número de pontos -1.

Aqui, vemos que todos os números, quando grafados em Português, começam com d. 
Se esta for sa lei de formação, o próximo número é duzentos. Em outra´s 
línguas, como o Inglês, isto não vale. 

 

2. Por que cada filho pegu o cavalo do irmão.

Ou então, ambos eram pessoas altruístas, cada um pegou seu próprio cavalo e 
procurou chegar em primeiro lugar para que o irmão ficasse com a fazenda.

 

Artur

 

 

 

5 Feb 2011 11:00:01 -0300
Subject: [obm-l] Questões lógicas
From: marco.bi...@gmail.com
To: obm-l@mat.puc-rio.br

Duas questões lógicas para os colegas deleitarem-se:




1. Qual o número X na sequência: 2, 10, 12, 16, 17, 18, 19, X, ...?


2. Um fazendeiro decidiu doar sua fazenda para apenas um de seus dois filhos, 
que a teria logo após a sua morte. Ele decidiu que deveriam circundar a fazenda 
numa volta a cavalo, e que o dono do cavalo mais lento, este ficaria com a 
fazenda. Haviam dois cavalos pretos no estábulo, e cada um pertencia a um 
filho. Os filhos, então, pegaram um cavalo, e apressaram-se para completar a 
volta em primeiro. Por que?
-- 
Marco Bivar Jr.
  

[obm-l] Re: [obm-l] Questões lógicas

2011-02-25 Por tôpico João Luís Gomes Guimarães
1. 200

2. Se cada um tenciona vencer a corrida, significa que cada irmão pegou o 
cavalo que pertencia ao outro

From: Marco Bivar Jr. 
Sent: Friday, February 25, 2011 11:00 AM
To: obm-l@mat.puc-rio.br 
Subject: [obm-l] Questões lógicas

Duas questões lógicas para os colegas deleitarem-se:


1. Qual o número X na sequência: 2, 10, 12, 16, 17, 18, 19, X, ...?

2. Um fazendeiro decidiu doar sua fazenda para apenas um de seus dois filhos, 
que a teria logo após a sua morte. Ele decidiu que deveriam circundar a fazenda 
numa volta a cavalo, e que o dono do cavalo mais lento, este ficaria com a 
fazenda. Haviam dois cavalos pretos no estábulo, e cada um pertencia a um 
filho. Os filhos, então, pegaram um cavalo, e apressaram-se para completar a 
volta em primeiro. Por que?

-- 
Marco Bivar Jr.


[obm-l] Re: [obm-l] RE: [obm-l] Questões lógicas

2011-02-25 Por tôpico Bernardo Freitas Paulo da Costa
2011/2/25 Artur Steiner artur_stei...@hotmail.com:
 2. Por que cada filho pegu o cavalo do irmão.
2. também não parece um problema matemático.

 Ou então, ambos eram pessoas altruístas, cada um pegou seu próprio cavalo e
 procurou chegar em primeiro lugar para que o irmão ficasse com a fazenda.
Ou entao a fazenda estava endividada até o pescoço... e ninguém queria
ficar com ela.

E para defender a língua pátria: para os colegas *se* deleitarem,
preposição exige próclise. E Havia dois cavalos, pelamordedeus.

 Artur
-- 
Bernardo Freitas Paulo da Costa

=
Instruções para entrar na lista, sair da lista e usar a lista em
http://www.mat.puc-rio.br/~obmlistas/obm-l.html
=


[obm-l] Questões resolvam pra ver - ajuda

2010-10-05 Por tôpico Robério Alves



01) sendo A={ n/n=2p-1 e p pertence a B } qual é a condição
sobre B para que n seja um número ímpar ?




02) Um subconjunto X de número natrais contém 12 multiplos de 4, 7 multiplos de 
6 e 5 multiplos de 12 e 8 números impares. Qual o número de elementos de X ?



  

[obm-l] Re: [obm-l] Questões resolvam pra ver - ajuda

2010-10-05 Por tôpico Vinícius
Quem pode me ajudar urgente?
Cada cartela de uma coleção é formada por seis quadrados coloridos,
justapostos
como indica a figura abaixo.
 __
I___I___I___I
I___I___I___I

Em cada cartela, dois quadrados foram coloridos de azul,
dois de verde e dois de rosa. A coleção apresenta todas as
possibilidades de distribuição dessas cores nas cartelas nas
condições citadas e não existem cartelas com a mesma
distribuição de cores. Retirando-se ao acaso uma cartela da
coleção, a probabilidade de que somente uma coluna
apresente os quadrados de mesma cor é de:


[obm-l] Questões do colégio naval 2010

2010-08-13 Por tôpico Luiz Paulo
Bom dia colegas da lista, por esses dias ocorreu o concurso de admissão ao 
colégio naval. Alguns alunos me trouxeram a prova para dar uma olhada e duas 
questões me chamaram a atenção em especial e gostaria da ajuda de vocês.
 
Questão 1
Estudando o quadrado dos números naturais um aluno, um aluno conseguiu 
determinar corretamente o número de soluções inteiras e positivas da equação 
5x^2+11y^2=876543.
Qual foi o número de soluções que esse aluno obteve?
 
Questão 2
Sejam p(x)=2x^2010-5x^2-13x+7 e q(x)=x^2+x+1. Tomando r(x) como sendo o resto 
da divisão de p(x) por q(x), o valor de r(2) será?
Resolvendo por números complexos fica fácil, só dá trabalho, é só fatorar q(x) 
em produto de dois fatores de 1º grau, só que essas raízes são complexas e 
preferencialmente escritas na forma trigonométrica para poder usar a  fórmula 
de moivre quando for substituir em p(x) para obter os coeficientes de r(x) que 
será da forma r(x)=ax+b...
Entretanto, essa prova é para alunos que nem entraram no ensino médio e por 
isso não conhecem Moivre! Por isso, gostaria de saber se vocês têm uma solução 
mais simples para essa questão.
 
Muito obrigado, Luiz.
 
 
 
 
 
 


  

[obm-l] Res: [obm-l] Questões do colégio naval 2010

2010-08-13 Por tôpico Márcio Pinheiro
Para a questão 1, um caminho é observar os possíveis algarismos das unidades do 
quadrado de um número inteiro qualquer (0, 1, 4, 5, 6 ou 9), de um múltiplo de 
5 
(0 ou 5) e de um múltiplo de 11, previamente multiplicados por um quadrado 
(idem 
aos 6 primeiros). Enfim, basta analisar as possibilidades dos algarismos finais 
de 5x^2+11y^2 pra concluir que nenhum pode ser 3 (de 876543).
Para a 2, um caminho (não tão simples, mas educativo) é o seguinte: pode-se 
garantir que o resto deve assumir a forma r(x) = ax + b, com a e b reais. Logo, 
pelo algoritmo da divisão: p(x) = q(x)*Q(x) + r(x), para todo x (até mesmo 
complexo), em que Q(x) é o quociente da divisão. Logo: 2x^2010-5x^2-13x+7 = 
(x^2+x+1)*Q(x) + ax+b, qualquer que seja o x. A ideia é sumir com o Q(x), 
desconhecido e desinteressante, aqui, fazendo x assumir valores convenientes 
(as 
raízes de q(x)). Porém, como se sabe, tais valores não são reais. Sejam m e n 
os 
mesmos (distintos). Então, m^2+m+1 = n^2+n+1 = 0. Multiplicando respectivamente 
por m - 1 e n - 1, conclui-se que: (m - 1)*(m^2+m+1) = (n - 1)*(n^2+n+1) = 0, 
ou 
seja, m^3 - 1 = n^3 - 1 = 0. Observe-se que m^3 = 1 = (m^3)^670 = m^2010 = 1 e 
que m^2 = - m - 1, bem como para n. Logo, fazendo respectivamente x = m e x = n 
no algoritmo da divisão, vem que: (2m^2010-5m^2-13m+7 = 2*1-5(-m-1)-13m+7 = 14 
- 
8m)
14 - 8m = am + b e 14 - 8n = an + b. Subtraindo, conclui-se que a = - 8 
(NOTANDO 
QUE m É DISTINTO DE n). Substituindo, que b = 14. Logo, r(x) = - 8x + 14, do 
que: r(2) = - 2.
Outro caminho, menos laborioso, é fazer no braço a divisão pelo método da 
chave.
Espero ter ajudado.
Márcio Pinheiro.





De: Luiz Paulo paulolui...@yahoo.com.br
Para: obm-l@mat.puc-rio.br
Enviadas: Sexta-feira, 13 de Agosto de 2010 10:10:04
Assunto: [obm-l] Questões do colégio naval 2010


Bom dia colegas da lista, por esses dias ocorreu o concurso de admissão ao 
colégio naval. Alguns alunos me trouxeram a prova para dar uma olhada e duas 
questões me chamaram a atenção em especial e gostaria da ajuda de vocês.

Questão 1
Estudando o quadrado dos números naturais um aluno, um aluno conseguiu 
determinar corretamente o número de soluções inteiras e positivas da equação 
5x^2+11y^2=876543.
Qual foi o número de soluções que esse aluno obteve?

Questão 2
Sejam p(x)=2x^2010-5x^2-13x+7 e q(x)=x^2+x+1. Tomando r(x) como sendo o resto 
da 
divisão de p(x) por q(x), o valor de r(2) será?
Resolvendo por números complexos fica fácil, só dá trabalho, é só fatorar q(x) 
em produto de dois fatores de 1º grau, só que essas raízes são complexas e 
preferencialmente escritas na forma trigonométrica para poder usar a  fórmula 
de 
moivre quando for substituir em p(x) para obter os coeficientes de r(x) que 
será 
da forma r(x)=ax+b...
Entretanto, essa prova é para alunos que nem entraram no ensino médio e por 
isso 
não conhecem Moivre! Por isso, gostaria de saber se vocês têm uma solução mais 
simples para essa questão.

Muito obrigado, Luiz.


  

[obm-l] Questões simples

2009-10-21 Por tôpico Diogo FN
VocÊ pode ajudar?

01. Durante quanto tempo em um dia a marcação de um relógio digital, no modo 24 
horas, indica o número de horas superior ao número de minutos?

02. Mariana, Carlos e Paula são irmãos e cada um deles tem uma quantidade 
diferentes de filhos. Carlos tem o dobro do número de filhos de Paula e Paula 
tem o triplo do número de filhos de Mariana. Qual o total de filhos desses 
irmãos? 8/10/12/14 ou 16 filhos?

03. Paulo, Pedro e Plínio são amigos e fazem aniversário no mesmo dia. Sabe-se 
que eles nasceram no Rio de Janeiro, Belo Horizonte e São Paulo, em anos 
alternados. Paulo nasceu dois anos antes de Plínio. Pedro é o mais velho e não 
nasceu no Rio de Janeiro. Plínio tem 18 anos e é mineiro. O que posso afirmar?
a. Pedro tem 20 anos;
b. O mais novo é carioca;
c. Paulo é Paulista;
d. O carioca tem 20 anos;
e. O Carioca é mais novo que  o mineiro.

Agradeço pela atenção,

Diogo FN
http://diogofn.6te.net



  

Veja quais são os assuntos do momento no Yahoo! +Buscados
http://br.maisbuscados.yahoo.com

[obm-l] RE: [obm-l] Questões simples

2009-10-21 Por tôpico marcone augusto araújo borges

2) Mariana tem x filhos,Paula,3x e Carlos tem 6x.Total:10x,um múltiplo de 
10.Dos números apresentados,o único múltiplo de 10 é 10.Não é isso?
 


Date: Wed, 21 Oct 2009 09:06:43 -0700
From: diog...@yahoo.com.br
Subject: [obm-l] Questões simples
To: obm-l@mat.puc-rio.br





VocÊ pode ajudar?
 
01. Durante quanto tempo em um dia a marcação de um relógio digital, no modo 24 
horas, indica o número de horas superior ao número de minutos?
 
02. Mariana, Carlos e Paula são irmãos e cada um deles tem uma quantidade 
diferentes de filhos. Carlos tem o dobro do número de filhos de Paula e Paula 
tem o triplo do número de filhos de Mariana. Qual o total de filhos desses 
irmãos? 8/10/12/14 ou 16 filhos?
 
03. Paulo, Pedro e Plínio são amigos e fazem aniversário no mesmo dia. Sabe-se 
que eles nasceram no Rio de Janeiro, Belo Horizonte e São Paulo, em anos 
alternados. Paulo nasceu dois anos antes de Plínio. Pedro é o mais velho e não 
nasceu no Rio de Janeiro. Plínio tem 18 anos e é mineiro. O que posso afirmar?
a. Pedro tem 20 anos;
b. O mais novo é carioca;
c. Paulo é Paulista;
d. O carioca tem 20 anos;
e. O Carioca é mais novo que  o mineiro.
 
Agradeço pela atenção,
 
Diogo FN
http://diogofn.6te.net
 


Veja quais são os assuntos do momento no Yahoo! + Buscados: Top 10 - 
Celebridades - Música - Esportes 
_
Você sabia que pode acessar o Messenger direto do seu Hotmail? Descubra como!
http://www.microsoft.com/brasil/windows/windowslive/products/tutoriais.aspx

[obm-l] Questões de cálculo

2009-07-06 Por tôpico Celso Souza
Senhores,
 
   Estou com problemas para resolver duas questões, a saber:
 
1) Calcule a integral de superfície INT( F.dS) sobre a superfície x^2 + y^2 + 
z^2 = 9, onde F = x^3.i + y^3.j + z^3.k,
   Nesta questão foi sugerido usar o teorema do divergente e de coordenadas 
esféricas.
 
2) Determine o vetor normal unitário em um ponto (x, y, z) sobre a superfície z 
= a^2 - x^2 - y^2. Para tanto, considere que a superfície possa ser 
parametrizada segundo: x = a.sen(theta).cos(phi), y = a.sen(theta).sen(phi), z 
= a^2.cos^2(theta)
 
   Qualquer ajuda é bem vinda !
 
Obrigado,
 
Celso


  

Veja quais são os assuntos do momento no Yahoo! +Buscados
http://br.maisbuscados.yahoo.com

[obm-l] Questões

2009-06-08 Por tôpico Diogo FN

Amigos, vocês poderiam me indicar onde posso encontrar questões do assunto 
Álgebra Booleana?
Agradeço-lhes.


  Veja quais são os assuntos do momento no Yahoo! +Buscados
http://br.maisbuscados.yahoo.com

Re: [obm-l] Questões

2009-06-08 Por tôpico fabrici...@usp.br

http://www.vision.ime.usp.br/~jb/boolean%20algebra/aulas_mac0329.pdf


On Jun 8, 2009, at 16:20 , Diogo FN wrote:



Amigos, vocês poderiam me indicar onde posso encontrar questões do  
assunto Álgebra Booleana?

Agradeço-lhes.


  Veja quais são os assuntos do momento no Yahoo! +Buscados
http://br.maisbuscados.yahoo.com



=
Instruções para entrar na lista, sair da lista e usar a lista em
http://www.mat.puc-rio.br/~obmlistas/obm-l.html
=


[obm-l] Re: FW: Re: [obm-l] Re: [obm-l] Re: [obm-l] [obm-l] Questões de Mat. Básica

2009-05-04 Por tôpico Luciano de Siqueira Pimentel
Bom, vou me retirar da lista, afinal não é pertinente.


[obm-l] Re: [obm-l] Re: FW: Re: [ obm-l] Re: [obm-l] Re: [obm-l] [ obm-l] Questões de Mat. Básica

2009-05-04 Por tôpico Carlos Nehab

Caro Luciano,

Gostaria que você reconsiderasse sua posição, mas que ficasse atento ao 
que de forma muito pertinernte o Fabricio mencionou há 2 dias:


Os participantes da Lista têm enorme prazer em auxiliar , desde que 
fique claro que o solicitante está fazendo esforço para se desenvolver, 
explicite com clareza suas dificuldades, etc.
O que definitivamente eu abomino, por exemplo, é ver listas 
intermináveis de exercícios que, muitas vezes por preguiça de 
resolvê-las, mandam pra cá...


Abraços,
Carlos Nehab


Luciano de Siqueira Pimentel escreveu:

Bom, vou me retirar da lista, afinal não é pertinente.


=
Instruções para entrar na lista, sair da lista e usar a lista em
http://www.mat.puc-rio.br/~obmlistas/obm-l.html
=


[obm-l] RE: [obm-l] Re: [obm-l] Re: [obm -l] [obm-l] Questões de Mat. Bás ica

2009-05-02 Por tôpico Albert Bouskela
Olá Bruno, Paulo e demais colegas desta Lista,

Muito oportunas as mensagens do Bruno [*] e do Paulo. Neste sentido, além de 
compartilhar da mesma opinião, acredito que a solução proposta pelo Paulo seja 
mesmo a mais adequada: é preciso deixar de lado, ao relento, esses problemas 
típicos de exames para a admissão em alguma instituição. Os objetivos - é 
claro! - são: resguardar o propósito desta Lista; não torná-la enfadonha para 
aqueles que, de fato, devem participar deste fórum; evitar que os problemas 
pertinentes sejam poluídos pela mistura com outros mais simples, o que acabaria 
por jogar todas as questões numa vala comum que não despertaria o interesse de 
ninguém, ou quase ninguém etc.

Entretanto, o mais importante é o seguinte: vejo que o Bruno mantém o número 
666, o número da Besta do Apocalipse, no seu endereço de e-mail. Daí, vou 
perguntar ao Bruno:

[*] Na minha modestíssima opinião, já que não sou especialista na interpretação 
de textos bíblicos judaico-cristãos (embora já os tenha lido até que 
detidamente), e, além disto, desconheço como foram criados os números 
cabalísticos, os demônios e as coisas (coisas?) afins, acho que o número da 
Besta deveria ser primo. Explico-me: quando João escreveu o Livro do Apocalipse 
e nele fixou o número da Besta, fixou, na mesma passagem, o número do Senhor, 
7. Veja que o número do Senhor é primo, i.e., indivisível. É, também, o maior 
número primo de um único algarismo - João chega a afirmar que se trata de um 
número perfeito (e João estava iluminado pelo Espírito Santo quando estabeleceu 
esta numerologia toda!). Mas e o 666? 666 pode ser decomposto em 3 fatores 
primos (2, 3 e 37) e tem 10 divisores diferentes (e diferentes de 1 e dele 
próprio). Dá ou não dá o que pensar?

Uma curiosidade bem legal: durante a 2ª Guerra Mundial, os Aliados descobriram 
que se as letras do alfabeto (k incluído) fossem numeradas a partir de 100 
(a=100, b=101, c=102...), a soma correspondente ao nome de Hitler daria 666 
(=107+108+119+111+104+117). Deve ser por causa daquele bigodinho pra lá de 
ridículo que o cara usava.

Saudações a todos,
Albert Bouskela
bousk...@gmail.com
bousk...@ymail.com

 -Original Message-
 From: owner-ob...@mat.puc-rio.br [mailto:owner-ob...@mat.puc-rio.br]
 On Behalf Of Paulo Santa Rita
 Sent: Wednesday, April 29, 2009 8:53 AM
 To: obm-l@mat.puc-rio.br
 Subject: [obm-l] Re: [obm-l] Re: [obm-l] [obm-l] Questões de Mat. Básica
 
 Ola Bruno e demais colegas
 desta lista ... OBM-L,
 
 A mensagem do Bruno e muito boa. Este espaco e uma LISTA DE DISCUSSAO
 DE PROBLEMAS DE MATEMATICA OLIMPICA, nao e lugar para se propor
 problemas de vestibulares ou concursos publicos. Digo isso, em
 primeiro lugar, porque esse era o
 objetivo original deste ambiente, conforme pode se ver na pagina da
 OBM. Se o Prof
 Nicolau nao alterou este objetivo, ele continua o mesmo ... Alem
 disso, estudantes de
 concursos e vestibulares tem inumeros outros espacos na Internet para
 colocarem e discutirem seus problemas especificos, ao contrario dos
 estudantes que se preparam
 para Olimpiadas, com muito poucas opcoes.
 
 Ha alguns anos, estudantes de olimpiadas de diversas partes do Mundo
 assistiam as nossas discussoes. Eu receibia mensagens de alunos de
 paises da America do Sul, dos EUA e da Europa interessados nos nossos
 problemas, discussoes e solucoes. Me lembro que na traducao dos
 problemas russos  :
 
 http://www.mat.puc-rio.br/~nicolau/psr/
 
 Eu precisei disponibilizar a traducao na pagina do Prof Nicolau,
 tantos e tao diversificados eram os pedidos.
 
 E o que estamos vendo agora ? A nossa tao estimada lista cheia de
 problemas triviais, altamente distantes do ideal olimpico e verdadeira
 fonte de solucoes para alunos preguicosos que nao querem pensar. Isso
 afugenta os alunos serios, os Prof's competentes e muitas
 outras pessoas que poderiam estar colocando aqui belas questoes e
 belas solucoes, ajudando assim aquele nosso amigo de um estado
 distante, que gostaria de se preparar para as
 Olimpiadas de Matematica e que nao dispoe de locais de treinamento
 proximo as suas casas.
 
 A maneira mais sabia de combater estas coisas, eu penso, e nao
 responder a estas questoes, desestimulando assim aqueles que estao,
 conscientes ou nao, desvirtuando este espaco de seu belo ideal
 original.
 
 Um abraco a Todos
 PSR, 42904090841
 
 EM TEMPO : O Euler nos ensinou a calcular a soma dos inversos dos
 quadrados dos numeros naturais. Nomeadamente ele mostrou que :
 
 1 + (1/2)^2 + (1/3)^2 + ... = (pi)^2 /6
 
 Mas tambem e verdade que ele tentou somar os inversos dos cubos dos
 numeros naturais sem sucesso. Parece mesmo que esta soma ainda hoje e
 um problema em aberto. Pois bem.  Expresse
 
 T = 1 + (1/2)^3 + (1/3)^3 + ...
 
 Como uma soma de numeros binomiais na qual NENHUM dos numeros
 binomias
 aparece em denominador ou elevado a potencias diferentes de 1.
 
 
 2009/4/29 Bruno França dos Reis bfr...@gmail.com:
  Luciano, teoricamente esta lista tem

Re: FW: Re: [obm-l] Re: [obm-l] Re: [obm-l] [obm-l] Questões de Mat. Básica

2009-05-02 Por tôpico Carlos Nehab
 
Besta do Apocalipse, no seu endereço de e-mail. Daí, vou perguntar ao Bruno:




[*] Na minha modestíssima opinião, já que não 
sou especialista na interpretação de textos 
bíblicos judaico-cristãos (embora já os tenha 
lido até que detidamente), e, além disto, 
desconheço como foram criados os números 
cabalísticos, os demônios e as coisas (coisas?) 
afins, acho que o número da Besta deveria ser 
primo. Explico-me: quando João escreveu o Livro 
do Apocalipse e nele fixou o número da Besta, 
fixou, na mesma passagem, o número do Senhor, 7. 
Veja que o número do Senhor é primo, i.e., 
indivisível. É, também, o maior número primo de 
um único algarismo – João chega a afirmar que se 
trata de um número perfeito (e João estava 
iluminado pelo Espírito Santo quando estabeleceu 
esta numerologia toda!). Mas e o 666? 666 pode 
ser decomposto em 3 fatores primos (2, 3 e 37) e 
tem 10 divisores diferentes (e diferentes de 1 e 
dele próprio). Dá ou não dá o que pensar?




Uma curiosidade bem legal: durante a 2ª Guerra 
Mundial, os Aliados descobriram que se as letras 
do alfabeto (a letra “k” incluída) fossem 
numeradas a partir de 100 (a=100, b=101, 
c=102...), a soma correspondente ao nome de 
Hitler resultaria igual a 666 
(=107+108+119+111+104+117). Deve ser por causa 
daquele bigodinho pra lá de ridículo que o cara usava.




Bem, agora que já fiz as minhas gracinhas, vou 
esquecer minha polidez e dar um conselho ao Luciano:




Meu caro,



Antes de tudo, quero deixar bem claro que não 
quero ofendê-lo e nem menosprezá-lo! Encare o 
que vou lhe dizer como sendo as palavras 
(amargas, eu sei) de quem atua na Consultoria de 
Engenharia há mais de 30 anos. Lá vai:




Não é possível (i.e., não deveria ser) um aluno 
chegar ao final do ensino, hodiernamente chamado 
de “médio”, sem saber resolver – com bastante 
facilidade – o problema adiante. Isto se torna 
uma deficiência ainda mais aguda, quando esse 
aluno (esse é, em princípio, o seu caso) postula 
uma vaga numa Faculdade de Engenharia. Lá (na 
Faculdade de Engenharia), logo na 1ª aula de 
Cálculo I, no 1º período do Ciclo Básico, um 
desafortunado professor vai tentar lhe ensinar o 
conceito de “limite” e, depois, as operações de 
derivação e integração e daí pra frente... É por 
isso que, nas melhores Faculdades de Engenharia 
do Brasil, o índice de reprovação em Cálculo I é 
bem superior a 50% e o abandono do Curso, 
durante o Ciclo Básico, também ultrapassa 
bastante esse percentual. A frustração dos 
alunos, claramente despreparados para as 
disciplinas de Cálculo e Física, torna-se 
insuportável e assim, sem outra opção, esses 
alunos são forçados a perder um ou dois anos da 
sua escolarização, indo tentar a graduação em outra área do conhecimento.




É evidente que o sistema brasileiro de educação 
(se é que existe este sistema!) deveria cuidar 
disso, impedindo o acesso de um aluno 
despreparado ao nível superior do ensino – mas 
assim, infelizmente, não é! Talvez até já tenha sido, mas há décadas atrás...




Portanto, peço que você pondere se é mesmo a 
Engenharia a formação profissional que você 
almeja e se, para esta formação, você está minimamente preparado.




Perdoe-me a dureza! Desejo-lhe, sinceramente, sucesso!

“Um lojista sabe que, para não ter prejuízo, o 
preço de venda de seus produtos deve ser no 
mínimo 44% superior ao preço de custo. Porém ele 
prepara a tabela de preços de venda 
acrescentando 80% ao preço de custo, porque sabe 
que o cliente gosta de obter desconto no momento 
da compra. Qual o maior desconto que ele pode 
conceder ao cliente, sobre o preço de tabela, de modo a não ter prejuízo?


A) 10%

B) 15%

C) 20%

D) 25%

E) 36%”



Saudações a todos,

Albert Bouskela

mailto:bousk...@gmail.combousk...@gmail.com

mailto:bousk...@ymail.combousk...@ymail.com
--- Em qua, 29/4/09, Paulo Santa Rita 
mailto:paulo.santar...@gmail.compaulo.santar...@gmail.com escreveu:


De: Paulo Santa Rita 
mailto:paulo.santar...@gmail.compaulo.santar...@gmail.com

Assunto: [obm-l] Re: [obm-l] Re: [obm-l] [obm-l] Questões de Mat. Básica
Para: mailto:obm-l@mat.puc-rio.brobm-l@mat.puc-rio.br
Data: Quarta-feira, 29 de Abril de 2009, 11:53

Ola Bruno e demais colegas
desta lista ... OBM-L,

A mensagem do Bruno e muito boa. Este espaco e uma LISTA DE DISCUSSAO
DE PROBLEMAS DE MATEMATICA OLIMPICA, nao e lugar para se propor
problemas de vestibulares ou concursos publicos. Digo isso, em
primeiro lugar, porque esse era o
objetivo original deste ambiente, conforme pode se ver na pagina da
OBM. Se o Prof
Nicolau nao alterou este objetivo, ele continua o mesmo ... Alem
disso, estudantes de
concursos e vestibulares tem inumeros outros espacos na Internet para
colocarem e discutirem seus problemas especificos, ao contrario dos
estudantes que se preparam
para Olimpiadas, com muito poucas opcoes.

Ha alguns anos, estudantes de olimpiadas de diversas partes do Mundo
assistiam as nossas discussoes. Eu receibia mensagens de alunos de
paises da America do Sul

[obm-l] [obm-l] Questões de Mat. Básica

2009-04-29 Por tôpico Luciano de Siqueira Pimentel
1) Numa certa cidade, foi adotado o seguinte sistema de rodízio de carros:
duas vezes por semana, de segunda a sexta, cada carro fica proibido de
circular, de acordo com o final de sua placa (alg. das unidades). O número
médio de finais de placa proibidos diferentes para cada dia de proibição é:
A) 4
B) 1
C) 3
D) 2
E) indefinido

2) Um lojista sabe que, para não ter prejuízo, o preço de venda de seus
produtos deve ser no mínimo 44% superior ao preço de custo. Porém ele
prepara a tabela de preços de venda acrescentando 80% ao preço de custo,
porque sabe que o cliente gosta de obter desconto no momento da compra. Qual
o maior desconto que ele pode conceder ao cliente, sobre o preço de tabela,
de modo a não ter prejuízo?
A) 10%
B) 15%
C) 20%
D) 25%
E) 36%

3) O número de soluções reais da equação x^2 = 2^x é:
A) 0
B) 1
C) 2
D) 3
E) 4

P.S.: Nessa questão aí eu só achei 2 soluções: x=2 ou x=4

Gostaria de saber mais ou menos como funciona a lista. Estou me preparando
para o vest. do ITA, portanto gostaria de participar da lista mandando
outros tipos de exercícios (de Matemática, é claro). Eu poderia fazer isso
ou seria muito inconveniente?
Abraços!


[obm-l] Re: [obm-l] [obm-l] Questões de Mat. Básica

2009-04-29 Por tôpico Bruno França dos Reis
Luciano, teoricamente esta lista tem por objetivo a discussão de problemas
olímpicos (afinal de contas, veja o nome da lista), e não a resolução de
lista de exercícios. Questões mais abertas, que exigem mais reflexão do que
simplesmente cálculo bobo, são sempre bem recebidas pela maioria.
Infelizmente, nos últimos tempos a lista tem se transformado nisso. Há
pessoas que só fazem isso por aqui, colocam suas listas de exercícios para
que os outros resolvam, sem nem sequer colocar uma mensagem (começa com a
lista de exercícios e assina em baixo, com um apelido).

Finalmente, muitos dos problemas colocados aqui já foram discutidos, e estão
nos arquivos da lista. Claro que se o intuito for promover uma nova
abordagem a um problema antigo, isso é fantástico. Agora, só pra saber a
resposta, ou para discutir a mesma coisa, seria preferível consultar os
arquivos.

Bruno

--
Bruno FRANÇA DOS REIS

msn: brunoreis...@hotmail.com
skype: brunoreis666
tel: +33 (0)6 28 43 42 16

http://brunoreis.com
http://blog.brunoreis.com

GPG Key: http://brunoreis.com/bruno-public.key

e^(pi*i)+1=0


2009/4/29 Luciano de Siqueira Pimentel luciano@gmail.com

 1) Numa certa cidade, foi adotado o seguinte sistema de rodízio de carros:
 duas vezes por semana, de segunda a sexta, cada carro fica proibido de
 circular, de acordo com o final de sua placa (alg. das unidades). O número
 médio de finais de placa proibidos diferentes para cada dia de proibição é:
 A) 4
 B) 1
 C) 3
 D) 2
 E) indefinido

 2) Um lojista sabe que, para não ter prejuízo, o preço de venda de seus
 produtos deve ser no mínimo 44% superior ao preço de custo. Porém ele
 prepara a tabela de preços de venda acrescentando 80% ao preço de custo,
 porque sabe que o cliente gosta de obter desconto no momento da compra. Qual
 o maior desconto que ele pode conceder ao cliente, sobre o preço de tabela,
 de modo a não ter prejuízo?
 A) 10%
 B) 15%
 C) 20%
 D) 25%
 E) 36%

 3) O número de soluções reais da equação x^2 = 2^x é:
 A) 0
 B) 1
 C) 2
 D) 3
 E) 4

 P.S.: Nessa questão aí eu só achei 2 soluções: x=2 ou x=4

 Gostaria de saber mais ou menos como funciona a lista. Estou me preparando
 para o vest. do ITA, portanto gostaria de participar da lista mandando
 outros tipos de exercícios (de Matemática, é claro). Eu poderia fazer isso
 ou seria muito inconveniente?
 Abraços!





[obm-l] Re: [obm-l] Re: [obm-l] [obm-l] Questões de Mat. B ásica

2009-04-29 Por tôpico Paulo Santa Rita
Ola Bruno e demais colegas
desta lista ... OBM-L,

A mensagem do Bruno e muito boa. Este espaco e uma LISTA DE DISCUSSAO
DE PROBLEMAS DE MATEMATICA OLIMPICA, nao e lugar para se propor
problemas de vestibulares ou concursos publicos. Digo isso, em
primeiro lugar, porque esse era o
objetivo original deste ambiente, conforme pode se ver na pagina da
OBM. Se o Prof
Nicolau nao alterou este objetivo, ele continua o mesmo ... Alem
disso, estudantes de
concursos e vestibulares tem inumeros outros espacos na Internet para
colocarem e discutirem seus problemas especificos, ao contrario dos
estudantes que se preparam
para Olimpiadas, com muito poucas opcoes.

Ha alguns anos, estudantes de olimpiadas de diversas partes do Mundo
assistiam as nossas discussoes. Eu receibia mensagens de alunos de
paises da America do Sul, dos EUA e da Europa interessados nos nossos
problemas, discussoes e solucoes. Me lembro que na traducao dos
problemas russos  :

http://www.mat.puc-rio.br/~nicolau/psr/

Eu precisei disponibilizar a traducao na pagina do Prof Nicolau,
tantos e tao diversificados eram os pedidos.

E o que estamos vendo agora ? A nossa tao estimada lista cheia de
problemas triviais, altamente distantes do ideal olimpico e verdadeira
fonte de solucoes para alunos preguicosos que nao querem pensar. Isso
afugenta os alunos serios, os Prof's competentes e muitas
outras pessoas que poderiam estar colocando aqui belas questoes e
belas solucoes, ajudando assim aquele nosso amigo de um estado
distante, que gostaria de se preparar para as
Olimpiadas de Matematica e que nao dispoe de locais de treinamento
proximo as suas casas.

A maneira mais sabia de combater estas coisas, eu penso, e nao
responder a estas questoes, desestimulando assim aqueles que estao,
conscientes ou nao, desvirtuando este espaco de seu belo ideal
original.

Um abraco a Todos
PSR, 42904090841

EM TEMPO : O Euler nos ensinou a calcular a soma dos inversos dos
quadrados dos numeros naturais. Nomeadamente ele mostrou que :

1 + (1/2)^2 + (1/3)^2 + ... = (pi)^2 /6

Mas tambem e verdade que ele tentou somar os inversos dos cubos dos
numeros naturais sem sucesso. Parece mesmo que esta soma ainda hoje e
um problema em aberto. Pois bem.  Expresse

T = 1 + (1/2)^3 + (1/3)^3 + ...

Como uma soma de numeros binomiais na qual NENHUM dos numeros binomias
aparece em denominador ou elevado a potencias diferentes de 1.


2009/4/29 Bruno França dos Reis bfr...@gmail.com:
 Luciano, teoricamente esta lista tem por objetivo a discussão de problemas
 olímpicos (afinal de contas, veja o nome da lista), e não a resolução de
 lista de exercícios. Questões mais abertas, que exigem mais reflexão do que
 simplesmente cálculo bobo, são sempre bem recebidas pela maioria.
 Infelizmente, nos últimos tempos a lista tem se transformado nisso. Há
 pessoas que só fazem isso por aqui, colocam suas listas de exercícios para
 que os outros resolvam, sem nem sequer colocar uma mensagem (começa com a
 lista de exercícios e assina em baixo, com um apelido).
 Finalmente, muitos dos problemas colocados aqui já foram discutidos, e estão
 nos arquivos da lista. Claro que se o intuito for promover uma nova
 abordagem a um problema antigo, isso é fantástico. Agora, só pra saber a
 resposta, ou para discutir a mesma coisa, seria preferível consultar os
 arquivos.
 Bruno

 --
 Bruno FRANÇA DOS REIS

 msn: brunoreis...@hotmail.com
 skype: brunoreis666
 tel: +33 (0)6 28 43 42 16

 http://brunoreis.com
 http://blog.brunoreis.com

 GPG Key: http://brunoreis.com/bruno-public.key

 e^(pi*i)+1=0


 2009/4/29 Luciano de Siqueira Pimentel luciano@gmail.com

 1) Numa certa cidade, foi adotado o seguinte sistema de rodízio de carros:
 duas vezes por semana, de segunda a sexta, cada carro fica proibido de
 circular, de acordo com o final de sua placa (alg. das unidades). O número
 médio de finais de placa proibidos diferentes para cada dia de proibição é:
 A) 4
 B) 1
 C) 3
 D) 2
 E) indefinido

 2) Um lojista sabe que, para não ter prejuízo, o preço de venda de seus
 produtos deve ser no mínimo 44% superior ao preço de custo. Porém ele
 prepara a tabela de preços de venda acrescentando 80% ao preço de custo,
 porque sabe que o cliente gosta de obter desconto no momento da compra. Qual
 o maior desconto que ele pode conceder ao cliente, sobre o preço de tabela,
 de modo a não ter prejuízo?
 A) 10%
 B) 15%
 C) 20%
 D) 25%
 E) 36%

 3) O número de soluções reais da equação x^2 = 2^x é:
 A) 0
 B) 1
 C) 2
 D) 3
 E) 4

 P.S.: Nessa questão aí eu só achei 2 soluções: x=2 ou x=4

 Gostaria de saber mais ou menos como funciona a lista. Estou me preparando
 para o vest. do ITA, portanto gostaria de participar da lista mandando
 outros tipos de exercícios (de Matemática, é claro). Eu poderia fazer isso
 ou seria muito inconveniente?
 Abraços!





=
Instru��es para entrar na lista, sair da lista e usar a lista em

[obm-l] RE: [obm-l] RE: [obm -l] Re: [obm-l] Re: [obm-l] Questões de Combinatória. (ajuda)

2009-04-28 Por tôpico Jordan Piva

Tudo bem, isso acontece. Espero ter ajudado tb.

Abrcs

From: joao_maldona...@hotmail.com
To: obm-l@mat.puc-rio.br
Subject: [obm-l] RE: [obm-l] Re: [obm-l] Re: [obm-l] Questões de Combinatória. 
(ajuda)
Date: Sun, 26 Apr 2009 10:17:09 -0300








Obrigado Rafael e Jordan, foi uma completa falta de desatenção mesmo, acho que 
eu estava com pressa indo para o show do cézar menotti e fabiano que nem 
percebi os erros (só pra você ver na q.4, contei a solução 2^6.3^6 e não contei 
2^6 nem 3^6). Desculpe pelos erros Vinícius, não vai acontecer de novo.

Abraço

From: rafael.a...@gmail.com
Date: Sat, 25 Apr 2009 13:42:05 -0300
Subject: [obm-l] Re: [obm-l] Re: [obm-l] Questões de Combinatória. (ajuda)
To: obm-l@mat.puc-rio.br

Existem mais possibilidades a serem removidas na questão 4... Sabemos que se um 
número é, simultaneamente, um quadrado perfeito e um cubo perfeito, então ele é 
uma sexta potência.

Logo, basta remover todas as sextas potências de 1 a 100=10^6, ou seja, 
remover 10:



Então temos: 1000 + 100 - 10 = 1090.

Como o problema pergunta quantos números NÃO são quadrados nem cubos, a 
resposta é 100-1090 = 998910.

2009/4/24 Joao Maldonado joao_maldonad...@yahoo.com.br




Ola Vinícius, aí vai...

1.) O número não vai começar com 0 e o número deve começar com 53, 54, 56, 57, 
6 ou 7.
53, 54, 56 ou 57 - 4.6!/3!
6 ou 7 - 2.7!/3!
Total = 6.5.4.(4+2.7) = 120.18 = 2160 possibilidades.



2.)
6! = 720 posibilidades (porém nesse resultado o mesmo cubo pode ser
encontrado de 6 maneiras somente fazendo uma rotação de um outro cubo),
caso contrário seriam 6!/6 = 5! = 120 possibilidades

3.) a) n!
b)
Caso a minha interpretação esteja correta como voxê colocou a conjunção
e ao invés da ou no final da frase, não poderia acontecer as 3
coisas SIMULTANEAMENTE, ou seja, o primeiro lugar ser o número 1, o
segundo o número 2 e o terceiro o número 4 é uma possibilidade válida.
Consequentementeteríamos (n-3)! possibilidades da corrida terminar com
1-2-3, assim a resposta é: n! - (n-3)!

4.) Esse quatro é mais legalzinho.
OK, quadrado perfeito: 1² = 1 e 1000² = 100 - Teremos 1000 quadrados 
perfeitos.
cubos perfeitos - 1³ = 1 e 100³ = 100 - Teremos 100 cubos perfeitos.


Toda
quarta potência é um quadrado então consequentemente podemos ignorar
esta opção. Temos que tirar os casos em que x² = y³ - ou seja, x =
a1^6k.a2^6k...an^6k e y = b1^6k.b2^6k...bn^6k para todo ai e bi primos
(além da solução x=1).
Temos no máximo x ou y produto das potências de 2 primos pois 2^6.3^6.5^6  
100
Temos k = 1 pois: 2^12.3^12  100
Possibilidades: (1) ; 2^6.3^6 = (46656) ; 2^6.5^6 = (100)


Total = 1000 + 100 - 3 = 1097 possiblidades.

Abraço,

João

--- Em sex, 24/4/09, Vinícius pvni...@gmail.com escreveu:



De: Vinícius pvni...@gmail.com
Assunto: [obm-l] Questões de Combinatória. (ajuda)
Para: obm-l@mat.puc-rio.br


Data: Sexta-feira, 24 de Abril de 2009, 16:21

1.  Quantos números inteiros de cinco algarismos distintos e maiores do que 
53.000 podemser formados com os algarismos 0, 1, 2, 3, 4, 5, 6 e 7?


2.  De quantos modos se pode pintar um cubo, usando seis cores fixas
 distintas, sendo cada
face de uma cor?
3.  Em uma corrida há n participantes. Antes de a corrida começar, cada 
participante recebeum número entre 1 e n.a) De quantas maneiras diferentes os 
participantes podem terminar a corrida?


b) De quantas maneiras o 1o lugar NÃO é o participante número 1, o 2o lugar NÃO 
é oparticipante número 2 e o 3o lugar NÃO é o participante número 3?
4. Quantos inteiros entre 1 e 100, inclusive, não são quadrados perfeitos, 
nem cubos


perfeitos,nem quartas potências perfeitas?




  Veja quais são os assuntos do momento no Yahoo! + Buscados: Top 10 - 
Celebridades - Música - Esportes



-- 
Rafael

Quer saber qual produto Windows Live combina melhor com o seu perfil? Clique 
aqui e descubra!
_
Descubra seu lado desconhecido com o novo Windows Live!
http://www.windowslive.com.br

[obm-l] RE: [obm-l] Questões de Combinatória. (a juda)

2009-04-27 Por tôpico João Maldonado

Ola Vinícius, aí vai...

1.) O número não vai começar com 0 e o número deve começar com 53, 54, 56, 57, 
6 ou 7.
53, 54, 56 ou 57 - 4.6!/3!
6 ou 7 - 2.7!/3!
Total = 6.5.4.(4+2.7) = 120.18 = 2160 possibilidades.

2.) 6! = 720 posibilidades (porém nesse resultado o mesmo cubo pode ser 
encontrado de 6 maneiras somente fazendo uma rotação de um outro cubo), caso 
contrário seriam 6!/6 = 5! = 120 possibilidades

3.) a) n!
b) Caso a minha interpretação esteja correta como voxê colocou a conjunção e 
ao invés da ou no final da frase, não poderia acontecer as 3 coisas 
SIMULTANEAMENTE, ou seja, o primeiro lugar ser o número 1, o segundo o número 2 
e o terceiro o número 4 é uma possibilidade válida. Consequentementeteríamos 
(n-3)! possibilidades da corrida terminar com 1-2-3, assim a resposta é: n! - 
(n-3)!

4.) Esse quatro é mais legalzinho.
OK, quadrado perfeito: 1² = 1 e 1000² = 100 - Teremos 1000 quadrados 
perfeitos.
cubos perfeitos - 1³ = 1 e 100³ = 100 - Teremos 100 cubos perfeitos.
Toda quarta potência é um quadrado então consequentemente podemos ignorar esta 
opção. Temos que tirar os casos em que x² = y³ - ou seja, x = 
a1^6k.a2^6k...an^6k e y = b1^6k.b2^6k...bn^6k para todo ai e bi primos (além da 
solução x=1).
Temos no máximo x ou y produto das potências de 2 primos pois 2^6.3^6.5^6  
100
Temos k = 1 pois: 2^12.3^12  100
Possibilidades: (1) ; 2^6.3^6 = (46656) ; 2^6.5^6 = (100)
Total = 1000 + 100 - 3 = 1097 possiblidades.

Abraço,

João


Date: Fri, 24 Apr 2009 13:21:05 -0300
Subject: [obm-l] Questões de Combinatória. (ajuda)
From: pvni...@gmail.com
To: obm-l@mat.puc-rio.br

1.  Quantos números inteiros de cinco algarismos distintos e maiores do que 
53.000 podemser formados com os algarismos 0, 1, 2, 3, 4, 5, 6 e 7?
2.  De quantos modos se pode pintar um cubo, usando seis cores fixas distintas, 
sendo cada
face de uma cor?
3.  Em uma corrida há n participantes. Antes de a corrida começar, cada 
participante recebeum número entre 1 e n.a) De quantas maneiras diferentes os 
participantes podem terminar a corrida?
b) De quantas maneiras o 1o lugar NÃO é o participante número 1, o 2o lugar NÃO 
é oparticipante número 2 e o 3o lugar NÃO é o participante número 3?
4. Quantos inteiros entre 1 e 100, inclusive, não são quadrados perfeitos, 
nem cubos
perfeitos,nem quartas potências perfeitas?
_
Faça já uma busa e ganhe um wink do Messenger. Está esperando o que? É grátis!
http://www.ibud.com.br/

[obm-l] Re: [obm-l] RE: [obm-l ] Questões de Combin atória . (ajuda)

2009-04-27 Por tôpico Vinícius
Que isso João, os erros acontecem. Muito obrigado pela força galera!grande
abraço


[obm-l] RE: [obm-l] Re: [obm -l] Questões de Comb inatória. (ajuda)

2009-04-25 Por tôpico Jordan Piva

Bem Joao ha um erro na sua solucao p/ questao 2, veja que na realidade teríamos 
inicialmente  6!=720 formas porém você tem que descontar as rotacoes do cubo. 
Fixe uma face (como se estivesse segurando o cubo com uma face em sua direção). 
Com esta face fixa voltada para você quantas rotações podemos fazer? quatro. 
Como podemos fixar qualquer uma das 6 faces temos um total de 6x4=24 casos 
repetidos a serem desconsiderados assim:

 

Resp.: 720/24=30

 

 


Date: Fri, 24 Apr 2009 16:20:33 -0700
From: joao_maldonad...@yahoo.com.br
Subject: [obm-l] Re: [obm-l] Questões de Combinatória. (ajuda)
To: obm-l@mat.puc-rio.br





Ola Vinícius, aí vai...

1.) O número não vai começar com 0 e o número deve começar com 53, 54, 56, 57, 
6 ou 7.
53, 54, 56 ou 57 - 4.6!/3!
6 ou 7 - 2.7!/3!
Total = 6.5.4.(4+2.7) = 120.18 = 2160 possibilidades.

2.) 6! = 720 posibilidades (porém nesse resultado o mesmo cubo pode ser 
encontrado de 6 maneiras somente fazendo uma rotação de um outro cubo), caso 
contrário seriam 6!/6 = 5! = 120 possibilidades

3.) a) n!
b) Caso a minha interpretação esteja correta como voxê colocou a conjunção e 
ao invés da ou no final da frase, não poderia acontecer as 3 coisas 
SIMULTANEAMENTE, ou seja, o primeiro lugar ser o número 1, o segundo o número 2 
e o terceiro o número 4 é uma possibilidade válida. Consequentementeteríamos 
(n-3)! possibilidades da corrida terminar com 1-2-3, assim a resposta é: n! - 
(n-3)!

4.) Esse quatro é mais legalzinho.
OK, quadrado perfeito: 1² = 1 e 1000² = 100 - Teremos 1000 quadrados 
perfeitos.
cubos perfeitos - 1³ = 1 e 100³ = 100 - Teremos 100 cubos perfeitos.
Toda quarta potência é um quadrado então consequentemente podemos ignorar esta 
opção. Temos que tirar os casos em que x² = y³ - ou seja, x = 
a1^6k.a2^6k...an^6k e y = b1^6k.b2^6k...bn^6k para todo ai e bi primos (além da 
solução x=1).
Temos no máximo x ou y produto das potências de 2 primos pois 2^6.3^6.5^6  
100
Temos k = 1 pois: 2^12.3^12  100
Possibilidades: (1) ; 2^6.3^6 = (46656) ; 2^6.5^6 = (100)
Total = 1000 + 100 - 3 = 1097 possiblidades.

Abraço,

João

--- Em sex, 24/4/09, Vinícius pvni...@gmail.com escreveu:


De: Vinícius pvni...@gmail.com
Assunto: [obm-l] Questões de Combinatória. (ajuda)
Para: obm-l@mat.puc-rio.br
Data: Sexta-feira, 24 de Abril de 2009, 16:21



1.  Quantos números inteiros de cinco algarismos distintos e maiores do que 
53.000 podem
ser formados com os algarismos 0, 1, 2, 3, 4, 5, 6 e 7?


2.  De quantos modos se pode pintar um cubo, usando seis cores fixas distintas, 
sendo cada
face de uma cor?


3.  Em uma corrida há n participantes. Antes de a corrida começar, cada 
participante recebe
um número entre 1 e n.
a) De quantas maneiras diferentes os participantes podem terminar a corrida?
b) De quantas maneiras o 1o lugar NÃO é o participante número 1, o 2o lugar NÃO 
é o
participante número 2 e o 3o lugar NÃO é o participante número 3?


4. Quantos inteiros entre 1 e 100, inclusive, não são quadrados perfeitos, 
nem cubos
perfeitos,nem quartas potências perfeitas?


Veja quais são os assuntos do momento no Yahoo! + Buscados: Top 10 - 
Celebridades - Música - Esportes
_
Messenger 2009: Instale já!
http://download.live.com

[obm-l] Re: [obm-l] Re: [obm-l] Questões de Combinatória. (ajuda)

2009-04-25 Por tôpico Rafael Ando
Existem mais possibilidades a serem removidas na questão 4... Sabemos que se
um número é, simultaneamente, um quadrado perfeito e um cubo perfeito, então
ele é uma sexta potência.

Logo, basta remover todas as sextas potências de 1 a 100=10^6, ou seja,
remover 10:

Então temos: 1000 + 100 - 10 = 1090.

Como o problema pergunta quantos números NÃO são quadrados nem cubos, a
resposta é 100-1090 = 998910.

2009/4/24 Joao Maldonado joao_maldonad...@yahoo.com.br

 Ola Vinícius, aí vai...

 1.) O número não vai começar com 0 e o número deve começar com 53, 54, 56,
 57, 6 ou 7.
 53, 54, 56 ou 57 - 4.6!/3!
 6 ou 7 - 2.7!/3!
 Total = 6.5.4.(4+2.7) = 120.18 = 2160 possibilidades.

 2.) 6! = 720 posibilidades (porém nesse resultado o mesmo cubo pode ser
 encontrado de 6 maneiras somente fazendo uma rotação de um outro cubo), caso
 contrário seriam 6!/6 = 5! = 120 possibilidades

 3.) a) n!
 b) Caso a minha interpretação esteja correta como voxê colocou a conjunção
 e ao invés da ou no final da frase, não poderia acontecer as 3 coisas
 SIMULTANEAMENTE, ou seja, o primeiro lugar ser o número 1, o segundo o
 número 2 e o terceiro o número 4 é uma possibilidade válida.
 Consequentementeteríamos (n-3)! possibilidades da corrida terminar com
 1-2-3, assim a resposta é: n! - (n-3)!

 4.) Esse quatro é mais legalzinho.
 OK, quadrado perfeito: 1² = 1 e 1000² = 100 - Teremos 1000 quadrados
 perfeitos.
 cubos perfeitos - 1³ = 1 e 100³ = 100 - Teremos 100 cubos perfeitos.
 Toda quarta potência é um quadrado então consequentemente podemos ignorar
 esta opção. Temos que tirar os casos em que x² = y³ - ou seja, x =
 a1^6k.a2^6k...an^6k e y = b1^6k.b2^6k...bn^6k para todo ai e bi primos (além
 da solução x=1).
 Temos no máximo x ou y produto das potências de 2 primos pois 2^6.3^6.5^6 
 100
 Temos k = 1 pois: 2^12.3^12  100
 Possibilidades: (1) ; 2^6.3^6 = (46656) ; 2^6.5^6 = (100)
 Total = 1000 + 100 - 3 = 1097 possiblidades.

 Abraço,

 João

 --- Em *sex, 24/4/09, Vinícius pvni...@gmail.com* escreveu:


 De: Vinícius pvni...@gmail.com
 Assunto: [obm-l] Questões de Combinatória. (ajuda)
 Para: obm-l@mat.puc-rio.br
 Data: Sexta-feira, 24 de Abril de 2009, 16:21


 1.  Quantos números inteiros de cinco algarismos distintos e maiores do que
 53.000 podem
 ser formados com os algarismos 0, 1, 2, 3, 4, 5, 6 e 7?

 2.  De quantos modos se pode pintar um cubo, usando seis cores fixas
 distintas, sendo cada
 face de uma cor?

 3.  Em uma corrida há n participantes. Antes de a corrida começar, cada
 participante recebe
 um número entre 1 e n.
 a) De quantas maneiras diferentes os participantes podem terminar a
 corrida?
 b) De quantas maneiras o 1o lugar NÃO é o participante número 1, o 2o lugar
 NÃO é o
 participante número 2 e o 3o lugar NÃO é o participante número 3?

 4. Quantos inteiros entre 1 e 100, inclusive, não são quadrados
 perfeitos, nem cubos
 perfeitos,nem quartas potências perfeitas?


 --
 Veja quais são os assuntos do momento no Yahoo! + Buscados: Top 
 10http://br.rd.yahoo.com/mail/taglines/mail/*http://br.maisbuscados.yahoo.com/-
 Celebridadeshttp://br.rd.yahoo.com/mail/taglines/mail/*http://br.maisbuscados.yahoo.com/celebridades/-
 Músicahttp://br.rd.yahoo.com/mail/taglines/mail/*http://br.maisbuscados.yahoo.com/m%C3%BAsica/-
 Esporteshttp://br.rd.yahoo.com/mail/taglines/mail/*http://br.maisbuscados.yahoo.com/esportes/




-- 
Rafael


[obm-l] Questões de Combinatória. (ajuda)

2009-04-24 Por tôpico Vinícius
1.  Quantos números inteiros de cinco algarismos distintos e maiores do que
53.000 podem
ser formados com os algarismos 0, 1, 2, 3, 4, 5, 6 e 7?

2.  De quantos modos se pode pintar um cubo, usando seis cores fixas
distintas, sendo cada
face de uma cor?

3.  Em uma corrida há n participantes. Antes de a corrida começar, cada
participante recebe
um número entre 1 e n.
a) De quantas maneiras diferentes os participantes podem terminar a corrida?
b) De quantas maneiras o 1o lugar NÃO é o participante número 1, o 2o lugar
NÃO é o
participante número 2 e o 3o lugar NÃO é o participante número 3?

4. Quantos inteiros entre 1 e 100, inclusive, não são quadrados
perfeitos, nem cubos
perfeitos,nem quartas potências perfeitas?


[obm-l] Re: [obm-l] Questões de Combinatória. (ajuda )

2009-04-24 Por tôpico Joao Maldonado
Ola Vinícius, aí vai...

1.) O número não vai começar com 0 e o número deve começar com 53, 54, 56, 57, 
6 ou 7.
53, 54, 56 ou 57 - 4.6!/3!
6 ou 7 - 2.7!/3!
Total = 6.5.4.(4+2.7) = 120.18 = 2160 possibilidades.

2.)
6! = 720 posibilidades (porém nesse resultado o mesmo cubo pode ser
encontrado de 6 maneiras somente fazendo uma rotação de um outro cubo),
caso contrário seriam 6!/6 = 5! = 120 possibilidades

3.) a) n!
b)
Caso a minha interpretação esteja correta como voxê colocou a conjunção
e ao invés da ou no final da frase, não poderia acontecer as 3
coisas SIMULTANEAMENTE, ou seja, o primeiro lugar ser o número 1, o
segundo o número 2 e o terceiro o número 4 é uma possibilidade válida.
Consequentementeteríamos (n-3)! possibilidades da corrida terminar com
1-2-3, assim a resposta é: n! - (n-3)!

4.) Esse quatro é mais legalzinho.
OK, quadrado perfeito: 1² = 1 e 1000² = 100 - Teremos 1000 quadrados 
perfeitos.
cubos perfeitos - 1³ = 1 e 100³ = 100 - Teremos 100 cubos perfeitos.
Toda
quarta potência é um quadrado então consequentemente podemos ignorar
esta opção. Temos que tirar os casos em que x² = y³ - ou seja, x =
a1^6k.a2^6k...an^6k e y = b1^6k.b2^6k...bn^6k para todo ai e bi primos
(além da solução x=1).
Temos no máximo x ou y produto das potências de 2 primos pois 2^6.3^6.5^6  
100
Temos k = 1 pois: 2^12.3^12  100
Possibilidades: (1) ; 2^6.3^6 = (46656) ; 2^6.5^6 = (100)
Total = 1000 + 100 - 3 = 1097 possiblidades.

Abraço,

João

--- Em sex, 24/4/09, Vinícius pvni...@gmail.com escreveu:

De: Vinícius pvni...@gmail.com
Assunto: [obm-l] Questões de Combinatória. (ajuda)
Para: obm-l@mat.puc-rio.br
Data: Sexta-feira, 24 de Abril de 2009, 16:21

1.  Quantos números inteiros de cinco algarismos distintos e maiores do que 
53.000 podemser formados com os algarismos 0, 1, 2, 3, 4, 5, 6 e 7?
2.  De quantos modos se pode pintar um cubo, usando seis cores fixas distintas, 
sendo cada
face de uma cor?
3.  Em uma corrida há n participantes. Antes de a corrida começar, cada 
participante recebeum número entre 1 e n.a) De quantas maneiras diferentes os 
participantes podem terminar a corrida?
b) De quantas maneiras o 1o lugar NÃO é o participante número 1, o 2o lugar NÃO 
é oparticipante número 2 e o 3o lugar NÃO é o participante número 3?
4. Quantos inteiros entre 1 e 100, inclusive, não são quadrados perfeitos, 
nem cubos
perfeitos,nem quartas potências perfeitas?



  Veja quais são os assuntos do momento no Yahoo! +Buscados
http://br.maisbuscados.yahoo.com

[obm-l] Re: [obm-l] questões LIVRO DE ANÁLISE DO ELON V1

2009-03-31 Por tôpico Denisson
Questão 4:

= sE A intersecção B complementar é vazio entao para todo x pertencente a A
implica que x não pertence a B complementar o que implica que x pertence a
B, logo A está contido em B.

= Se A está contido em B então para todo x pertencente a A então x pertence
a B, logo x não pertence a B complementar. isto quer dizer que A intersecção
com B complementar é vazio.


questão 5 - Basta tomar A intersecção com B não vazia e tomar C como um
conjunto que não tem elementos comuns nem com A nem com B.



2009/3/30 Robÿe9rio Alves prof_robe...@yahoo.com.br



PÁGINA 29 DO LIVRO DE ANÁLISE DO ELON

 04) Dados A, B está contido  em E, prove que A está contido em B se,
 somente se, A ∩ Complementar de B =  Ø

 Questão 5) Dê exemplos de conjuntos A, B, C tais que ( A U B ) ∩ C ≠ A U (
 B ∩C)


 QUESTÃO 8)  Prove que A = B se, e somente se, ( A ∩ Complementar de B ) U
 ( Complementar de A ∩ B ) = Ø





  --
 Veja quais são os assuntos do momento no Yahoo! + Buscados: Top 
 10http://br.rd.yahoo.com/mail/taglines/mail/*http://br.maisbuscados.yahoo.com/-
 Celebridadeshttp://br.rd.yahoo.com/mail/taglines/mail/*http://br.maisbuscados.yahoo.com/celebridades/-
 Músicahttp://br.rd.yahoo.com/mail/taglines/mail/*http://br.maisbuscados.yahoo.com/m%C3%BAsica/-
 Esporteshttp://br.rd.yahoo.com/mail/taglines/mail/*http://br.maisbuscados.yahoo.com/esportes/




-- 
Denisson


[obm-l] questões LIVRO DE ANÁLISE DO ELON

2009-03-30 Por tôpico Robÿffffe9rio Alves








PÁGINA 29 DO LIVRO DE ANÁLISE DO ELON 
 
04) Dados A, B está contido  em E, prove que A está contido em B se, somente 
se, A ∩ Complementar de B =  Ø
 
Questão 5) Dê exemplos de conjuntos A, B, C tais que ( A U B ) ∩ C ≠ A U ( B ∩C)
 
 
QUESTÃO 8)  Prove que A = B se, e somente se, ( A ∩ Complementar de B ) U ( 
Complementar de A ∩ B ) = Ø
 
 
 


  Veja quais são os assuntos do momento no Yahoo! +Buscados
http://br.maisbuscados.yahoo.com

[obm-l] questões LIVRO DE ANÁLISE DO ELON V1

2009-03-30 Por tôpico Robÿffffe9rio Alves








PÁGINA 29 DO LIVRO DE ANÁLISE DO ELON 
 
04) Dados A, B está contido  em E, prove que A está contido em B se, somente 
se, A ∩ Complementar de B =  Ø
 
Questão 5) Dê exemplos de conjuntos A, B, C tais que ( A U B ) ∩ C ≠ A U ( B ∩C)
 
 
QUESTÃO 8)  Prove que A = B se, e somente se, ( A ∩ Complementar de B ) U ( 
Complementar de A ∩ B ) = Ø
 
 
 


  Veja quais são os assuntos do momento no Yahoo! +Buscados
http://br.maisbuscados.yahoo.com

[obm-l] Re: [obm-l] questões LIVRO DE ANÁLISE DO ELON

2009-03-30 Por tôpico Marcelo Salhab Brogliato
Olá Robério,

vou tentar fazer a 4..
ida) Se A C B, entao para todo a E A, a E B, logo a \E compl(B), assim, nao
existe a E A tq a E compl(B), logo: A inter compl(B) = {}.
volta) Se A inter compl(B) = {}, entao nao existe a E A tq a E compl(B),
assim para todo a E A temos que a \E compl(B), logo a E compl(compl(B)) = B,
logo, A C B.

espero ter ajudado.. e espero que esse monte de letras seja legível! hehehe

abraços,
Salhab



2009/3/30 Robÿe9rio Alves prof_robe...@yahoo.com.br



PÁGINA 29 DO LIVRO DE ANÁLISE DO ELON

 04) Dados A, B está contido  em E, prove que A está contido em B se,
 somente se, A ∩ Complementar de B =  Ø

 Questão 5) Dê exemplos de conjuntos A, B, C tais que ( A U B ) ∩ C ≠ A U (
 B ∩C)


 QUESTÃO 8)  Prove que A = B se, e somente se, ( A ∩ Complementar de B ) U
 ( Complementar de A ∩ B ) = Ø





 --
 Veja quais são os assuntos do momento no Yahoo! + Buscados: Top 
 10http://br.rd.yahoo.com/mail/taglines/mail/*http://br.maisbuscados.yahoo.com/-
 Celebridadeshttp://br.rd.yahoo.com/mail/taglines/mail/*http://br.maisbuscados.yahoo.com/celebridades/-
 Músicahttp://br.rd.yahoo.com/mail/taglines/mail/*http://br.maisbuscados.yahoo.com/m%C3%BAsica/-
 Esporteshttp://br.rd.yahoo.com/mail/taglines/mail/*http://br.maisbuscados.yahoo.com/esportes/



[obm-l] Re: [obm-l] questões topologia da reta

2009-01-27 Por tôpico Rafael Assis
Para resolver o segundo, basta ver como é feito o conjunto de Cantor:
Na primeira iteração, retira-se o terço do meio do intervalo [0,1], ou seja,
um intervalo de comprimento um terço.
Na segunda iteração retiram-se dois intervalos de comprimento um terço de um
terço, isto é, dois nonos.
Note que os comprimentos dos intervalos omitidos( você pode fazer mais
iterações para observar isto) a cada iteração forma uma progressão
geométrica infinita cujo primeiro termo é um terço e a razão 2 terços.Logo,
a soma
da serie formada por tais termos deve ser 1.

Saudações,

Rafael


RE: [obm-l] questões topologia da reta

2009-01-27 Por tôpico LEANDRO L RECOVA

Eu esqueci de escrever que X = UNIAO_{1 a n} I_{xi} intersecao X. Desculpe. 

From: leandrorec...@msn.comto: ob...@mat.puc-rio.brsubject: RE: [obm-l] 
questões topologia da retaDate: Mon, 26 Jan 2009 13:36:41 -0800

Primeiro exercicio: Ja que X e compacto, voce consegue uma cobertura finita de 
intervalos I_{xi} com centro em x_{i} tal que X esta na uniao desses 
intervalos. Voce tambem pode escrever X = Intersecao de I_{xi} com X. Agora, 
como f e localmente limitada, entao ela e limitada em cada f(I_{xi} intersecao 
X). Deixo a conclusao pra voce. Regards, Leandro 

Date: Sun, 25 Jan 2009 21:16:57 -0200Subject: [obm-l] questões topologia da 
retaFrom: murilo.kr...@gmail.comto: ob...@mat.puc-rio.brprezados,estou 
apanhando nessas duas questões, alguém poderia me dar uma força? Seja X C R. 
Uma funcão f : X - R chama-se locamente limitada quando para cadax 
pertencente a X existe um intervalo aberto Ix, contendo x, talque f I Ix 
(interseção) X e limitada. Mostre quese X é compacto, toda função f : X - R 
localmente limitada e limitada. Prove que a soma da serie cujos termos são os 
comprimentos dos intervalos omitidos paraformar o conjunto de Cantor é igual a 
1.abraços,Murilo

RE: [obm-l] questões topologia da reta

2009-01-26 Por tôpico LEANDRO L RECOVA

Primeiro exercicio:
 
Ja que X e compacto, voce consegue uma cobertura finita de intervalos I_{xi} 
com centro em x_{i} tal que X esta na uniao desses intervalos. Voce tambem pode 
escrever X = Intersecao de I_{xi} com X. Agora, como f e localmente limitada, 
entao ela e limitada em cada f(I_{xi} intersecao X). Deixo a conclusao pra 
voce.
 
Regards,
 
Leandro
 



Date: Sun, 25 Jan 2009 21:16:57 -0200Subject: [obm-l] questões topologia da 
retaFrom: murilo.kr...@gmail.comto: ob...@mat.puc-rio.brprezados,estou 
apanhando nessas duas questões, alguém poderia me dar uma força? Seja X C R. 
Uma funcão f : X - R chama-se locamente limitada quando para cadax 
pertencente a X existe um intervalo aberto Ix, contendo x, talque f I Ix 
(interseção) X e limitada. Mostre quese X é compacto, toda função f : X - R 
localmente limitada e limitada. Prove que a soma da serie cujos termos são os 
comprimentos dos intervalos omitidos paraformar o conjunto de Cantor é igual a 
1.abraços,Murilo

[obm-l] questões topologia da reta

2009-01-25 Por tôpico Murilo Krell
prezados,
estou apanhando nessas duas questões, alguém poderia me dar uma força?


 Seja X C R. Uma funcão f : X - R chama-se locamente limitada quando para
cada
x pertencente a X existe um intervalo aberto Ix, contendo x, talque f I Ix
(interseção) X e limitada. Mostre que
se X é compacto, toda função f : X - R localmente limitada e limitada.

 Prove que a soma da serie cujos termos são os comprimentos dos intervalos
omitidos para
formar o conjunto de Cantor é igual a 1.

abraços,
Murilo


[obm-l] Questões de Geometria e Teoria dos Números (nova pos tagem)

2008-09-02 Por tôpico luiz silva






Olá Pessoal,
 
Estou postando novamente estes problemas :

Geometria
 
1) Considere um ângulo  90 BÂC 180, com ABAC. Com centro em B e raio AB, 
trace um arco de circunferência (AD) de 120. Com centro em C, e raio AC, 
trace outro arco de circunferência (AE) de 120, de modo que DÂEBÂC(a figura é 
côncava). Marque, sobre o arco de circunferência AD um ponto F, tal que DF= l12 
( L minúsculo..rs ). Marque, sobre o arco de circunferência AE o ponto G, tal 
que GE= l12(referente ao arco em que se está marcando o ponto). Trace o 
segmento de reta que une GF, marcando seu ponto M, médio. Marque um ponto H, 
externo ao polígono ABDFGEC, tal que o ângulo D^HE = 120 e HD=HE. O segmento HB 
intercepta o arco AD em P, e o segmento MB intercepta o arco AD em Q. Calcule o 
ângulo PÂQ.  
 
2) Considere o quadrilátero PABC, onde PA=PB=PC, se o ângulo B^PC= 22, calcule 
CÂB.
 
3) Dado o hexágono regular ABCDEF, , um ponto P exterior a este hexagono, com 
PÂF=150 e PA=PF, um ponto Q exterior a este hexágono, tal que F^QE=150 e QF=QE. 
Os segmentos QB e PE conrtam-se em L; QB e PD em M; QC e PD em N e QC e PE em 
O. Calcule L^NO
 
4) Dado um triângulo escaleno qualquer ABC. Trace a altura AH (H pé da altura). 
A partir de H, trace uma perpendicular a AB que intercepte este lado num ponto 
D e uma perpendicular ao lado AC, que intercepte este lado em um ponto E. A 
partir de D, trace uma perpendicular a AC, que intercepte este lado em F e de E 
trace uma perpendicular a AB, que intercepte o segmento DF em G. Sabendo 
que BC=a, AB=c e AC=b e AH=h, calcule a área do quedrilátero DHEG em funçaõ dos 
lados do triângulo e da altura h. 
 
Teoria dos Números
 
1) Prove que a equação diofantina x2 + y2 = zn possui infinitas soluções 
inteiras não triviais para qualquer n natural.
 
2) Prove que a equãção diofantina  xn + yn = zn+1 possui infinitas soluções 
inteiras não triviais para qualquer n, natural.

--- Em qui, 14/8/08, luiz silva [EMAIL PROTECTED] escreveu:

De: luiz silva [EMAIL PROTECTED]
Assunto: Re: [obm-l] Questões de Geometria e Teoria dos Números
Para: obm-l@mat.puc-rio.br
Data: Quinta-feira, 14 de Agosto de 2008, 11:22







Ola Rafael,
 
Vc está correto. O enunciado deveria ser : 
 
Prove que a equação diofantina x2 + y2 = zn possui soluções inteiras 
(x,y,z) para qualquer n natural. 
 
Abs
Felipe

--- Em qui, 14/8/08, Rafael Ando [EMAIL PROTECTED] escreveu:

De: Rafael Ando [EMAIL PROTECTED]
Assunto: Re: [obm-l] Questões de Geometria e Teoria dos Números
Para: obm-l@mat.puc-rio.br
Data: Quinta-feira, 14 de Agosto de 2008, 10:03



Ah, uma coisa... note que Prove que a equação diofantina x2 + y2 = zn possui 
infinitas soluções inteiras NAO eh a mesma coisa que qqer potência de n pode 
ser representada com a soma de 2 quadrados - mesmo que vc tivesse dito 
qualquer potencia de z ou qualquer n-esima potencia)... 3^3 = 27, por 
exemplo, nao pode ser escrito como soma de quadrados... Acredito mesmo que a 
maioria dos pares (z,n) nao tenha solucao!
Acho ainda que o enunciado nao esta correto. nao seria algo do tipo: Prove 
que, para todo n, x2 + y2 = zn possui infinitas soluções inteiras? Pra mim nao 
eh a mesma coisa pois se n for uma das variaveis eu posso escolher x e y 
qualquer, n=1 e z = x^2 + y^2...


2008/8/14 luiz silva [EMAIL PROTECTED]









Olá Martin/Pessoal,
 
Qdo entrei para esta lista, enviei um email com algmas questões que tinha 
criado, para análise de vcs. Porém, acho estranho não ter , ainda, visualisado 
esta msg na minha caixa de entrada, o que me faz pensar que deve ter havido 
algum problema com o envio desta msg.Dessa foram, estou enviando novamente.
 
Caso esta msg já tenha ido para a lista, favor desconsiderar este email.
 
Se possível, gostaria de um feedback de vcs, com relação a dificuldade destes 
problemas.

 

 Geometria
 
1) Considere um ângulo  90 BÂC 180, com ABAC. Com centro em B e raio AB, 
trace um arco de circunferência (AD) de 120. Com centro em C, e raio AC, 
trace outro arco de circunferência (AE) de 120, de modo que DÂEBÂC(a figura é 
côncava). Marque, sobre o arco de circunferência AD um ponto F, tal que DF= l12 
( L minúsculo..rs ). Marque, sobre o arco de circunferência AE o ponto G, tal 
que GE= l12(referente ao arco em que se está marcando o ponto). Trace o 
segmento de reta que une GF, marcando seu ponto M, médio. Marque um ponto H, 
externo ao polígono ABDFGEC, tal que o ângulo D^HE = 120 e HD=HE. O segmento HB 
intercepta o arco AD em P, e o segmento MB intercepta o arco AD em Q. Calcule o 
ângulo PÂQ.  
 
2) Considere o quadrilátero PABC, onde PA=PB=PC, se o ângulo B^PC= 22, calcule 
CÂB.
 
3) Dado o hexágono regular ABCDEF, , um ponto P exterior a este hexagono, com 
PÂF=150 e PA=PF, um ponto Q exterior a este hexágono, tal que F^QE=150 e QF=QE. 
Os segmentos QB e PE conrtam-se em L; QB e PD em M; QC e PD em N e QC e PE em 
O. Calcule L^NO
 
4) Dado um triângulo escaleno qualquer ABC. Trace a altura AH (H pé da altura). 
A partir de H, trace uma

[obm-l] Re: [obm-l] Questões de Geometria e Teoria dos N úmeros

2008-08-16 Por tôpico luiz silva
Rafael,
 
A mesma correção de enunciado é valida para a 2a. questão sobre teoria dos 
números.
 
Abs
Felipe

--- Em qui, 14/8/08, Rafael Ando [EMAIL PROTECTED] escreveu:

De: Rafael Ando [EMAIL PROTECTED]
Assunto: Re: [obm-l] Questões de Geometria e Teoria dos Números
Para: obm-l@mat.puc-rio.br
Data: Quinta-feira, 14 de Agosto de 2008, 10:03



Ah, uma coisa... note que Prove que a equação diofantina x2 + y2 = zn possui 
infinitas soluções inteiras NAO eh a mesma coisa que qqer potência de n pode 
ser representada com a soma de 2 quadrados - mesmo que vc tivesse dito 
qualquer potencia de z ou qualquer n-esima potencia)... 3^3 = 27, por 
exemplo, nao pode ser escrito como soma de quadrados... Acredito mesmo que a 
maioria dos pares (z,n) nao tenha solucao!
Acho ainda que o enunciado nao esta correto. nao seria algo do tipo: Prove 
que, para todo n, x2 + y2 = zn possui infinitas soluções inteiras? Pra mim nao 
eh a mesma coisa pois se n for uma das variaveis eu posso escolher x e y 
qualquer, n=1 e z = x^2 + y^2...


2008/8/14 luiz silva [EMAIL PROTECTED]









Olá Martin/Pessoal,
 
Qdo entrei para esta lista, enviei um email com algmas questões que tinha 
criado, para análise de vcs. Porém, acho estranho não ter , ainda, visualisado 
esta msg na minha caixa de entrada, o que me faz pensar que deve ter havido 
algum problema com o envio desta msg.Dessa foram, estou enviando novamente.
 
Caso esta msg já tenha ido para a lista, favor desconsiderar este email.
 
Se possível, gostaria de um feedback de vcs, com relação a dificuldade destes 
problemas.

 

 Geometria
 
1) Considere um ângulo  90 BÂC 180, com ABAC. Com centro em B e raio AB, 
trace um arco de circunferência (AD) de 120. Com centro em C, e raio AC, 
trace outro arco de circunferência (AE) de 120, de modo que DÂEBÂC(a figura é 
côncava). Marque, sobre o arco de circunferência AD um ponto F, tal que DF= l12 
( L minúsculo..rs ). Marque, sobre o arco de circunferência AE o ponto G, tal 
que GE= l12(referente ao arco em que se está marcando o ponto). Trace o 
segmento de reta que une GF, marcando seu ponto M, médio. Marque um ponto H, 
externo ao polígono ABDFGEC, tal que o ângulo D^HE = 120 e HD=HE. O segmento HB 
intercepta o arco AD em P, e o segmento MB intercepta o arco AD em Q. Calcule o 
ângulo PÂQ.  
 
2) Considere o quadrilátero PABC, onde PA=PB=PC, se o ângulo B^PC= 22, calcule 
CÂB.
 
3) Dado o hexágono regular ABCDEF, , um ponto P exterior a este hexagono, com 
PÂF=150 e PA=PF, um ponto Q exterior a este hexágono, tal que F^QE=150 e QF=QE. 
Os segmentos QB e PE conrtam-se em L; QB e PD em M; QC e PD em N e QC e PE em 
O. Calcule L^NO
 
4) Dado um triângulo escaleno qualquer ABC. Trace a altura AH (H pé da altura). 
A partir de H, trace uma perpendicular a AB que intercepte este lado num ponto 
D e uma perpendicular ao lado AC, que intercepte este lado em um ponto E. A 
partir de D, trace uma perpendicular a AC, que intercepte este lado em F e de E 
trace uma perpendicular a AB, que intercepte o segmento DF em G. Sabendo 
que BC=a, AB=c e AC=b e AH=h, calcule a área do quedrilátero DHEG em funçaõ dos 
lados do triângulo e da altura h. 
 
Teoria dos Números
 
1) Prove que a equação diofantina x2 + y2 = zn possui infinitas soluções 
inteiras (ou seja, que qqer potência de n pode ser representada com a soma de 2 
quadrados).
 
2) Prove que a equãção diofantina  xn + yn = zn+1 possui infinitas soluções 
inteiras 



Novos endereços, o Yahoo! que você conhece. Crie um email novo com a sua cara 
@ymail.com ou @rocketmail.com.


-- 
Rafael



  Novos endereços, o Yahoo! que você conhece. Crie um email novo com a sua 
cara @ymail.com ou @rocketmail.com.
http://br.new.mail.yahoo.com/addresses

[obm-l] Questões de Geometria e Teoria dos Números

2008-08-14 Por tôpico luiz silva



Olá Martin/Pessoal,
 
Qdo entrei para esta lista, enviei um email com algmas questões que tinha 
criado, para análise de vcs. Porém, acho estranho não ter , ainda, visualisado 
esta msg na minha caixa de entrada, o que me faz pensar que deve ter havido 
algum problema com o envio desta msg.Dessa foram, estou enviando novamente.
 
Caso esta msg já tenha ido para a lista, favor desconsiderar este email.
 
Se possível, gostaria de um feedback de vcs, com relação a dificuldade destes 
problemas.

 

 Geometria
 
1) Considere um ângulo  90 BÂC 180, com ABAC. Com centro em B e raio AB, 
trace um arco de circunferência (AD) de 120. Com centro em C, e raio AC, 
trace outro arco de circunferência (AE) de 120, de modo que DÂEBÂC(a figura é 
côncava). Marque, sobre o arco de circunferência AD um ponto F, tal que DF= l12 
( L minúsculo..rs ). Marque, sobre o arco de circunferência AE o ponto G, tal 
que GE= l12(referente ao arco em que se está marcando o ponto). Trace o 
segmento de reta que une GF, marcando seu ponto M, médio. Marque um ponto H, 
externo ao polígono ABDFGEC, tal que o ângulo D^HE = 120 e HD=HE. O segmento HB 
intercepta o arco AD em P, e o segmento MB intercepta o arco AD em Q. Calcule o 
ângulo PÂQ.  
 
2) Considere o quadrilátero PABC, onde PA=PB=PC, se o ângulo B^PC= 22, calcule 
CÂB.
 
3) Dado o hexágono regular ABCDEF, , um ponto P exterior a este hexagono, com 
PÂF=150 e PA=PF, um ponto Q exterior a este hexágono, tal que F^QE=150 e QF=QE. 
Os segmentos QB e PE conrtam-se em L; QB e PD em M; QC e PD em N e QC e PE em 
O. Calcule L^NO
 
4) Dado um triângulo escaleno qualquer ABC. Trace a altura AH (H pé da altura). 
A partir de H, trace uma perpendicular a AB que intercepte este lado num ponto 
D e uma perpendicular ao lado AC, que intercepte este lado em um ponto E. A 
partir de D, trace uma perpendicular a AC, que intercepte este lado em F e de E 
trace uma perpendicular a AB, que intercepte o segmento DF em G. Sabendo 
que BC=a, AB=c e AC=b e AH=h, calcule a área do quedrilátero DHEG em funçaõ dos 
lados do triângulo e da altura h. 
 
Teoria dos Números
 
1) Prove que a equação diofantina x2 + y2 = zn possui infinitas soluções 
inteiras (ou seja, que qqer potência de n pode ser representada com a soma de 2 
quadrados).
 
2) Prove que a equãção diofantina  xn + yn = zn+1 possui infinitas soluções 
inteiras 


  Novos endereços, o Yahoo! que você conhece. Crie um email novo com a sua 
cara @ymail.com ou @rocketmail.com.
http://br.new.mail.yahoo.com/addresses

Re: [obm-l] Questões de Geometria e Teoria dos Números

2008-08-14 Por tôpico Rafael Ando
Ah, uma coisa... note que Prove que a equação diofantina *x2 + y2 =
z**n* possui
infinitas soluções inteiras NAO eh a mesma coisa que qqer potência de n
pode ser representada com a soma de 2 quadrados - mesmo que vc tivesse dito
qualquer potencia de z ou qualquer n-esima potencia)... 3^3 = 27, por
exemplo, nao pode ser escrito como soma de quadrados... Acredito mesmo que a
maioria dos pares (z,n) nao tenha solucao!
Acho ainda que o enunciado nao esta correto. nao seria algo do tipo:
Prove que, *para todo n*, x2 + y2 = zn possui infinitas soluções inteiras?
Pra mim nao eh a mesma coisa pois se n for uma das variaveis eu posso
escolher x e y qualquer, n=1 e z = x^2 + y^2...

2008/8/14 luiz silva [EMAIL PROTECTED]

  Olá Martin/Pessoal,

 Qdo entrei para esta lista, enviei um email com algmas questões que tinha
 criado, para análise de vcs. Porém, acho estranho não ter , ainda,
 visualisado esta msg na minha caixa de entrada, o que me faz pensar que deve
 ter havido algum problema com o envio desta msg.Dessa foram, estou enviando
 novamente.

 Caso esta msg já tenha ido para a lista, favor desconsiderar este email.

 Se possível, gostaria de um feedback de vcs, com relação a dificuldade
 destes problemas.

  * Geometria*

 1) Considere um ângulo  90 BÂC 180, com ABAC. Com centro em B e raio
 AB, trace um arco de circunferência (AD) de 120. Com centro em C, e raio AC,
 trace outro arco de circunferência (AE) de 120, de modo que DÂEBÂC(a figura
 é côncava). Marque, sobre o arco de circunferência AD um ponto F, tal que
 DF= l12 ( L minúsculo..rs ). Marque, sobre o arco de circunferência AE o
 ponto G, tal que GE= l12(referente ao arco em que se está marcando o ponto).
 Trace o segmento de reta que une GF, marcando seu ponto M, médio. Marque um
 ponto H, externo ao polígono ABDFGEC, tal que o ângulo D^HE = 120 e HD=HE. O
 segmento HB intercepta o arco AD em P, e o segmento MB intercepta o arco
 AD em Q. Calcule o ângulo PÂQ.

 2) Considere o quadrilátero PABC, onde PA=PB=PC, se o ângulo B^PC= 22,
 calcule CÂB.

 3) Dado o hexágono regular ABCDEF, , um ponto P exterior a este hexagono,
 com PÂF=150 e PA=PF, um ponto Q exterior a este hexágono, tal que F^QE=150 e
 QF=QE. Os segmentos QB e PE conrtam-se em L; QB e PD em M; QC e PD em N e QC
 e PE em O. Calcule L^NO

 4) Dado um triângulo escaleno qualquer ABC. Trace a altura AH (H pé da
 altura). A partir de H, trace uma perpendicular a AB que intercepte este
 lado num ponto D e uma perpendicular ao lado AC, que intercepte este lado em
 um ponto E. A partir de D, trace uma perpendicular a AC, que intercepte este
 lado em F e de E trace uma perpendicular a AB, que intercepte o segmento DF
 em G. Sabendo que BC=a, AB=c e AC=b e AH=h, calcule a área do quedrilátero
 DHEG em funçaõ dos lados do triângulo e da altura h.

 *Teoria dos Números*

 1) Prove que a equação diofantina *x2 + y2 = z**n* possui infinitas
 soluções inteiras (ou seja, que qqer potência de n pode ser representada com
 a soma de 2 quadrados).

 2) Prove que a equãção diofantina * **xn + yn = zn+1 *possui infinitas
 soluções inteiras


 --
 Novos endereços, o Yahoo! que você conhece. Crie um email 
 novohttp://br.rd.yahoo.com/mail/taglines/mail/*http://br.new.mail.yahoo.com/addressescom
  a sua cara @
 ymail.com ou @rocketmail.com.




-- 
Rafael


[obm-l] Re: [obm-l] Questões de Geometria e Teoria dos N úmeros

2008-08-14 Por tôpico luiz silva
Ola Rafael,
 
Vc está correto. O enunciado deveria ser : 
 
Prove que a equação diofantina x2 + y2 = zn possui soluções inteiras 
(x,y,z) para qualquer n natural. 
 
Abs
Felipe

--- Em qui, 14/8/08, Rafael Ando [EMAIL PROTECTED] escreveu:

De: Rafael Ando [EMAIL PROTECTED]
Assunto: Re: [obm-l] Questões de Geometria e Teoria dos Números
Para: obm-l@mat.puc-rio.br
Data: Quinta-feira, 14 de Agosto de 2008, 10:03



Ah, uma coisa... note que Prove que a equação diofantina x2 + y2 = zn possui 
infinitas soluções inteiras NAO eh a mesma coisa que qqer potência de n pode 
ser representada com a soma de 2 quadrados - mesmo que vc tivesse dito 
qualquer potencia de z ou qualquer n-esima potencia)... 3^3 = 27, por 
exemplo, nao pode ser escrito como soma de quadrados... Acredito mesmo que a 
maioria dos pares (z,n) nao tenha solucao!
Acho ainda que o enunciado nao esta correto. nao seria algo do tipo: Prove 
que, para todo n, x2 + y2 = zn possui infinitas soluções inteiras? Pra mim nao 
eh a mesma coisa pois se n for uma das variaveis eu posso escolher x e y 
qualquer, n=1 e z = x^2 + y^2...


2008/8/14 luiz silva [EMAIL PROTECTED]









Olá Martin/Pessoal,
 
Qdo entrei para esta lista, enviei um email com algmas questões que tinha 
criado, para análise de vcs. Porém, acho estranho não ter , ainda, visualisado 
esta msg na minha caixa de entrada, o que me faz pensar que deve ter havido 
algum problema com o envio desta msg.Dessa foram, estou enviando novamente.
 
Caso esta msg já tenha ido para a lista, favor desconsiderar este email.
 
Se possível, gostaria de um feedback de vcs, com relação a dificuldade destes 
problemas.

 

 Geometria
 
1) Considere um ângulo  90 BÂC 180, com ABAC. Com centro em B e raio AB, 
trace um arco de circunferência (AD) de 120. Com centro em C, e raio AC, 
trace outro arco de circunferência (AE) de 120, de modo que DÂEBÂC(a figura é 
côncava). Marque, sobre o arco de circunferência AD um ponto F, tal que DF= l12 
( L minúsculo..rs ). Marque, sobre o arco de circunferência AE o ponto G, tal 
que GE= l12(referente ao arco em que se está marcando o ponto). Trace o 
segmento de reta que une GF, marcando seu ponto M, médio. Marque um ponto H, 
externo ao polígono ABDFGEC, tal que o ângulo D^HE = 120 e HD=HE. O segmento HB 
intercepta o arco AD em P, e o segmento MB intercepta o arco AD em Q. Calcule o 
ângulo PÂQ.  
 
2) Considere o quadrilátero PABC, onde PA=PB=PC, se o ângulo B^PC= 22, calcule 
CÂB.
 
3) Dado o hexágono regular ABCDEF, , um ponto P exterior a este hexagono, com 
PÂF=150 e PA=PF, um ponto Q exterior a este hexágono, tal que F^QE=150 e QF=QE. 
Os segmentos QB e PE conrtam-se em L; QB e PD em M; QC e PD em N e QC e PE em 
O. Calcule L^NO
 
4) Dado um triângulo escaleno qualquer ABC. Trace a altura AH (H pé da altura). 
A partir de H, trace uma perpendicular a AB que intercepte este lado num ponto 
D e uma perpendicular ao lado AC, que intercepte este lado em um ponto E. A 
partir de D, trace uma perpendicular a AC, que intercepte este lado em F e de E 
trace uma perpendicular a AB, que intercepte o segmento DF em G. Sabendo 
que BC=a, AB=c e AC=b e AH=h, calcule a área do quedrilátero DHEG em funçaõ dos 
lados do triângulo e da altura h. 
 
Teoria dos Números
 
1) Prove que a equação diofantina x2 + y2 = zn possui infinitas soluções 
inteiras (ou seja, que qqer potência de n pode ser representada com a soma de 2 
quadrados).
 
2) Prove que a equãção diofantina  xn + yn = zn+1 possui infinitas soluções 
inteiras 



Novos endereços, o Yahoo! que você conhece. Crie um email novo com a sua cara 
@ymail.com ou @rocketmail.com.


-- 
Rafael



  Novos endereços, o Yahoo! que você conhece. Crie um email novo com a sua 
cara @ymail.com ou @rocketmail.com.
http://br.new.mail.yahoo.com/addresses

Re: [obm-l] Questões do livro Álgebra I

2008-01-30 Por tôpico gsrs

Obrigado a todos pela ajuda!



Mensagem Original:
Data: 22:20:24 29/01/2008
De: Marcelo Salhab Brogliato [EMAIL PROTECTED]
Assunto: Re: [obm-l] Questões do livro Álgebra I



Olá Gabriel,
não vou resolver.. apenas dar umas dicas..

1ª) Um número de três algarismos a, b e c (ac) é tal que, quando

invertemos a ordem de seus algarismos e subtraímos o novo número do
original, encontramos, na diferença, um número terminado em 4. Essa
diferença é igual a:

a) 954  b) 594  c) 454   d) 544  e) Impossível calcular


n = 100a + 10b + c
invertendo a ordem, temos: 100c + 10b + a
faça a subtração.. :)




2ª) Se x pertence a {0, 1, 2, ..., 25}, para quantos valores de x, x2 +
3x + 2 é múltiplo de 6?


vc quer saber para qtos valores de x, temos: x^2 + 3x + 2 == 0 (mod 6)
veja que x^2 + 3x + 2 = (x+1)(x+2), logo: (x+1)(x+2) == 0 (mod 6)
ou: (x+1)(x+2) = 6k = (2*3)*k = (2*k)*3 = 2*(3*k)

como os números (x+1) e (x+2) estao em sequencia, temos que ter uma
fatoracao de 6k em sequencia...
veja.. (2*3)*k esta em sequencia para k=1 (2*k)*3 esta em sequencia para
k=2...
assim, para x=1 e x=2, a condicao esta satisfeita...
veja que, para k=7, temos: 6*7 ... logo: x=5


3ª) O algarismo das unidades do número (5837) elevado a 649 é:


a) 1  b)3  c)5  d) 7  e) 9



para obtermos o algarismo das unidades, basta pegarmos a divisão do número
por 10
deste modo, temos que calcular: (5837)^(649) (mod 10)
mas, 5837 == 7 (mod 10)  logo: (5837)^(649) == 7^(649) (mod 10)
agora, calcule 7^2, 7^3, 7^4, ... (mod 10)... vc vai notar uma propriedade
interessante! :)

abraços,
Salhab





Aqui na Oi Internet você ganha ou ganha. Além de acesso grátis com
qualidade, ganha contas ilimitadas de email com 1 giga cada uma. Ganha
espaço ilimitado para hospedar sua página pessoal. Ganha flog, suporte
grátis e muito mais. Baixe grátis o Discador em
http://www.oi.com.br/discador e comece a ganhar.

Agora, se o seu negócio é voar na internet sem pagar uma fortuna,
assine Oi Internet banda larga e ganhe modem grátis. Clique em
http://www.oi.com.br/bandalarga e aproveite essa moleza!




=
Instruções para entrar na lista, sair da lista e usar a lista em
http://www.mat.puc-rio.br/~obmlistas/obm-l.html
=


Re: [obm-l] Questões do livro Álgebra I

2008-01-30 Por tôpico Bernardo Freitas Paulo da Costa
2008/1/30 Marcelo Salhab Brogliato [EMAIL PROTECTED]:
 Olá Gabriel,
 não vou resolver.. apenas dar umas dicas..

  2ª) Se x pertence a {0, 1, 2, ..., 25}, para quantos valores de x, x2 +
  3x + 2 é múltiplo de 6?
 vc quer saber para qtos valores de x, temos: x^2 + 3x + 2 == 0 (mod 6)
 veja que x^2 + 3x + 2 = (x+1)(x+2), logo: (x+1)(x+2) == 0 (mod 6)
  ou: (x+1)(x+2) = 6k = (2*3)*k = (2*k)*3 = 2*(3*k)

 como os números (x+1) e (x+2) estao em sequencia, temos que ter uma
 fatoracao de 6k em sequencia...
 veja.. (2*3)*k esta em sequencia para k=1 (2*k)*3 esta em sequencia para
 k=2...
  assim, para x=1 e x=2, a condicao esta satisfeita...
 veja que, para k=7, temos: 6*7 ... logo: x=5

Uma outra idéia, depois da fatoração, é ver que dentre x+1 e x+2 temos
sempre um número par. Então, pra ser divisível por 6, temos que ter na
verdade ou x+1 ou x+2 um múltiplo de 3 (repare que é impossível que os
dois sejam!) Ora, isso é muito difícil (ou você pode argumentar que eu
sou preguiçoso) vamos calcular o número de elementos de {0, 1 ..., 25}
que NÃO dão nem x+1 nem x+2 divisíveis por 3. Para isso, x tem que ser
divisível por 3 (se não for, é da forma 3n+1 ou 3n+2, que você pode
escrever 3m-2 ou 3m-1 para m=n+1, e daí...) e portanto, no conjunto
temos 0, 3, 6, 9, .., 21, 24 o que dá um total de 9. Portanto, do
conjunto original, que tinha 26 elementos, 17 originam x^2 + 3x + 2
divisíveis por 6.

Abraços,
-- 
Bernardo Freitas Paulo da Costa


 abraços,
 Salhab

=
Instruções para entrar na lista, sair da lista e usar a lista em
http://www.mat.puc-rio.br/~obmlistas/obm-l.html
=


[obm-l] Questões do livro Álgebra I

2008-01-29 Por tôpico gsrs
Olá,

Me chamo Gabriel e estou estudando por um livro antigo chamado Álgebra
I dos professores Augusto César Morgado, Eduardo Wagner e Miguel
Jorge. Ainda no primeiro capítulo intitulado Os Inteiros e aborda o
conjunto dos inteiros, ordem dos inteiros, princípio da boa
ordenação, divisibilidade, números primos, mmc, mdc, algoritmo de
euclides, função de euler, bases de numeração e congruências. Tive
dúvidas em resolver as seguintes questões:

1ª) Um número de três algarismos a, b e c (ac) é tal que, quando
invertemos a ordem de seus algarismos e subtraímos o novo número do
original, encontramos, na diferença, um número terminado em 4. Essa
diferença é igual a:

a) 954  b) 594  c) 454   d) 544  e) Impossível calcular


2ª) Se x pertence a {0, 1, 2, ..., 25}, para quantos valores de x, x2 +
3x + 2 é múltiplo de 6?


3ª) O algarismo das unidades do número (5837) elevado a 649 é:

a) 1  b)3  c)5  d) 7  e) 9

Conto c/ a ajuda de vcs.

   Obrigado

Aqui na Oi Internet você ganha ou ganha. Além de acesso grátis com
qualidade, ganha contas ilimitadas de email com 1 giga cada uma. Ganha
espaço ilimitado para hospedar sua página pessoal. Ganha flog, suporte
grátis e muito mais. Baixe grátis o Discador em
http://www.oi.com.br/discador e comece a ganhar.

Agora, se o seu negócio é voar na internet sem pagar uma fortuna,
assine Oi Internet banda larga e ganhe modem grátis. Clique em
http://www.oi.com.br/bandalarga e aproveite essa moleza!



=
Instruções para entrar na lista, sair da lista e usar a lista em
http://www.mat.puc-rio.br/~obmlistas/obm-l.html
=


Re: [obm-l] Questões do livro Álgebra I

2008-01-29 Por tôpico Alex pereira Bezerra
1)Seja N = abc o número procurado temos cba - abc = 100c+ 10b + a - 100a -
10b - c =99a - 99c = ..4, 99(a-c) = ..4,logo a -c = 6, temos 99 x 6 = 594

Em 29/01/08, [EMAIL PROTECTED] [EMAIL PROTECTED] escreveu:

 Olá,

 Me chamo Gabriel e estou estudando por um livro antigo chamado Álgebra
 I dos professores Augusto César Morgado, Eduardo Wagner e Miguel
 Jorge. Ainda no primeiro capítulo intitulado Os Inteiros e aborda o
 conjunto dos inteiros, ordem dos inteiros, princípio da boa
 ordenação, divisibilidade, números primos, mmc, mdc, algoritmo de
 euclides, função de euler, bases de numeração e congruências. Tive
 dúvidas em resolver as seguintes questões:

 1ª) Um número de três algarismos a, b e c (ac) é tal que, quando
 invertemos a ordem de seus algarismos e subtraímos o novo número do
 original, encontramos, na diferença, um número terminado em 4. Essa
 diferença é igual a:

 a) 954  b) 594  c) 454   d) 544  e) Impossível calcular


 2ª) Se x pertence a {0, 1, 2, ..., 25}, para quantos valores de x, x2 +
 3x + 2 é múltiplo de 6?


 3ª) O algarismo das unidades do número (5837) elevado a 649 é:

 a) 1  b)3  c)5  d) 7  e) 9

 Conto c/ a ajuda de vcs.

   Obrigado

 
 Aqui na Oi Internet você ganha ou ganha. Além de acesso grátis com
 qualidade, ganha contas ilimitadas de email com 1 giga cada uma. Ganha
 espaço ilimitado para hospedar sua página pessoal. Ganha flog, suporte
 grátis e muito mais. Baixe grátis o Discador em
 http://www.oi.com.br/discador e comece a ganhar.

 Agora, se o seu negócio é voar na internet sem pagar uma fortuna,
 assine Oi Internet banda larga e ganhe modem grátis. Clique em
 http://www.oi.com.br/bandalarga e aproveite essa moleza!



 =
 Instruções para entrar na lista, sair da lista e usar a lista em
 http://www.mat.puc-rio.br/~obmlistas/obm-l.html
 =



Re: [obm-l] Questões do livro Álgebra I

2008-01-29 Por tôpico Marcelo Salhab Brogliato
Olá Gabriel,
não vou resolver.. apenas dar umas dicas..

1ª) Um número de três algarismos a, b e c (ac) é tal que, quando
 invertemos a ordem de seus algarismos e subtraímos o novo número do
 original, encontramos, na diferença, um número terminado em 4. Essa
 diferença é igual a:

 a) 954  b) 594  c) 454   d) 544  e) Impossível calcular

n = 100a + 10b + c
invertendo a ordem, temos: 100c + 10b + a
faça a subtração.. :)



 2ª) Se x pertence a {0, 1, 2, ..., 25}, para quantos valores de x, x2 +
 3x + 2 é múltiplo de 6?

vc quer saber para qtos valores de x, temos: x^2 + 3x + 2 == 0 (mod 6)
veja que x^2 + 3x + 2 = (x+1)(x+2), logo: (x+1)(x+2) == 0 (mod 6)
ou: (x+1)(x+2) = 6k = (2*3)*k = (2*k)*3 = 2*(3*k)

como os números (x+1) e (x+2) estao em sequencia, temos que ter uma
fatoracao de 6k em sequencia...
veja.. (2*3)*k esta em sequencia para k=1 (2*k)*3 esta em sequencia para
k=2...
assim, para x=1 e x=2, a condicao esta satisfeita...
veja que, para k=7, temos: 6*7 ... logo: x=5


3ª) O algarismo das unidades do número (5837) elevado a 649 é:

 a) 1  b)3  c)5  d) 7  e) 9


para obtermos o algarismo das unidades, basta pegarmos a divisão do número
por 10
deste modo, temos que calcular: (5837)^(649) (mod 10)
mas, 5837 == 7 (mod 10)  logo: (5837)^(649) == 7^(649) (mod 10)
agora, calcule 7^2, 7^3, 7^4, ... (mod 10)... vc vai notar uma propriedade
interessante! :)

abraços,
Salhab


[obm-l] Res: [obm-l] Questões do livro Álgebra I

2008-01-29 Por tôpico Eduardo Estrada
3ª) 7^0 termina em 1, 7^1 termina em 7, 7^2 termina em 9, 7^3 termina em 3, 7^4 
termina em 1, 7^5 termina em 7, depois, recomeça o ciclo de terminações das 
potências de 7: 1,7,9,3,1,7,9,3,... Observe que tal ciclo possui 4 valores que 
se repetem sucessivamente. Agora, observe que 5837 termina em 7 e que 649 = 4 x 
162 + 1, ou seja, 649 possui resto 1 na divisão por 4, que é o tamanho dos 
ciclos. Logo, o 5837^649 termina em 7 

Resp.: Alt. D.

Abraço,
Eduardo

- Mensagem original 
De: Alex pereira Bezerra [EMAIL PROTECTED]
Para: obm-l@mat.puc-rio.br
Enviadas: Terça-feira, 29 de Janeiro de 2008 20:00:08
Assunto: Re: [obm-l] Questões do livro Álgebra I

1)Seja N = abc o número procurado temos cba - abc = 100c+ 10b + a - 100a - 10b 
- c =99a - 99c = ..4, 99(a-c) = ..4,logo a -c = 6, temos 99 x 6 = 594


Em 29/01/08, [EMAIL PROTECTED] [EMAIL PROTECTED] escreveu: 
Olá,

Me chamo Gabriel e estou estudando por um livro antigo chamado Álgebra
I dos professores Augusto César Morgado, Eduardo Wagner e Miguel
Jorge. Ainda no primeiro capítulo intitulado Os Inteiros e aborda o
conjunto dos inteiros, ordem dos inteiros, princípio da boa
ordenação, divisibilidade, números primos, mmc, mdc, algoritmo de
euclides, função de euler, bases de numeração e congruências. Tive
dúvidas em resolver as seguintes questões:

1ª) Um número de três algarismos a, b e c (ac) é tal que, quando
invertemos a ordem de seus algarismos e subtraímos o novo número do
original, encontramos, na diferença, um número terminado em 4. Essa
diferença é igual a:

a) 954  b) 594  c) 454   d) 544  e) Impossível calcular


2ª) Se x pertence a {0, 1, 2, ..., 25}, para quantos valores de x, x2 +
3x + 2 é múltiplo de 6?


3ª) O algarismo das unidades do número (5837) elevado a 649 é:

a) 1  b)3  c)5  d) 7  e) 9

Conto c/ a ajuda de vcs.

  Obrigado

Aqui na Oi Internet você ganha ou ganha. Além de acesso grátis com
qualidade, ganha contas ilimitadas de email com 1 giga cada uma. Ganha
espaço ilimitado para hospedar sua página pessoal. Ganha flog, suporte
grátis e muito mais. Baixe grátis o Discador em
http://www.oi.com.br/discador e comece a ganhar.

Agora, se o seu negócio é voar na internet sem pagar uma fortuna,
assine Oi Internet banda larga e ganhe modem grátis. Clique em
http://www.oi.com.br/bandalarga e aproveite essa moleza!



=
Instruções para entrar na lista, sair da lista e usar a lista em
http://www.mat.puc-rio.br/~obmlistas/obm-l.html
=


  Abra sua conta no Yahoo! Mail, o único sem limite de espaço para 
armazenamento!
http://br.mail.yahoo.com/

[obm-l] Re: [obm-l] Re: [obm-l] Questões da OBM

2007-10-30 Por tôpico barola
É, tem razão.
Deixei passar tal argumento..
Entendi agora.

Obrigada.
Abraçosss..
  - Original Message - 
  From: Fetofs Ashu 
  To: obm-l@mat.puc-rio.br 
  Sent: Monday, October 29, 2007 8:20 PM
  Subject: Re: [obm-l] Re: [obm-l] Questões da OBM


  Bárbara,

  Lembra do meu ponto 1? Se você tem um grupo (x, y, z, w), só há um termo que 
pode vir antes desses quatro termos, quaisquer sejam eles.

  Provar o ponto 1 é trivial, já que precisamos de um valor congruente a 
w-(z+y+x) mod 10 e não há dois termos dos possíveis membros da sequência (0 a 
9) que têm o mesmo valor mod 10. 

  Mesmo se você não entender de aritmética modular, o ponto 1 é muito 
intuitivo. Pegue alguns grupos (x, y, z, w) quaisquer e veja se você consegue 
achar dois termos que podem vir antes desses. Você vai logo se cansar, já que 
não tem jeito :) 

  (não continue até entender o que eu disse até agora)

  Logo a sequência (i, j, k, l) não pode vir depois de um h e depois de um 
l ao mesmo tempo (claro, se considerarmos que h é diferente de l). Concluímos 
que a situação proposta é impossível. 

  Note que para rejeitar um ciclo de período indefinido, precisamos do ponto 2. 
Como cada grupo (c, d, e, f) só tem um termo que pode antecedê-lo (chamaremos 
de b), o grupo (b, c, d, e) também só tem um termo que pode antecedê-lo (um 
termo a qualquer). Logo, cada termo só pode vir de uma sequência definida 
(por exemplo, os números 1, 1, 1, 3 só podem vir depois de 0, 8, 9, 2, 9, 0, 
etc.) 

  Fernando Oliveira




  On 10/29/07, [EMAIL PROTECTED]  [EMAIL PROTECTED] wrote:
Desculpe a ignorância, mas porque não podemos pensar que o ciclo seja 
com um período parcial?
Assim: a,b,c,d,e,f,g,h,i,j,k,l,i,j,k,l,i,j,k,l,i,j,k,l,.
Acho que sua solução está certa, só faltou provar que não dá certo para 
esse caso, concorda?

Mesmo assim, você, o Nicolau e todos os grandes alunos e mestres desta 
lista tem me ensinado muito!
Obrigada mesmo!

- Original Message - 
  From: Fetofs Ashu 
  To: obm-l@mat.puc-rio.br 
  Sent: Monday, October 29, 2007 11:28 AM
  Subject: Re: [obm-l] Questões da OBM


  Salhab e Bárbara,

  1) Vamos andar para trás. Se você tem um grupo (x, y, z, w), só há um 
termo que pode vir antes desses quatro termos, quaisquer sejam eles.
  2) Continuando o processo de 1) temos que todo grupo só pode ser obtido 
através de uma sequência definida.
  3) Um grupo deve se repetir, pois o número de grupos possíveis é finito.
  4) Como na primeira vez que esse grupo apareceu (1, 2, 3, 4) fazia parte 
da sequência, deve fazer na segunda vez também, já que a sequência é única.

  Fernando Oliveira 



[obm-l] Re: [obm-l] Questões da OBM

2007-10-29 Por tôpico barola
 Oi Shine!
 Achei realmente muito interessante a sua solução para o problema das 
pilhas!
No entanto, não entendi, assim como o Salhab, como 1,2,3,4 vai aparecer de 
novo!
Henrique, meu nome é Bárbara sim. Só que eu tive que colocar um apelido 
no e-mail,
   pois o meu original havia sido clonado ou sei lá o quê! hehe..

Bjos

  - Original Message - 
  From: Marcelo Salhab Brogliato 
  To: obm-l@mat.puc-rio.br 
  Sent: Sunday, October 28, 2007 10:25 PM
  Subject: Re: [obm-l] Questões da OBM


  Olá Shine,
  gostei mto da sua resposta...
  mas nao entendi como vc provou que 1,2,3,4 vai aparecer novamente...

  abracos,
  Salhab



  On 10/27/07, Carlos Yuzo Shine [EMAIL PROTECTED] wrote:
Oi,

O problema 3 tem uma solução bem bonita (não é minha,
eu vi não me lembro onde): imagine que há 100*101/2 =
5050 cordas, cada uma amarrando cada par de pedras.
Então, para Esmeralda separar uma pilha de a+b pedras 
em uma pilha de a pedras e outra de b pedras, ela deve
cortar ab cordas! Como no final devemos ter pedras
soltas, devemos cortar todas as cordas, de modo que a
soma pedida é igual à quantidade de cordas, que é 
5050.

No problema 2, item a, suponha por absurdo que
apareçam 2,0,0,4 nessa ordem. Então, voltando a
seqüência obtemos 2,2,0,0,4; 6,2,2,0,0,4... e só
obtemos números pares, absurdo, pois começamos com
1,2,3,4.

O item b é mais interessante: a seqüência é periódica
(assim como qualquer recursão linear homogênea). Para
ver isso, use casa dos pombos: considere todas as 10^4
quádruplas (a,b,c,d) de algarismos. Agora pense nas 
quádruplas (x,y,z,w) de quatro termos consecutivos da
seqüência dada. Após pelo menos 10^4 + 1 termos,
alguma quádrupla (x,y,z,w) vai se repetir, e a
seqüência vai ciclar a partir daí.

Infelizmente, (x,y,z,w) não é necessariamente 
(1,2,3,4). O que fazer então? Considere o começo da
seqüência mais uma quantidade grande de ciclos (o
suficiente para que seja o dobro do tamanho do começo
da seqüência sem ciclos). Se você voltar a seqüência 
(assim como no item a) de dois pontos diferentes, o
fim do primeiro ciclo e o fim do pedaço considerado da
seqüência, vai obter os mesmos dígitos. Entre eles,
vai aparecer 1,2,3,4 no começo se voltar do primeiro 
ponto e a mesma coisa, 1,2,3,4, se voltar do segundo
ponto. Assim, 1,2,3,4 aparece de novo na seqüência.

[]'s
Shine

--- Marcelo Salhab Brogliato [EMAIL PROTECTED] 
wrote:

 Olá Barola,

 ainda estou tentando resolver.. mas não consegui...
 achei a questão MUITO interessante...
 e espero que o item B seja falso.. é um indicio de
 que a sequencia nao eh 
 periodica..
 resta sabermos se ela nao fica periodica apos um
 tempo... por exemplo:
 aparecendo um segundo 9, 4, 8, 7.. entende?
 entao, poderiamos utiliza-la, por exemplo, para a
 geracao de numeros
 aleatorios...
 uma outra questao interessante é: qual a
 distribuicao de probabilidades
 dessa sequencia?
 como a sequencia esta limitada entre 0 e 9, se
 contarmos qtos 0 
 aparecerem... dps qtos 1 aparecem.. e
 assim por diante... e fizermos n-inf, essas
 quantidades seriam iguais?!

 estou tentando.. se eu conseguir mando alguma
 coisa..
 mas estou realmente sem ideias... 

 junto contigo, fico no aguardo da solucao de alguem
 da lista!

 abraços,
 Salhab




 On 10/24/07, [EMAIL PROTECTED] 
 [EMAIL PROTECTED] wrote:
 
   Oi gente! Alguém pode resolver estas? São da 3ª
 fase da OBM, mas pelo 
  visto o site não disponibiliza o gabarito.
 
 
 
  *PROBLEMA 2*
 
  A seqüência de algarismos
 
  1, 2, 3, 4, 0, 9, 6, 9, 4, 8, 7, … 
 
 
 
  é construída da seguinte maneira: cada elemento, a
 partir do quinto, é
  igual ao último algarismo da soma dos quatro
 anteriores.
  
  a) Os algarismos 2, 0, 0, 4, juntos e nesta ordem,
 aparecem na seqüência?
 
  b) Os algarismos iniciais 1, 2, 3, 4, juntos e
 nesta ordem, aparecem
  novamente na seqüência? 
 
 
 
 
 
  *PROBLEMA 3*
 
  Esmeralda tem uma pilha com 100 pedras. Ela divide
 essa pilha em duas novas pilhas e em seguida 
 multiplica as
 
  quantidades de pedras nessas duas novas pilhas e
 escreve o produto em um quadro. Ela então escolhe
 uma pilha
 
  com mais de uma pedra e repete esse procedimento: 
 a pilha é dividida em duas, as quantidades de pedras
 nessas
 
  duas pilhas são multiplicadas e o produto escrito
 no quadro. Esta operação é realizada até se obter
 apenas pilhas 
 
  com 1 pedra cada. Quais são os possíveis valores
 da soma de todos os produtos escritos no quadro

Re: [obm-l] Questões da OBM

2007-10-29 Por tôpico Nicolau C. Saldanha
On 10/24/07, [EMAIL PROTECTED] [EMAIL PROTECTED] wrote:
 PROBLEMA 2

 A seqüência de algarismos

 1, 2, 3, 4, 0, 9, 6, 9, 4, 8, 7, …



 é construída da seguinte maneira: cada elemento, a partir do quinto, é igual
 ao último algarismo da soma dos quatro anteriores.

 a) Os algarismos 2, 0, 0, 4, juntos e nesta ordem, aparecem na seqüência?

 b) Os algarismos iniciais 1, 2, 3, 4, juntos e nesta ordem, aparecem
 novamente na seqüência?

O Shine já respondeu, vou mostrar como determinar quando aparecem os
algarismos 1,2,3,4.

Antes de mais nada podemos trabalhar independentemente módulo 2 e módulo 5.
Módulo 2 a seqüência é
1,0,1,0,0,1,0,1,0,0,1,0,1,0,0,...
ou seja, tem período 5.

Módulo 5 a seqüência começa assim:
1,2,3,4,0,4,2,0,...
e pode parecer intimidador procurar o período. Se considerarmos uma seq definida
pela mesma regra mas com a[0] = 1, a[1] = a[2] = a[3] = 0 teremos o seguinte:
[00] 1, 0, 0, 0, 1, 1, 2, 4, 3, 0
[10] 4, 1, 3, 3, 1, 3, 0, 2, 1, 1
[20] 4, 3, 4, 2, 3, 2, 1, 3, 4, 0
[30] 3, 0, 2, 0, 0, 2, 4, 1, 2, 4
[40] 1, 3, 0, 3, 2, 3, 3, 1, 4, 1
[50] 4, 0, 4, 4, 2, 0, 0, 1, 3, 4
[60] 3, 1, 1, 4, 4, 0, 4, 2, 0, 1
[70] 2, 0, 3, 1, 1, 0, 0, 2, 3, 0
[80] 0, 0, 3, 3, 1, 2, 4, 0, 2, 3
donde a[78+n] = 3*a[n] e portanto a[312+n] = 3^4*a[n] = a[n].

Assim o período é 5*312 = 1560.

N.

=
Instruções para entrar na lista, sair da lista e usar a lista em
http://www.mat.puc-rio.br/~obmlistas/obm-l.html
=


Re: [obm-l] Questões da OBM

2007-10-29 Por tôpico Fetofs Ashu
Salhab e Bárbara,

1) Vamos andar para trás. Se você tem um grupo (x, y, z, w), só há um termo
que pode vir antes desses quatro termos, quaisquer sejam eles.
2) Continuando o processo de 1) temos que todo grupo só pode ser obtido
através de uma sequência definida.
3) Um grupo deve se repetir, pois o número de grupos possíveis é finito.
4) Como na primeira vez que esse grupo apareceu (1, 2, 3, 4) fazia parte da
sequência, deve fazer na segunda vez também, já que a sequência é única.

Fernando Oliveira


Re: [obm-l] Questões da OBM

2007-10-29 Por tôpico Nicolau C. Saldanha
Relendo a minha própria mensagem achei que não tinha ficado claro
pq os períodos das duas seqs módulo 5 seriam iguais.

Observe a seq da outra mensagem:

 Se considerarmos uma seq definida
 pela mesma regra mas com a[0] = 1, a[1] = a[2] = a[3] = 0 teremos o seguinte:
 [00] 1, 0, 0, 0, 1, 1, 2, 4, 3, 0
 [10] 4, 1, 3, 3, 1, 3, 0, 2, 1, 1
 [20] 4, 3, 4, 2, 3, 2, 1, 3, 4, 0
 [30] 3, 0, 2, 0, 0, 2, 4, 1, 2, 4
 [40] 1, 3, 0, 3, 2, 3, 3, 1, 4, 1
 [50] 4, 0, 4, 4, 2, 0, 0, 1, 3, 4
 [60] 3, 1, 1, 4, 4, 0, 4, 2, 0, 1
 [70] 2, 0, 3, 1, 1, 0, 0, 2, 3, 0
 [80] 0, 0, 3, 3, 1, 2, 4, 0, 2, 3
 donde a[78+n] = 3*a[n] e portanto a[312+n] = 3^4*a[n] = a[n].

Note que a[38] = 2, a[39] = 4, a[40] = 1, a[41] = 3 donde
a[116] = 3*2 = 1, a[117] = 3*4 = 2, a[118] = 3*1 = 3, a[119] = 3*3 = 4
donde a seq do problema é uma mera defasagem da seq a[n].

N.

=
Instruções para entrar na lista, sair da lista e usar a lista em
http://www.mat.puc-rio.br/~obmlistas/obm-l.html
=


[obm-l] Re: [obm-l] Questões da OBM

2007-10-29 Por tôpico barola
Desculpe a ignorância, mas porque não podemos pensar que o ciclo seja com 
um período parcial?
Assim: a,b,c,d,e,f,g,h,i,j,k,l,i,j,k,l,i,j,k,l,i,j,k,l,.
Acho que sua solução está certa, só faltou provar que não dá certo para 
esse caso, concorda?

Mesmo assim, você, o Nicolau e todos os grandes alunos e mestres desta 
lista tem me ensinado muito!
Obrigada mesmo!

- Original Message - 
  From: Fetofs Ashu 
  To: obm-l@mat.puc-rio.br 
  Sent: Monday, October 29, 2007 11:28 AM
  Subject: Re: [obm-l] Questões da OBM


  Salhab e Bárbara,

  1) Vamos andar para trás. Se você tem um grupo (x, y, z, w), só há um termo 
que pode vir antes desses quatro termos, quaisquer sejam eles.
  2) Continuando o processo de 1) temos que todo grupo só pode ser obtido 
através de uma sequência definida.
  3) Um grupo deve se repetir, pois o número de grupos possíveis é finito.
  4) Como na primeira vez que esse grupo apareceu (1, 2, 3, 4) fazia parte da 
sequência, deve fazer na segunda vez também, já que a sequência é única.

  Fernando Oliveira 

Re: [obm-l] Questões da OBM

2007-10-29 Por tôpico Henrique Rennó
On 10/29/07, Nicolau C. Saldanha [EMAIL PROTECTED] wrote:
 On 10/24/07, [EMAIL PROTECTED] [EMAIL PROTECTED] wrote:
  PROBLEMA 2
 
  A seqüência de algarismos
 
  1, 2, 3, 4, 0, 9, 6, 9, 4, 8, 7, …
 
 
 
  é construída da seguinte maneira: cada elemento, a partir do quinto, é igual
  ao último algarismo da soma dos quatro anteriores.
 
  a) Os algarismos 2, 0, 0, 4, juntos e nesta ordem, aparecem na seqüência?
 
  b) Os algarismos iniciais 1, 2, 3, 4, juntos e nesta ordem, aparecem
  novamente na seqüência?

 O Shine já respondeu, vou mostrar como determinar quando aparecem os
 algarismos 1,2,3,4.

 Antes de mais nada podemos trabalhar independentemente módulo 2 e módulo 5.
 Módulo 2 a seqüência é
 1,0,1,0,0,1,0,1,0,0,1,0,1,0,0,...
 ou seja, tem período 5.

 Módulo 5 a seqüência começa assim:
 1,2,3,4,0,4,2,0,...
 e pode parecer intimidador procurar o período. Se considerarmos uma seq 
 definida
 pela mesma regra mas com a[0] = 1, a[1] = a[2] = a[3] = 0 teremos o seguinte:
 [00] 1, 0, 0, 0, 1, 1, 2, 4, 3, 0
 [10] 4, 1, 3, 3, 1, 3, 0, 2, 1, 1
 [20] 4, 3, 4, 2, 3, 2, 1, 3, 4, 0
 [30] 3, 0, 2, 0, 0, 2, 4, 1, 2, 4
 [40] 1, 3, 0, 3, 2, 3, 3, 1, 4, 1
 [50] 4, 0, 4, 4, 2, 0, 0, 1, 3, 4
 [60] 3, 1, 1, 4, 4, 0, 4, 2, 0, 1
 [70] 2, 0, 3, 1, 1, 0, 0, 2, 3, 0
 [80] 0, 0, 3, 3, 1, 2, 4, 0, 2, 3
 donde a[78+n] = 3*a[n] e portanto a[312+n] = 3^4*a[n] = a[n].

 Assim o período é 5*312 = 1560.

Não entendi porque o período é 5*312 = 1560.

-- 
Henrique

=
Instruções para entrar na lista, sair da lista e usar a lista em
http://www.mat.puc-rio.br/~obmlistas/obm-l.html
=


Re: [obm-l] Re: [obm-l] Questões da OBM

2007-10-29 Por tôpico Fetofs Ashu
Bárbara,

Lembra do meu ponto 1? Se você tem um grupo (x, y, z, w), só há um termo
que pode vir antes desses quatro termos, quaisquer sejam eles.

Provar o ponto 1 é trivial, já que precisamos de um valor congruente a
w-(z+y+x) mod 10 e não há dois termos dos possíveis membros da sequência (0
a 9) que têm o mesmo valor mod 10.

Mesmo se você não entender de aritmética modular, o ponto 1 é muito
intuitivo. Pegue alguns grupos (x, y, z, w) quaisquer e veja se você
consegue achar dois termos que podem vir antes desses. Você vai logo se
cansar, já que não tem jeito :)

(não continue até entender o que eu disse até agora)

Logo a sequência (i, j, k, l) não pode vir depois de um h e depois de um
l ao mesmo tempo (claro, se considerarmos que h é diferente de l).
Concluímos que a situação proposta é impossível.

Note que para rejeitar um ciclo de período indefinido, precisamos do ponto
2. Como cada grupo (c, d, e, f) só tem um termo que pode antecedê-lo
(chamaremos de b), o grupo (b, c, d, e) também só tem um termo que pode
antecedê-lo (um termo a qualquer). Logo, cada termo só pode vir de uma
sequência definida (por exemplo, os números 1, 1, 1, 3 só podem vir depois
de 0, 8, 9, 2, 9, 0, etc.)

Fernando Oliveira



On 10/29/07, [EMAIL PROTECTED] [EMAIL PROTECTED] wrote:

  Desculpe a ignorância, mas porque não podemos pensar que o ciclo seja
 com um período parcial?
 Assim: a,b,c,d,e,f,g,h,i,j,k,l,i,j,k,l,i,j,k,l,i,j,k,l,.
 Acho que sua solução está certa, só faltou provar que não dá certo
 para esse caso, concorda?

 Mesmo assim, você, o Nicolau e todos os grandes alunos e mestres desta
 lista tem me ensinado muito!
 Obrigada mesmo!

 - Original Message -

 *From:* Fetofs Ashu [EMAIL PROTECTED]
 *To:* obm-l@mat.puc-rio.br
 *Sent:* Monday, October 29, 2007 11:28 AM
 *Subject:* Re: [obm-l] Questões da OBM

 Salhab e Bárbara,

 1) Vamos andar para trás. Se você tem um grupo (x, y, z, w), só há um
 termo que pode vir antes desses quatro termos, quaisquer sejam eles.
 2) Continuando o processo de 1) temos que todo grupo só pode ser obtido
 através de uma sequência definida.
 3) Um grupo deve se repetir, pois o número de grupos possíveis é finito.
 4) Como na primeira vez que esse grupo apareceu (1, 2, 3, 4) fazia parte
 da sequência, deve fazer na segunda vez também, já que a sequência é única.

 Fernando Oliveira




Re: [obm-l] Questões da OBM

2007-10-28 Por tôpico Henrique Rennó
On 10/24/07, [EMAIL PROTECTED] [EMAIL PROTECTED] wrote:
 Oi gente! Alguém pode resolver estas? São da 3ª fase da OBM, mas pelo visto
 o site não disponibiliza o gabarito.

 PROBLEMA 2

 A seqüência de algarismos

 1, 2, 3, 4, 0, 9, 6, 9, 4, 8, 7, …

 é construída da seguinte maneira: cada elemento, a partir do quinto, é igual
 ao último algarismo da soma dos quatro anteriores.

 a) Os algarismos 2, 0, 0, 4, juntos e nesta ordem, aparecem na seqüência?

 b) Os algarismos iniciais 1, 2, 3, 4, juntos e nesta ordem, aparecem
 novamente na seqüência?

a) Para o 4 aparecer depois de 2,0,0 deve haver um número par já que
2,0,0 são pares e somados a outro número deve dar um número par
terminado em 4. O número seria 2. Logo teríamos 2,2,0,0,4. Essa lógica
prossegue já que para chegarmos em 0 teríamos que somar a três números
pares um número a fim de obter outro par. Assim, antes de 2,2,0,0,4
teríamos outro número par. Se continuarmos veremos que serão sempre
números pares antes e portanto não seria possível chegar em números
ímpares e alguns ímpares estão no ínício da seqüência. Logo 2,0,0,4
nunca apareceriam juntos e nessa ordem na seqüência.

Uma dúvida? Seu nome é Bárbaral? Com L no final? Que diferente!

-- 
Henrique

=
Instruções para entrar na lista, sair da lista e usar a lista em
http://www.mat.puc-rio.br/~obmlistas/obm-l.html
=


Re: [obm-l] Questões da OBM

2007-10-28 Por tôpico Marcelo Salhab Brogliato
Olá Shine,
gostei mto da sua resposta...
mas nao entendi como vc provou que 1,2,3,4 vai aparecer novamente...

abracos,
Salhab


On 10/27/07, Carlos Yuzo Shine [EMAIL PROTECTED] wrote:

 Oi,

 O problema 3 tem uma solução bem bonita (não é minha,
 eu vi não me lembro onde): imagine que há 100*101/2 =
 5050 cordas, cada uma amarrando cada par de pedras.
 Então, para Esmeralda separar uma pilha de a+b pedras
 em uma pilha de a pedras e outra de b pedras, ela deve
 cortar ab cordas! Como no final devemos ter pedras
 soltas, devemos cortar todas as cordas, de modo que a
 soma pedida é igual à quantidade de cordas, que é
 5050.

 No problema 2, item a, suponha por absurdo que
 apareçam 2,0,0,4 nessa ordem. Então, voltando a
 seqüência obtemos 2,2,0,0,4; 6,2,2,0,0,4... e só
 obtemos números pares, absurdo, pois começamos com
 1,2,3,4.

 O item b é mais interessante: a seqüência é periódica
 (assim como qualquer recursão linear homogênea). Para
 ver isso, use casa dos pombos: considere todas as 10^4
 quádruplas (a,b,c,d) de algarismos. Agora pense nas
 quádruplas (x,y,z,w) de quatro termos consecutivos da
 seqüência dada. Após pelo menos 10^4 + 1 termos,
 alguma quádrupla (x,y,z,w) vai se repetir, e a
 seqüência vai ciclar a partir daí.

 Infelizmente, (x,y,z,w) não é necessariamente
 (1,2,3,4). O que fazer então? Considere o começo da
 seqüência mais uma quantidade grande de ciclos (o
 suficiente para que seja o dobro do tamanho do começo
 da seqüência sem ciclos). Se você voltar a seqüência
 (assim como no item a) de dois pontos diferentes, o
 fim do primeiro ciclo e o fim do pedaço considerado da
 seqüência, vai obter os mesmos dígitos. Entre eles,
 vai aparecer 1,2,3,4 no começo se voltar do primeiro
 ponto e a mesma coisa, 1,2,3,4, se voltar do segundo
 ponto. Assim, 1,2,3,4 aparece de novo na seqüência.

 []'s
 Shine

 --- Marcelo Salhab Brogliato [EMAIL PROTECTED]
 wrote:

  Olá Barola,
 
  ainda estou tentando resolver.. mas não consegui...
  achei a questão MUITO interessante...
  e espero que o item B seja falso.. é um indicio de
  que a sequencia nao eh
  periodica..
  resta sabermos se ela nao fica periodica apos um
  tempo... por exemplo:
  aparecendo um segundo 9, 4, 8, 7.. entende?
  entao, poderiamos utiliza-la, por exemplo, para a
  geracao de numeros
  aleatorios...
  uma outra questao interessante é: qual a
  distribuicao de probabilidades
  dessa sequencia?
  como a sequencia esta limitada entre 0 e 9, se
  contarmos qtos 0
  aparecerem... dps qtos 1 aparecem.. e
  assim por diante... e fizermos n-inf, essas
  quantidades seriam iguais?!
 
  estou tentando.. se eu conseguir mando alguma
  coisa..
  mas estou realmente sem ideias...
 
  junto contigo, fico no aguardo da solucao de alguem
  da lista!
 
  abraços,
  Salhab
 
 
 
 
  On 10/24/07, [EMAIL PROTECTED] 
  [EMAIL PROTECTED] wrote:
  
Oi gente! Alguém pode resolver estas? São da 3ª
  fase da OBM, mas pelo
   visto o site não disponibiliza o gabarito.
  
  
  
   *PROBLEMA 2*
  
   A seqüência de algarismos
  
   1, 2, 3, 4, 0, 9, 6, 9, 4, 8, 7, …
  
  
  
   é construída da seguinte maneira: cada elemento, a
  partir do quinto, é
   igual ao último algarismo da soma dos quatro
  anteriores.
  
   a) Os algarismos 2, 0, 0, 4, juntos e nesta ordem,
  aparecem na seqüência?
  
   b) Os algarismos iniciais 1, 2, 3, 4, juntos e
  nesta ordem, aparecem
   novamente na seqüência?
  
  
  
  
  
   *PROBLEMA 3*
  
   Esmeralda tem uma pilha com 100 pedras. Ela divide
  essa pilha em duas novas pilhas e em seguida
  multiplica as
  
   quantidades de pedras nessas duas novas pilhas e
  escreve o produto em um quadro. Ela então escolhe
  uma pilha
  
   com mais de uma pedra e repete esse procedimento:
  a pilha é dividida em duas, as quantidades de pedras
  nessas
  
   duas pilhas são multiplicadas e o produto escrito
  no quadro. Esta operação é realizada até se obter
  apenas pilhas
  
   com 1 pedra cada. Quais são os possíveis valores
  da soma de todos os produtos escritos no quadro?
  
  
  
   Desde já, agradeço.
  
   Bárbaral Nedel.
  
  
 


 __
 Do You Yahoo!?
 Tired of spam?  Yahoo! Mail has the best spam protection around
 http://mail.yahoo.com
 =
 Instruções para entrar na lista, sair da lista e usar a lista em
 http://www.mat.puc-rio.br/~obmlistas/obm-l.html
 =



[obm-l] Re: [obm-l] Questões da OBM

2007-10-27 Por tôpico barola
Valeu pela dica!
  - Original Message - 
  From: Fetofs Ashu 
  To: obm-l@mat.puc-rio.br 
  Sent: Friday, October 26, 2007 10:49 PM
  Subject: Re: [obm-l] Questões da OBM


  Seria uma boa idéia procurar nas revistas Eureka, se os problemas são 
relativamente recentes, pois lá é onde o gabarito da 3ª fase é normalmente 
disponibilizado.


Re: [obm-l] Questões da OBM

2007-10-27 Por tôpico Marcelo Salhab Brogliato
Olá Barola,

ainda estou tentando resolver.. mas não consegui...
achei a questão MUITO interessante...
e espero que o item B seja falso.. é um indicio de que a sequencia nao eh
periodica..
resta sabermos se ela nao fica periodica apos um tempo... por exemplo:
aparecendo um segundo 9, 4, 8, 7.. entende?
entao, poderiamos utiliza-la, por exemplo, para a geracao de numeros
aleatorios...
uma outra questao interessante é: qual a distribuicao de probabilidades
dessa sequencia?
como a sequencia esta limitada entre 0 e 9, se contarmos qtos 0
aparecerem... dps qtos 1 aparecem.. e
assim por diante... e fizermos n-inf, essas quantidades seriam iguais?!

estou tentando.. se eu conseguir mando alguma coisa..
mas estou realmente sem ideias...

junto contigo, fico no aguardo da solucao de alguem da lista!

abraços,
Salhab




On 10/24/07, [EMAIL PROTECTED]  [EMAIL PROTECTED] wrote:

  Oi gente! Alguém pode resolver estas? São da 3ª fase da OBM, mas pelo
 visto o site não disponibiliza o gabarito.



 *PROBLEMA 2*

 A seqüência de algarismos

 1, 2, 3, 4, 0, 9, 6, 9, 4, 8, 7, …



 é construída da seguinte maneira: cada elemento, a partir do quinto, é
 igual ao último algarismo da soma dos quatro anteriores.

 a) Os algarismos 2, 0, 0, 4, juntos e nesta ordem, aparecem na seqüência?

 b) Os algarismos iniciais 1, 2, 3, 4, juntos e nesta ordem, aparecem
 novamente na seqüência?





 *PROBLEMA 3*

 Esmeralda tem uma pilha com 100 pedras. Ela divide essa pilha em duas novas 
 pilhas e em seguida multiplica as

 quantidades de pedras nessas duas novas pilhas e escreve o produto em um 
 quadro. Ela então escolhe uma pilha

 com mais de uma pedra e repete esse procedimento: a pilha é dividida em duas, 
 as quantidades de pedras nessas

 duas pilhas são multiplicadas e o produto escrito no quadro. Esta operação é 
 realizada até se obter apenas pilhas

 com 1 pedra cada. Quais são os possíveis valores da soma de todos os produtos 
 escritos no quadro?



 Desde já, agradeço.

 Bárbaral Nedel.




Re: [obm-l] Questões da OBM

2007-10-26 Por tôpico Fetofs Ashu
Seria uma boa idéia procurar nas revistas Eureka, se os problemas são
relativamente recentes, pois lá é onde o gabarito da 3ª fase é normalmente
disponibilizado.


[obm-l] Questões da OBM

2007-10-24 Por tôpico barola
Oi gente! Alguém pode resolver estas? São da 3ª fase da OBM, mas pelo visto o 
site não disponibiliza o gabarito.


PROBLEMA 2

A seqüência de algarismos

1, 2, 3, 4, 0, 9, 6, 9, 4, 8, 7, .

 

é construída da seguinte maneira: cada elemento, a partir do quinto, é igual ao 
último algarismo da soma dos quatro anteriores. 

a) Os algarismos 2, 0, 0, 4, juntos e nesta ordem, aparecem na seqüência?

b) Os algarismos iniciais 1, 2, 3, 4, juntos e nesta ordem, aparecem novamente 
na seqüência?

 

 

PROBLEMA 3

Esmeralda tem uma pilha com 100 pedras. Ela divide essa pilha em duas novas 
pilhas e em seguida multiplica as quantidades de pedras nessas duas novas 
pilhas e escreve o produto em um quadro. Ela então escolhe uma pilha com mais 
de uma pedra e repete esse procedimento: a pilha é dividida em duas, as 
quantidades de pedras nessas duas pilhas são multiplicadas e o produto escrito 
no quadro. Esta operação é realizada até se obter apenas pilhas com 1 pedra 
cada. Quais são os possíveis valores da soma de todos os produtos escritos no 
quadro?Desde já, agradeço.Bárbaral Nedel.

[obm-l] Questões

2007-09-30 Por tôpico Anderson



=
Instruções para entrar na lista, sair da lista e usar a lista em
http://www.mat.puc-rio.br/~nicolau/olimp/obm-l.html
=


[obm-l] Questões

2007-09-30 Por tôpico Anderson
Ajuda nos resultados...

1- G tem 2n pedaços de papel numerados de 1 até 2n. Ele remove n pedaços de 
papel que são numerados consecutivamente. A soma do número restante de pedaços 
de papel é 1615. Ache todos os valores possivéis de n.

2- Em um quadrilátero ABCD, AB=CD, âng. ABC=77, e Ãng. BCD=150º. Seja P o ponto 
de intersecção da perpendicular dos bissetores de BC e AD. Ache o âng. BPC.


=
Instruções para entrar na lista, sair da lista e usar a lista em
http://www.mat.puc-rio.br/~nicolau/olimp/obm-l.html
=


[obm-l] QUESTÕES INVULGARES!

2007-01-16 Por tôpico Jorge Luis Rodrigues e Silva Luis
Ok! Marcelo, Rogério e demais colegas! Eis outras questões inusitadas 
propostas em concursos...


Sabe-se que numa equipe de futebol, há um atacante que sempre mente, um 
zagueiro que sempre fala a verdade e meio-campista que às vezes fala a 
verdade e às vezes mente. Na saída do estádio, dirigindo-se a um torcedor 
que não sabia o resultado do jogo que terminara, um deles declarou Foi 
empate o segundo disse Não foi empate e o terceiro falou nós perdemos. 
O torcedor reconheceu sómente o meio-campista, mas pode deduzir o resultado 
do jogo com certeza. Afinal, qual a declaração do meio-campista e o 
resultado do jogo?   (AFTN-96)


Dado um triângulo inscrito noutro pelos pontos médios, quantos caminhos 
diferentes existem entre os pontos extremos do triângulo circunscrito não 
passando por um determinado ponto médio e sem passsar duas vezes por um 
mesmo ponto? (IBGE-06)


Nota: Surpresa, mesmo, foi a resposta dada como certa pela comissão 
organizadora...


Há um dito popular que afirma que cão que late não morde. Levando-se em 
conta sómente essa afirmação, pode-se concluir que se um animal latir e 
morder...   (DERT-06)



Abraços! .Chicão..., grato pelos Links!

_
O Windows Live Spaces é seu espaço na internet com fotos (500 por mês), blog 
e agora com rede social http://spaces.live.com/


=
Instruções para entrar na lista, sair da lista e usar a lista em
http://www.mat.puc-rio.br/~nicolau/olimp/obm-l.html
=


Re: [obm-l] QUESTÕES INVULGARES!

2007-01-16 Por tôpico Rogerio Ponce
Ola' Jorge e colegas da lista,

Se o meio-campista afirmasse perdemos , o torcedor ficaria sem saber a 
verdade, pois as outras duas afirmacoes poderiam ser atribuidas a qualquer dos 
outros dois jogadores.
Idem se o meio-campista afirmasse nao empatou.
Entao o meio-campista disse empatou.
Bem, se quem fala a verdade dissesse perdemos , a afirmacao nao empatou 
seria dita pelo mentiroso, apesar de tambem ser verdadeira. Portanto, quem fala 
a verdade disse nao empatou , e o mentiroso disse perdemos.
Logo, o time ganhou!

[]s
Rogerio Ponce


Jorge Luis Rodrigues e Silva Luis [EMAIL PROTECTED] escreveu: Ok! Marcelo, 
Rogério e demais colegas! Eis outras questões inusitadas 
propostas em concursos...

Sabe-se que numa equipe de futebol, há um atacante que sempre mente, um 
zagueiro que sempre fala a verdade e meio-campista que às vezes fala a 
verdade e às vezes mente. Na saída do estádio, dirigindo-se a um torcedor 
que não sabia o resultado do jogo que terminara, um deles declarou Foi 
empate o segundo disse Não foi empate e o terceiro falou nós perdemos. 
O torcedor reconheceu sómente o meio-campista, mas pode deduzir o resultado 
do jogo com certeza. Afinal, qual a declaração do meio-campista e o 
resultado do jogo?   (AFTN-96)
.
.
.
Abraços! .Chicão..., grato pelos Links!


 __
Fale com seus amigos  de graça com o novo Yahoo! Messenger 
http://br.messenger.yahoo.com/ 

Re: [obm-l] QUESTÕES INVULGARES!

2007-01-16 Por tôpico Rogerio Ponce
Ola' Jorge e colegas da lista,
se um animal latir e morder, entao nao se trata de um cao.
Mas poderia ser uma cadela, certo?
:-)

[]s
Rogerio Ponce

PS: o ditado mais correto seria Cao que late nao morde...enquanto late!



Jorge Luis Rodrigues e Silva Luis [EMAIL PROTECTED] escreveu: .
.
.
Há um dito popular que afirma que cão que late não morde. Levando-se em 
conta sómente essa afirmação, pode-se concluir que se um animal latir e 
morder...   (DERT-06)



 __
Fale com seus amigos  de graça com o novo Yahoo! Messenger 
http://br.messenger.yahoo.com/ 

Re: [obm-l] QUESTÕES INVULGARES!

2007-01-16 Por tôpico Rogerio Ponce
Ola' Jorge e colegas da lista,
so' existe um caminho que passe pelos 3 vertices (os tais pontos extremos) do 
triangulo circunscrito.

Entretanto, o enunciado menciona entre os pontos (e nao pelos pontos), de 
forma que , considerando apenas um par de vertices isoladamente, entao 
encontramos 2 caminhos para cada par de vertices, ou seja, um total de 6 
caminhos para os 3 pares possiveis de vertices.

[]s
Rogerio Ponce


Jorge Luis Rodrigues e Silva Luis [EMAIL PROTECTED] escreveu:
Dado um triângulo inscrito noutro pelos pontos médios, quantos caminhos 
diferentes existem entre os pontos extremos do triângulo circunscrito não 
passando por um determinado ponto médio e sem passsar duas vezes por um 
mesmo ponto? (IBGE-06)

Nota: Surpresa, mesmo, foi a resposta dada como certa pela comissão 
organizadora...


 __
Fale com seus amigos  de graça com o novo Yahoo! Messenger 
http://br.messenger.yahoo.com/ 

Re: [obm-l] RES: [obm-l] Questões de Desigua ldades (MA,MH, MG e MQ)

2007-01-07 Por tôpico Chicao Valadares
Valeu Ralph, a resolução da primeira questão é muito
interessante.  
Quanto a segunda questão, interessante notar como na
matemática há questões que são facilmente resolvidas
por algumas pessoas, no entanto, lentamente por
outras, como foi o meu caso. Eu fiz outras
aparentemente mais complicadas mas empaquei nessa. 
Abraços. 



O Binômio de Newton é tão belo como a Vênus de Milo.
O que há é pouca gente para dar por isso... 
Fernando Pessoa - Poesias de Alvaro Campos

_
As informações existentes nessa mensagem e no(s) arquivo(s) anexado(s) 
são
para uso restrito, sendo seu sigilo protegido por lei. Caso não seja
destinatário, saiba que leitura, divulgação ou cópia são proibidas. 
Favor
apagar as informações e notificar o remetente. O uso impróprio será 
tratado
conforme as normas da empresa e a legislação em vigor. Agradecemos sua
colaboração.


The information mentioned in this message and in the archives attached 
are
of restricted use, and its privacy is protected by law. If you are not 
the
addressee, be aware that reading, disclosure or copy are forbidden. 
Please
delete this information and notify the sender. Inappropriate use will 
be
tracted according to company's rules and valid laws. Thank you for your
cooperation.

__
Fale com seus amigos  de graça com o novo Yahoo! Messenger 
http://br.messenger.yahoo.com/ 
=
Instruções para entrar na lista, sair da lista e usar a lista em
http://www.mat.puc-rio.br/~nicolau/olimp/obm-l.html
=


Re: [obm-l] QUESTÕES DE CONCURSOS!

2007-01-05 Por tôpico Rogerio Ponce
Ola' Jorge e pessoal da lista,

Prob. (A em Paris) = 3/7
Prob. (B em Paris) = 2/7
Prob (A em Paris)  (B em Paris) = 1/7

Logo,
Prob (A em Paris)  ~(B em Paris) = 3/7 - 1/7 = 2/7

Como A esta' em Paris,
Prob (B em Paris) = [1/7] / [ 1/7 + 2/7 ]  = 1/3

[]'s
Rogerio Ponce


Jorge Luis Rodrigues e Silva Luis [EMAIL PROTECTED] escreveu: C sabe que A e 
B estão viajando pela Europa. Com as informações que dispõe, 
ele estima corretamente que a probabilidade de A estar hoje em Paris é 3/7, 
que a probabilidade de B estar hoje em Paris é 2/7, e que a probabilidade de 
ambos, estarem hoje em Paris é 1/7. C, então, recebe um telefonema de A 
informando que ela está hoje em Paris. Com a informação recebida pelo 
telefonema de A, C agora estima corretamente que a probabilidade de B também 
estar hoje em Paris é igual a:   
(MPU - 04)



 __
Fale com seus amigos  de graça com o novo Yahoo! Messenger 
http://br.messenger.yahoo.com/ 

[obm-l] Questões de Desigualdades (MA,MH, MG e MQ)

2007-01-05 Por tôpico Chicao Valadares
1 - Prove que se as desigualdades das médias(MA,MH, MG
e MQ) é válida p/ n naturais diferentes de zero
então ela é válida p/ n-1 naturais diferentes de
zero.

2 - Prove que para todo n maior ou igual a 1 a
desigualdade é sempre é verdadeira: 
(1 + 1/n)^(1/n)  (1 + 1/(n+1))^(1/(n+1)) 
, ou seja , a sequencia a_n = (1 + 1/n)^(1/n) é
estritamente crescente para n natural diferente de
zero(só p/ lembrar que quando n- infinito = a_n -
e mas isso não deve ser usado no problema).

PS1: 
MA - Media Aritmetica  
MG - Media Geometrica
MH - Media Harmonica
MQ - Media Quadratica

PS2: A questão 2 eu consegui fazer expandindo o
binômio mas ela deve ser feita usando as desigualdades
das médias apenas, coisa que eu não consegui.

Abraços.

O Binômio de Newton é tão belo como a Vênus de Milo.
O que há é pouca gente para dar por isso... 
Fernando Pessoa - Poesias de Alvaro Campos

_
As informações existentes nessa mensagem e no(s) arquivo(s) anexado(s) 
são
para uso restrito, sendo seu sigilo protegido por lei. Caso não seja
destinatário, saiba que leitura, divulgação ou cópia são proibidas. 
Favor
apagar as informações e notificar o remetente. O uso impróprio será 
tratado
conforme as normas da empresa e a legislação em vigor. Agradecemos sua
colaboração.


The information mentioned in this message and in the archives attached 
are
of restricted use, and its privacy is protected by law. If you are not 
the
addressee, be aware that reading, disclosure or copy are forbidden. 
Please
delete this information and notify the sender. Inappropriate use will 
be
tracted according to company's rules and valid laws. Thank you for your
cooperation.

__
Fale com seus amigos  de graça com o novo Yahoo! Messenger 
http://br.messenger.yahoo.com/ 
=
Instruções para entrar na lista, sair da lista e usar a lista em
http://www.mat.puc-rio.br/~nicolau/olimp/obm-l.html
=


[obm-l] RES: [obm-l] Questões de Desigualdades (MA,M H, MG e MQ)

2007-01-05 Por tôpico Ralph Teixeira
1 - Sejam A, H, G e Q as médias dos n-1 números a1, a2, a3, ..., 
a(n-1). Queremos mostrar que HGAQ.

a) Considere os números a1, a2, ..., a(n-1) e G. Pela hipótese, para 
estes **n** números vale MH  MG  MA. Para não tirar a sua diversão, faça as 
contas: de MHMG vai sair HG e de MGMA sai GA. Se quiser detalhes, estão no 
PS abaixo.

b) Agora tome a1, a2, ..., a(n-1) e Q e use MAMQ para estes n números. 
Vai sair que AQ.

2) Considere os números a1=a2=a3=...=an=1+1/n e a(n+1)=1. Usando MAMG 
para eles, temos...

Abraço, Ralph

P.S. Detalhes de 1): 

a) MG = (a1a2...a(n-1)G)^(1/n) = (G^(n-1).G)^(1/n) = G

MA=(a1+a2+...+a(n-1)+G)/n=((n-1)A+G)/n  MG = G, então fazendo as 
contas, AG

MH=n/((n-1)/H+1/G)  MG=G, então, GH.

b) Então n(MQ)^2=a1^2+a2^2+...+a(n-1)^2+Q^2=nQ^2, isto é, MQ=Q

Assim, MA = (a1+a2+...+a(n-1)+Q)/n = ((n-1)A+Q)/n  MQ=Q implica AQ.

Detalhes de 2):

MG = ((1+1/n)^n)^(1/(n+1))  MA = (n(1+1/n)+1)/(n+1) = (n+2)/(n+1) = 
1+1/(n+1).

Elevando a n+1, sai o que queremos.

-Mensagem original- 
De: [EMAIL PROTECTED] em nome de Chicao Valadares 
Enviada: sex 1/5/2007 6:12 
Para: obm-l@mat.puc-rio.br 
Cc: 
Assunto: [obm-l] Questões de Desigualdades (MA,MH, MG e MQ)



1 - Prove que se as desigualdades das médias(MA,MH, MG
e MQ) é válida p/ n naturais diferentes de zero
então ela é válida p/ n-1 naturais diferentes de
zero.

2 - Prove que para todo n maior ou igual a 1 a
desigualdade é sempre é verdadeira:
(1 + 1/n)^(1/n)  (1 + 1/(n+1))^(1/(n+1))
, ou seja , a sequencia a_n = (1 + 1/n)^(1/n) é
estritamente crescente para n natural diferente de
zero(só p/ lembrar que quando n- infinito = a_n -
e mas isso não deve ser usado no problema).

PS1:
MA - Media Aritmetica 
MG - Media Geometrica
MH - Media Harmonica   
MQ - Media Quadratica

PS2: A questão 2 eu consegui fazer expandindo o
binômio mas ela deve ser feita usando as desigualdades
das médias apenas, coisa que eu não consegui.

Abraços.

winmail.dat

[obm-l] QUESTÕES DE CONCURSOS!

2007-01-04 Por tôpico Jorge Luis Rodrigues e Silva Luis
C sabe que A e B estão viajando pela Europa. Com as informações que dispõe, 
ele estima corretamente que a probabilidade de A estar hoje em Paris é 3/7, 
que a probabilidade de B estar hoje em Paris é 2/7, e que a probabilidade de 
ambos, estarem hoje em Paris é 1/7. C, então, recebe um telefonema de A 
informando que ela está hoje em Paris. Com a informação recebida pelo 
telefonema de A, C agora estima corretamente que a probabilidade de B também 
estar hoje em Paris é igual a:   
   (MPU - 04)


Maria ganhou de João nove pulseiras, quatro delas de prata e cinco delas de 
ouro. Maria ganhou de Pedro onze pulseiras, oito delas de prata e tres delas 
de ouro. Maria guarda todas essas pulseiras - e apenas essas - em sua 
pequena caixa de jóias. Uma noite, arrumando-se apressadamente para ir ao 
cinema com João, Maria retira, ao acaso, uma pulseira de sua pequena caixa 
de jóias. Ela vê, então, que retirou uma pulseira de prata. Levando-se em 
conta tais informações, a probabilidade de que a pulseira de prata que Maria 
retirou seja uma das pulseiras que ganhou de João é igual a: (MPU - 04)


Um baralho comum contém 52 cartas de 4 tipos (naipes) diferentes: em cada 
naipe, que consiste de 13 cartas, 3 dessas cartas contêm as figuras do rei, 
da dama e do valete, respectivamente. A probabilidade de se extrair uma 
carta e ela conter uma figura ou ser uma carta de paus é igual a:

(TCU - 04)


A propósito, porque no velho jogo de par-ou-ímpar os dois jogadores terão 
sempre a mesma probabilidade de vencer, se aparentemente um deles já começa 
em desvantagem? (BACEN-94)



Divirtam-se!

_
Experimente o novo Windows Live Messenger! 
http://get.live.com/messenger/overview


=
Instruções para entrar na lista, sair da lista e usar a lista em
http://www.mat.puc-rio.br/~nicolau/olimp/obm-l.html
=


[obm-l] Re: [obm-l] QUESTÕES DE CONCURSOS!

2007-01-04 Por tôpico Marcelo Salhab Brogliato

Olá,

no jogo de par ou impar ambos tem a mesma probabilidade de vencer pois:
0+0=0
0+1=1
0+2=2
0+3=3
0+4=4
0+5=5
1+0=1
1+1=2
1+2=3
1+3=4
1+4=5
1+5=6
:
:
continuando, obteremos as sequencias:
0 1 2 3 4 5
1 2 3 4 5 6
2 3 4 5 6 7
3 4 5 6 7 8
4 5 6 7 8 9
5 6 7 8 9 10

contando os pares e os impares, temos a mesma quantidade!

abracos,
Salhab


- Original Message - 
From: Jorge Luis Rodrigues e Silva Luis [EMAIL PROTECTED]

To: obm-l@mat.puc-rio.br
Sent: Thursday, January 04, 2007 10:26 AM
Subject: [obm-l] QUESTÕES DE CONCURSOS!


C sabe que A e B estão viajando pela Europa. Com as informações que dispõe, 
ele estima corretamente que a probabilidade de A estar hoje em Paris é 3/7, 
que a probabilidade de B estar hoje em Paris é 2/7, e que a probabilidade 
de ambos, estarem hoje em Paris é 1/7. C, então, recebe um telefonema de A 
informando que ela está hoje em Paris. Com a informação recebida pelo 
telefonema de A, C agora estima corretamente que a probabilidade de B 
também estar hoje em Paris é igual a: 
(MPU - 04)


Maria ganhou de João nove pulseiras, quatro delas de prata e cinco delas 
de ouro. Maria ganhou de Pedro onze pulseiras, oito delas de prata e tres 
delas de ouro. Maria guarda todas essas pulseiras - e apenas essas - em 
sua pequena caixa de jóias. Uma noite, arrumando-se apressadamente para ir 
ao cinema com João, Maria retira, ao acaso, uma pulseira de sua pequena 
caixa de jóias. Ela vê, então, que retirou uma pulseira de prata. 
Levando-se em conta tais informações, a probabilidade de que a pulseira de 
prata que Maria retirou seja uma das pulseiras que ganhou de João é igual 
a: (MPU - 04)


Um baralho comum contém 52 cartas de 4 tipos (naipes) diferentes: em cada 
naipe, que consiste de 13 cartas, 3 dessas cartas contêm as figuras do 
rei, da dama e do valete, respectivamente. A probabilidade de se extrair 
uma carta e ela conter uma figura ou ser uma carta de paus é igual a:


(TCU - 04)

A propósito, porque no velho jogo de par-ou-ímpar os dois jogadores terão 
sempre a mesma probabilidade de vencer, se aparentemente um deles já 
começa em desvantagem? (BACEN-94)



Divirtam-se!

_
Experimente o novo Windows Live Messenger! 
http://get.live.com/messenger/overview


=
Instruções para entrar na lista, sair da lista e usar a lista em
http://www.mat.puc-rio.br/~nicolau/olimp/obm-l.html
=


--
No virus found in this incoming message.
Checked by AVG Free Edition.
Version: 7.1.409 / Virus Database: 268.16.4/615 - Release Date: 3/1/2007




=
Instruções para entrar na lista, sair da lista e usar a lista em
http://www.mat.puc-rio.br/~nicolau/olimp/obm-l.html
=


[obm-l] Re: [obm-l] QUESTÕES DE CONCURSOS!

2007-01-04 Por tôpico Marcelo Salhab Brogliato

Olá,

A: pulseira de prata
B: pulseira de ouro
C: ganhou de Joao
D: ganhou de Pedro

P(A/C) = P(A inter C)/P(C)
P(C/A) = P(A inter C)/P(A)

= P(A/C) * P(C) = P(C/A) * P(A)

P(A/C) = 4/9 [probabilidade da pulseira ser de prata, dado que ela foi dada 
por Joao]

P(C) = 9/20 [probabilidade da pulseira ser de Joao]
P(A) = 12/20 [probabilidade da pulseira ser de prata]

= P(C/A) = 4/9 * 9/20 * 20/12 = 4/12 = 1/3

abracos,
Salhab

- Original Message - 
From: Jorge Luis Rodrigues e Silva Luis [EMAIL PROTECTED]

To: obm-l@mat.puc-rio.br
Sent: Thursday, January 04, 2007 10:26 AM
Subject: [obm-l] QUESTÕES DE CONCURSOS!


C sabe que A e B estão viajando pela Europa. Com as informações que dispõe, 
ele estima corretamente que a probabilidade de A estar hoje em Paris é 3/7, 
que a probabilidade de B estar hoje em Paris é 2/7, e que a probabilidade 
de ambos, estarem hoje em Paris é 1/7. C, então, recebe um telefonema de A 
informando que ela está hoje em Paris. Com a informação recebida pelo 
telefonema de A, C agora estima corretamente que a probabilidade de B 
também estar hoje em Paris é igual a: 
(MPU - 04)


Maria ganhou de João nove pulseiras, quatro delas de prata e cinco delas 
de ouro. Maria ganhou de Pedro onze pulseiras, oito delas de prata e tres 
delas de ouro. Maria guarda todas essas pulseiras - e apenas essas - em 
sua pequena caixa de jóias. Uma noite, arrumando-se apressadamente para ir 
ao cinema com João, Maria retira, ao acaso, uma pulseira de sua pequena 
caixa de jóias. Ela vê, então, que retirou uma pulseira de prata. 
Levando-se em conta tais informações, a probabilidade de que a pulseira de 
prata que Maria retirou seja uma das pulseiras que ganhou de João é igual 
a: (MPU - 04)


Um baralho comum contém 52 cartas de 4 tipos (naipes) diferentes: em cada 
naipe, que consiste de 13 cartas, 3 dessas cartas contêm as figuras do 
rei, da dama e do valete, respectivamente. A probabilidade de se extrair 
uma carta e ela conter uma figura ou ser uma carta de paus é igual a:


(TCU - 04)

A propósito, porque no velho jogo de par-ou-ímpar os dois jogadores terão 
sempre a mesma probabilidade de vencer, se aparentemente um deles já 
começa em desvantagem? (BACEN-94)



Divirtam-se!

_
Experimente o novo Windows Live Messenger! 
http://get.live.com/messenger/overview


=
Instruções para entrar na lista, sair da lista e usar a lista em
http://www.mat.puc-rio.br/~nicolau/olimp/obm-l.html
=


--
No virus found in this incoming message.
Checked by AVG Free Edition.
Version: 7.1.409 / Virus Database: 268.16.4/615 - Release Date: 3/1/2007




=
Instruções para entrar na lista, sair da lista e usar a lista em
http://www.mat.puc-rio.br/~nicolau/olimp/obm-l.html
=


[obm-l] Re: [obm-l] QUESTÕES DE CONCURSOS!

2007-01-04 Por tôpico Marcelo Salhab Brogliato

Olá,

P(AUB) = P(A) + P(B) - P(AinterB)

A = figura
B = carta de paus

no baralho temos 12 cartas com figura, entao: P(A) = 12/52
no baralho temos 13 cartas de paus, entao: P(B) = 13/52
no baralho temos 3 cartas com figura de paus, entao: P(A inter B) = 3/52

= P(AUB) = 12/52 + 13/52 - 3/52 = 22/52 = 11/26

abraços,
Salhab


- Original Message - 
From: Jorge Luis Rodrigues e Silva Luis [EMAIL PROTECTED]

To: obm-l@mat.puc-rio.br
Sent: Thursday, January 04, 2007 10:26 AM
Subject: [obm-l] QUESTÕES DE CONCURSOS!


C sabe que A e B estão viajando pela Europa. Com as informações que dispõe, 
ele estima corretamente que a probabilidade de A estar hoje em Paris é 3/7, 
que a probabilidade de B estar hoje em Paris é 2/7, e que a probabilidade 
de ambos, estarem hoje em Paris é 1/7. C, então, recebe um telefonema de A 
informando que ela está hoje em Paris. Com a informação recebida pelo 
telefonema de A, C agora estima corretamente que a probabilidade de B 
também estar hoje em Paris é igual a: 
(MPU - 04)


Maria ganhou de João nove pulseiras, quatro delas de prata e cinco delas 
de ouro. Maria ganhou de Pedro onze pulseiras, oito delas de prata e tres 
delas de ouro. Maria guarda todas essas pulseiras - e apenas essas - em 
sua pequena caixa de jóias. Uma noite, arrumando-se apressadamente para ir 
ao cinema com João, Maria retira, ao acaso, uma pulseira de sua pequena 
caixa de jóias. Ela vê, então, que retirou uma pulseira de prata. 
Levando-se em conta tais informações, a probabilidade de que a pulseira de 
prata que Maria retirou seja uma das pulseiras que ganhou de João é igual 
a: (MPU - 04)


Um baralho comum contém 52 cartas de 4 tipos (naipes) diferentes: em cada 
naipe, que consiste de 13 cartas, 3 dessas cartas contêm as figuras do 
rei, da dama e do valete, respectivamente. A probabilidade de se extrair 
uma carta e ela conter uma figura ou ser uma carta de paus é igual a:


(TCU - 04)

A propósito, porque no velho jogo de par-ou-ímpar os dois jogadores terão 
sempre a mesma probabilidade de vencer, se aparentemente um deles já 
começa em desvantagem? (BACEN-94)



Divirtam-se!

_
Experimente o novo Windows Live Messenger! 
http://get.live.com/messenger/overview


=
Instruções para entrar na lista, sair da lista e usar a lista em
http://www.mat.puc-rio.br/~nicolau/olimp/obm-l.html
=


--
No virus found in this incoming message.
Checked by AVG Free Edition.
Version: 7.1.409 / Virus Database: 268.16.4/615 - Release Date: 3/1/2007




=
Instruções para entrar na lista, sair da lista e usar a lista em
http://www.mat.puc-rio.br/~nicolau/olimp/obm-l.html
=


[obm-l] Re: [obm-l] QUESTÕES DE CONCURSOS!

2007-01-04 Por tôpico Marcelo Salhab Brogliato

Olá,

P(A) = 3/7
P(B) = 2/7
P(A inter B) = 1/7

P(B|A) = P(AinterB)/P(A) = 1/7 * 7/3 = 1/3

abraços,
Salhab


- Original Message - 
From: Jorge Luis Rodrigues e Silva Luis [EMAIL PROTECTED]

To: obm-l@mat.puc-rio.br
Sent: Thursday, January 04, 2007 10:26 AM
Subject: [obm-l] QUESTÕES DE CONCURSOS!


C sabe que A e B estão viajando pela Europa. Com as informações que dispõe, 
ele estima corretamente que a probabilidade de A estar hoje em Paris é 3/7, 
que a probabilidade de B estar hoje em Paris é 2/7, e que a probabilidade 
de ambos, estarem hoje em Paris é 1/7. C, então, recebe um telefonema de A 
informando que ela está hoje em Paris. Com a informação recebida pelo 
telefonema de A, C agora estima corretamente que a probabilidade de B 
também estar hoje em Paris é igual a: 
(MPU - 04)


Maria ganhou de João nove pulseiras, quatro delas de prata e cinco delas 
de ouro. Maria ganhou de Pedro onze pulseiras, oito delas de prata e tres 
delas de ouro. Maria guarda todas essas pulseiras - e apenas essas - em 
sua pequena caixa de jóias. Uma noite, arrumando-se apressadamente para ir 
ao cinema com João, Maria retira, ao acaso, uma pulseira de sua pequena 
caixa de jóias. Ela vê, então, que retirou uma pulseira de prata. 
Levando-se em conta tais informações, a probabilidade de que a pulseira de 
prata que Maria retirou seja uma das pulseiras que ganhou de João é igual 
a: (MPU - 04)


Um baralho comum contém 52 cartas de 4 tipos (naipes) diferentes: em cada 
naipe, que consiste de 13 cartas, 3 dessas cartas contêm as figuras do 
rei, da dama e do valete, respectivamente. A probabilidade de se extrair 
uma carta e ela conter uma figura ou ser uma carta de paus é igual a:


(TCU - 04)

A propósito, porque no velho jogo de par-ou-ímpar os dois jogadores terão 
sempre a mesma probabilidade de vencer, se aparentemente um deles já 
começa em desvantagem? (BACEN-94)



Divirtam-se!

_
Experimente o novo Windows Live Messenger! 
http://get.live.com/messenger/overview


=
Instruções para entrar na lista, sair da lista e usar a lista em
http://www.mat.puc-rio.br/~nicolau/olimp/obm-l.html
=


--
No virus found in this incoming message.
Checked by AVG Free Edition.
Version: 7.1.409 / Virus Database: 268.16.4/615 - Release Date: 3/1/2007




=
Instruções para entrar na lista, sair da lista e usar a lista em
http://www.mat.puc-rio.br/~nicolau/olimp/obm-l.html
=


[obm-l] QUESTÕES DE CONCURSO!

2006-07-31 Por tôpico Jorge Luis Rodrigues e Silva Luis

Olá, pessoal!

Planejando uma festa, um grupo de 9 crianças decidiu que cada uma delas 
deveria levar 1 litro de suco de fruta, a escolher entre laranja, limão e 
uva. Decidiram também comprar um bolo e salgadinhos; com o intuito de 
arrecadar dinheiro para a compra, resolveram vender bilhetes numerados com 3 
dígitos, cuja soma deverá ser igual a 9. O portador do bilhete vencedor 
ganhará a metade da quantia arrecadada. Será considerado vencedor o bilhete 
numerado com os dígitos a, b e c, que indicam, respectivamente, a quantidade 
de litros de sucos de laranja, uva e limão trazidos para a festa. Para que 
haja exatamente um vencedor, qual o número de bilhetes que devem ser feitos?

(FATEC)

Numa pequena empresa de montagem, com 50 empregados, o gerente resolveu 
avaliar o desempenho dos seus funcionários. Foi constatado que 5 
trabalhadores completaram o trabalho além do tempo exigido; 6 montavam os 
produtos com defeito; e 2 completavam o trabalho defeituosamente e além do 
tempo exigido. O trabalhador que se enquadrar em qualquer uma dessas três 
situações é considerado como de fraco desempenho. Qual a probabilidade de o 
gerente atribuir desempenho fraco a um trabalhador qualquer?  (B.N.B)


Ao escolherem as datas de seus vestibulares, três instituições de ensino 
decidiram que suas provas seriam realizadas na primeira semana de um 
determinado mês. A probabilidade de que essas provas não aconteçam em dias 
consecutivos é, aproximadamente:   (VESTIBULAR - RJ)

Nota: Taí um problema que vai dar o que falar...!

Em uma prova cada pergunta tem 3 alternativas, apenas 1 correta. Um 
candidato sabe 30% das respostas. Se ele deu a resposta correta para cada 
uma das perguntas, qual a probabilidade de ele ter chutado?   (IBMEC)
...ainda se encontra em aberto na lista...



Abraços!

_
Acompanhe os desfiles do evento São Paulo Fashion Week. 
http://www.msn.com.br/diversao/spfw/


=
Instruções para entrar na lista, sair da lista e usar a lista em
http://www.mat.puc-rio.br/~nicolau/olimp/obm-l.html
=


[obm-l] Re: [obm-l] Questões do Livro do Hefez

2006-05-02 Por tôpico Artur Costa Steiner
2) Vejamos o caso o caso a^p - a. Temos que o primo p= 5 eh impar, e desta
forma p-1 eh par. Assim, p-1 = 2p' para algum inteiro positivo p'.

Temos que a^p - a = a(a^(p-1) -1) = a(a^(2p') - 1) = a(a^p' + 1)(a^p' - 1).
Se a for par, entao eh imediato que a^p - a eh par. Se a for impar, entao
a^p'eh impar e os numeros (a^p' + 1) e (a^p' - 1) sao ambos pares. Logo,
tambem neste caso a^p - a eh par, sendo inclusive multiplo de 4.

Se a for multiplo de 3, entao eh imediato que a^p - a eh tambem multiplo de
3. Se a nao for multiplo de 3, temos 2 casos: se a for par, a^p' eh par.
Logo, um dos numeros (a^p' + 1) ou (a^p -1) eh multiplo de 3 (para todo
numero par n que nao seja multiplo de 3, n-1 ou n +1 eh multiplo de 3). Isto
nos mostra que a^p - a eh multiplo de 3. Se a nao for multiplo de 3 e for
impar, entao a^p' eh um impar nao multiplo de 3. Entao, dentre os numeros
pares (a^p' + 1) e (a^p' - 1) um deles eh necessariamente multiplo de 3 (se
n eh um impar nao multiplo de 3, entao um dos pares n-1 e n+1 eh sempre
multiplo de 3).
Chegamos assim aa conclusao de que, nas condicoes dadas, a^p - a eh sempre
par e multiplo de 3, logo eh multiplo de 6. 

Pelo pequeno teorema de Fermat, temos ainda que a^p = a (mod p). Logo a^p -
a eh multiplo de p. E como para o primo p temos  p 3, segue-se, em virtude
da conclusao anterior, que a^p - a eh multiplo de 6p, ou seja, 6p divide a^p
- p, conforme afirmado.

O outro deve ter um saida semelhante, depois vemos se dah pra sair.
De uma conferida, meu conhecimento de teoria dos numeros eh muito limitado.
Artur


  Agradeço  qualquer ajuda nas seguintes questões:
 
  1) Mostre que existe uma correspondência biunívoca
  entre pares de primos
  gêmeos e números n tais que n^2 -1 possui 4
  divisores.
 
  2) Seja p 3 um primo. Mostre que a^p - a  e a^p. b-
  b^p . a são divisíveis
  por 6p, para todos a0, com ab.
 
  3) seja p um primo ímpar. Mostre que se pode
  escrever p = y^2 - x^2, com  x
  e y positivos, de modo único.
 
  Obrigado
=

=
Instruções para entrar na lista, sair da lista e usar a lista em
http://www.mat.puc-rio.br/~nicolau/olimp/obm-l.html
=


[obm-l] Re: [obm-l] Questões do Livro do Hefez

2006-05-01 Por tôpico Ricardo Khawge

Desculpe Artur, já encontrei a mensagem



From: Artur Costa Steiner [EMAIL PROTECTED]
Reply-To: obm-l@mat.puc-rio.br
To: obm-l@mat.puc-rio.br
Subject: Re: [obm-l] Questões do Livro do Hefez
Date: Sun, 30 Apr 2006 23:55:37 -0700 (PDT)

1) Suponhamos que m = n^2 - 1 = (n+1)(n-1)  possua 4
divisores. Temos que n=3. Se n for impar, entao n- 1
e n+1 sao ambos pares, implicando que m seja multiplo
de 4. Se n =3, entao m =8 tem 4 divisores, mas isto
nao leva ainda aa correspondencia desejada. Se n=5
for impar, entao os numeros pares n-1 =4 e n+1 sao
divisores de m. Alem disto, m tem como divisores os
numeros 1 , 2 e m, de modo que para n=5, impar, m tem
pelo menos 5 divisores, contraraiamente aa hipotese.
Assim, valore impares de n nao implicam a
correspondencia de4sejada.
Se n=4 for par, entao n-1=3 e n+1 sao ambos
divisores impares de m. Alem disto, m tem por
divisores os numeros 1 e o proprio m. Dado que m tem
exatamente 4 divisores, segue-se que n-1 e n+1 sao
ambos primos, pois, se ao menos um deles fosse
composto, m teria pelo menos um divisor a mais do que
os citados, contrariamente aa hipotese basica.
Concluimos assim que, a cada valor par de n para o
qual n-1 e n+1 sejam primos - logo primos gemeos -
corresponde o par (n-1 , n+1) de primos gemeos.
Por outro lado, se n-1 e n+1 sao pimos gemeos, entao m
= n^2 -1 = (n-1)(n+1) tem por fatores primos unica e
exclusivamente n-1 e n+1 (teorema fundamental da
aritmetica). Como, alem disto, 1 e m sao divisores de
m, segue-se que m tem exatamente 4 divisores. Isto eh,
a cada par de primos gemeos, corresponde um numero da
forma n^2 -1. Concluimos, assim, que a correspondencia
entre o conjunto dos pares de primos gemeos e os
numeros da forma n^2 -1 eh ,de fato, biunivica, hah
uma bijecao entre os 2 conjuntos.

A questao 3 jah foi discutida na lista, de forma mais
geral, hah alguns dias, sob o titulo Diferenca de 2
quadrados. Basta fazer y = (p+1)/2 e x = (p-1)/2.

A questao 2 parece mais complicada, vamos tentar outra
hora.

Artur

--- Ricardo Khawge [EMAIL PROTECTED] wrote:

 Agradeço  qualquer ajuda nas seguintes questões:

 1) Mostre que existe uma correspondência biunívoca
 entre pares de primos
 gêmeos e números n tais que n^2 -1 possui 4
 divisores.

 2) Seja p 3 um primo. Mostre que a^p - a  e a^p. b-
 b^p . a são divisíveis
 por 6p, para todos a0, com ab.

 3) seja p um primo ímpar. Mostre que se pode
 escrever p = y^2 - x^2, com  x
 e y positivos, de modo único.

 Obrigado


_
 Ganhe tempo encontrando o arquivo ou e-mail que você
 precisa com Windows
 Desktop Search. Instale agora em
 http://desktop.msn.com.br


=
 Instruções para entrar na lista, sair da lista e
 usar a lista em
 http://www.mat.puc-rio.br/~nicolau/olimp/obm-l.html

=



__
Do You Yahoo!?
Tired of spam?  Yahoo! Mail has the best spam protection around
http://mail.yahoo.com
=
Instruções para entrar na lista, sair da lista e usar a lista em
http://www.mat.puc-rio.br/~nicolau/olimp/obm-l.html
=


_
Ligações gratuitas de PC-para-PC para qualquer lugar do Brasil e do mundo 
com o  MSN Messenger. Saiba mais em   
http://imagine-msn.com/messenger/default2.aspx?locale=pt-br


=
Instruções para entrar na lista, sair da lista e usar a lista em
http://www.mat.puc-rio.br/~nicolau/olimp/obm-l.html
=


[obm-l] Questões do Livro do Hefez

2006-04-30 Por tôpico Ricardo Khawge

Agradeço  qualquer ajuda nas seguintes questões:

1) Mostre que existe uma correspondência biunívoca entre pares de primos 
gêmeos e números n tais que n^2 -1 possui 4 divisores.


2) Seja p 3 um primo. Mostre que a^p - a  e a^p. b- b^p . a são divisíveis 
por 6p, para todos a0, com ab.


3) seja p um primo ímpar. Mostre que se pode escrever p = y^2 - x^2, com  x 
e y positivos, de modo único.


Obrigado

_
Ganhe tempo encontrando o arquivo ou e-mail que você precisa com Windows 
Desktop Search. Instale agora em  http://desktop.msn.com.br


=
Instruções para entrar na lista, sair da lista e usar a lista em
http://www.mat.puc-rio.br/~nicolau/olimp/obm-l.html
=


[obm-l] QUESTÕES DO PROVÃO!

2006-02-21 Por tôpico Jorge Luis Rodrigues e Silva Luis
Ok! Qwert e demais colegas! Eis algumas questões que por motivo ignorado 
apresentaram alto índice de erros no ENC-2001...


Qual o número de planos de simetria de um cubo? (Ver resolução na lista...)

A soma dos primeiros n termos de uma sequência numérica é 1/n. Calcular o 
vigésimo termo.


Qual a interseção dos planos das faces laterais opostas de uma pirâmide 
quadrangular regular?


Outra questão que merece destaque, perguntava através de alternativas, o 
modo de se obter o gráfico da função y=f(x+1) a partir do gráfico de y=f(x). 
É triste constatar que apenas 13% sabiam que a resposta é uma translação de 
uma unidade para a esquerda.


A propósito, será possível usando a calculadora do feirante extrair a raiz 
n-ésima de um número qualquer?  Resp: Sim


Abraços!

_
Seja um dos primeiros a testar o novo Windows Live Mail Beta. Acesse 
http://www.ideas.live.com/programpage.aspx?versionId=5d21c51a-b161-4314-9b0e-4911fb2b2e6d


=
Instruções para entrar na lista, sair da lista e usar a lista em
http://www.mat.puc-rio.br/~nicolau/olimp/obm-l.html
=


[obm-l] Questões de Teoria dos Números (Livro do Plínio)

2006-01-05 Por tôpico Sérgio Farias
Olá.   Estou tendo dificuldade com algumas questões do livro Introdução à Teoria dos Números do Plínio, editado pelo IMPA.Eis as questões:- "Provar que não existe n pertencente aos naturais tal que 7 divida(4n^2 - 3).  - "Mostrar que 3 é o único primo p tal que p, p+2 e p+4 são todos primos"  - "Mostrar que se b divide c, então MDC (a + c, b) = MDC (a, b)"Desde já agradeço.Cordialmente,  Sérgio Farias.
		 
Yahoo! doce lar. Faça do Yahoo! sua homepage.

Re: [obm-l] Questões de Teoria dos Números (Livro do Plínio)

2006-01-05 Por tôpico Bruno França dos Reis
Oi, Sergio
Aqui vão uns rabiscos das questões.

1)
n^2 mod 7 só pode assumir os valores 0, 1, 2 e 4, (veja que uma classe
completa de residuos modulo 7 é -3,-2,-1,0,1,2,3, e que se vc elevar os
membros ao quadrado, sobram só os positivos, então pra determinar as
possibilidades de n^2 mod 7 basta ver o valor de 0^2, 1^2, 2^2 e 3^2
mod 7) então 4n^2 só pode assumir 0, 1, 2, 4 também, e 4n^2 - 3,
portanto, não pode assumir nunca o valor 0, o que implica que 7 não
divide nenhum número da forma 4n^2 - 3.

2)
Pra esse aqui, veja que para 2 não dá, para 3 dá, e assuma que p é um
primo maior que 3. Então veja que, tomando QUALQUER inteiro (nao
precisa ser primo), OU n, OU n+2 OU n+4 (sendo estes ou's exclusivos,
isto é, a veracidade de um implica a falsidade dos outros 2, e
necessariamente há um verdadeiro) é um multiplo de 3. Isso também vale
para os primos maiores que 3.

3)
Seja c = kb. Sejam m = MDC(a, b) e a = mu, b = mv, com MDC(u,v)=1. Temos:
a + c = mu + kb = mu + kmv = m(u + kv)
MDC(a+b,c) = MDC(m(u+kv),mv) = m*MDC(u+kv,v)
Mas u+kv e v são primos entre si (por quê? use o fato de que u e v são primos entre si, e veja o que acontece modulo v).
Então MDC(u+kv,v) = 1 == MDC(a+b,c) = m = MDC(a,b)

AbraçoOn 1/5/06, Sérgio Farias [EMAIL PROTECTED] wrote:
Olá.   Estou tendo dificuldade com algumas questões do livro Introdução à Teoria dos Números do Plínio, editado pelo IMPA.Eis as questões:- Provar que não existe n pertencente aos naturais tal que 7 divida(4n^2 - 3).
  - Mostrar que 3 é o único primo p tal que p, p+2 e p+4 são todos primos  - Mostrar que se b divide c, então MDC (a + c, b) = MDC (a, b)Desde já agradeço.
Cordialmente,  Sérgio Farias.
		 
Yahoo! doce lar. Faça do Yahoo! sua homepage.

-- Bruno França dos Reisemail: bfreis - gmail.comgpg-key: 
http://planeta.terra.com.br/informatica/brunoreis/brunoreis.keyicq: 12626000e^(pi*i)+1=0


Re: [obm-l] Questões de Teoria dos Números (Livro do Plínio)

2006-01-05 Por tôpico Sérgio Farias
Primeiramente, meus sinceros agradecimentos.Algumas dúvidas:  1) O que vem a ser 'n^2 mod 7'? Nunca vi nada parecido com isso e tenho  certeza absoluta que o Plínio, pelo menos até agora (estou no capt 1),  não deu a definição e suas repectivas propriedades.2) "Então veja que, tomando QUALQUER inteiro (nao  precisa ser primo), OU n, OU n+2 OU n+4 (sendo estes ou's exclusivos,  isto é, a veracidade de um implica a falsidade dos outros 2, e  necessariamente há um verdadeiro) é um multiplo de 3"Gostaria de saber como provar que estes ou's são exclusivos e que nescessariamente há um verdadeiro.3)  Como provar que (u + kv, v) = 1?  "(por quê? use o fato de que u e v são primos entre si, e veja o que acontece modulo v)."  Tudo bem, MDC (u,v) =1. Não compreendi esta sua frase: "veja o que acontece modulo v"Do resto, tudo compreendi.Cordialmente,  Sérgio Farias.Bruno
 França dos Reis [EMAIL PROTECTED] escreveu:  Oi, Sergio  Aqui vão uns rabiscos das questões.1)  n^2 mod 7 só pode assumir os valores 0, 1, 2 e 4, (veja que uma classe  completa de residuos modulo 7 é -3,-2,-1,0,1,2,3, e que se vc elevar os  membros ao quadrado, sobram só os positivos, então pra determinar as  possibilidades de n^2 mod 7 basta ver o valor de 0^2, 1^2, 2^2 e 3^2  mod 7) então 4n^2 só pode assumir 0, 1, 2, 4 também, e 4n^2 - 3,  portanto, não pode assumir nunca o valor 0, o que implica que 7 não  divide nenhum número da forma 4n^2 - 3.2)  Pra esse aqui, veja que para 2 não dá, para 3 dá, e assuma que p é um  primo maior que 3. Então veja que, tomando QUALQUER inteiro (nao  precisa ser primo), OU n, OU n+2 OU n+4 (sendo estes ou's exclusivos,  isto é, a veracidade de um implica a falsidade dos outros!
 2, e 
 necessariamente há um verdadeiro) é um multiplo de 3. Isso também vale  para os primos maiores que 3.3)  Seja c = kb. Sejam m = MDC(a, b) e a = mu, b = mv, com MDC(u,v)=1. Temos:  a + c = mu + kb = mu + kmv = m(u + kv)  MDC(a+b,c) = MDC(m(u+kv),mv) = m*MDC(u+kv,v)  Mas u+kv e v são primos entre si (por quê? use o fato de que u e v são primos entre si, e veja o que acontece modulo v).  Então MDC(u+kv,v) = 1 == MDC(a+b,c) = m = MDC(a,b)AbraçoOn 1/5/06, Sérgio Farias [EMAIL PROTECTED] wrote:  Olá.   Estou tendo dificuldade com algumas questões do livro Introdução à Teoria dos Números do Plínio, editado pelo IMPA.   
 Eis as questões:- "Provar que não existe n pertencente aos naturais tal que 7 divida(4n^2 - 3).- "Mostrar que 3 é o único primo p tal que p, p+2 e p+4 são todos primos"  - "Mostrar que se b divide c, então MDC (a + c, b) = MDC (a, b)"Desde já agradeço.  Cordialmente,  Sérgio Farias.   Yahoo! doce lar. Faça do Yahoo! sua homepage.-- Bruno França dos Reisemail: bfreis - gmail.comgpg-key:  
 http://planeta.terra.com.br/informatica/brunoreis/brunoreis.keyicq: 12626000e^(pi*i)+1=0  
		 
Yahoo! doce lar. Faça do Yahoo! sua homepage.

Re: [obm-l] Questões de Teoria dos Números (Livro do Plínio)

2006-01-05 Por tôpico wellnet5

Valeu Diego, tô vendo que estás estudando,mas valeu mesmo pela resoluçao, estava com dúvidas.Até fevereiro nas aulas.Estuda garoto.

Atenciosamente wellington
-Original Message-From: diego andres [EMAIL PROTECTED]To: obm-l@mat.puc-rio.brSent: Thu, 5 Jan 2006 14:10:53 + (GMT)Subject: Re: [obm-l] Questões de Teoria dos Números (Livro do Plínio)



1-eh soh considerar n=7k,7k+1,7k+2,7k+3,7k+4,7k+5,7k+6 e sair testando:
7k eh congruente a 0 mod 7, 4(7k)^2-3 eh congruo a -3mod 77k+1 eh congruente a1 mod 7, 4(7k+1)^2-3 eh congruo a 1mod 77k+2 eh congruente a2 mod 7, 4(7k+2)^2-3 eh congruo a -1mod 7
7k+3 eh congruente a3 mod 7, 4(7k+3)^2-3 eh congruo a 5mod 7
assim voce termina o resto.
2-se p,p+2e p+4 sao primos observe que isso esta da forma: a,a+b,a+2b.
que eh um sistema deresiduo completo modulo 3,todos os elementos tem de ser impares,pois uma vez um par implica em todos pares,que eh impossivel,pois so existe um primo par(2)logo:
analise ele como residuos moldulo15 e voce verá que a unica opcao eh: 
15k+3,15k+5,15k+7
assim os unico possiveis eh 3,5,7.
3-se "b" divide "c" entao c=bk vamos supor MDC(a,b)=d entao MDC(a+c,b)pela definicao sera o maximo divisor que dividirá ambos "a" e "c" e como "b" divide "c" o limiar está em "a" logo MDC(a+c,b)=d=MDC(a,b).
Diego Andrés.
Sérgio Farias [EMAIL PROTECTED] escreveu:

Olá. 
Estou tendo dificuldade com algumas questões do livro Introdução à Teoria dos Números do Plínio, editado pelo IMPA.

Eis as questões:

- "Provar que não existe n pertencente aos naturais tal que 7 divida(4n^2 - 3).
- "Mostrar que 3 é o único primo p tal que p, p+2 e p+4 são todos primos"
- "Mostrar que se b divide c, então MDC (a + c, b) = MDC (a, b)"

Desde já agradeço.

Cordialmente,
Sérgio Farias.


Yahoo! doce lar. Faça do Yahoo! sua homepage.



Yahoo! doce lar. Faça do Yahoo! sua homepage. 



Try the New Netscape Mail Today!
Virtually Spam-Free | More Storage | Import Your Contact Listhttp://mail.netscape.com




Re: [obm-l] Questões de Geometria Interessantes

2006-01-02 Por tôpico Iuri
1) esse trapezio eh meio hexagono, logo a area eh metade da area do hexagono. A base menor do hexagono inscrito é R e a maior eh 2R, entao a area eh (B+b)*h/2 = 3 * R^2 * sqrt(3)/2 
Em 02/01/06, Adélman de Barros Villa Neto [EMAIL PROTECTED] escreveu:
1) Calcular o perímetro de um trapézio que está inscrito numa semicircunferência de raio R.2) Um fio de comprimento L é cortado em dois pedaços, um dos quais formará um círculo e o outro, um quadrado. Como deve ser cortado o fio para que a soma das áreas do círculo e do quadrado seja máxima?
3)Calcule o raio da base e a altura do cone de volume mínimo que pode circunscrever uma esfera de raio R.Agradeço pela Atenção.=Instruções para entrar na lista, sair da lista e usar a lista em
http://www.mat.puc-rio.br/~nicolau/olimp/obm-l.html=



[obm-l] Re:[obm-l] questões olim internac ional

2005-12-21 Por tôpico lponce
Ao amigo da obm
vai uma sugestão para o problema :

(Moldávia – 2000) Os números inteiros a,b,c satisfazem à relação a + b + c = 0. Mostre que o número 2(a4 + b 4 +c4 )é um quadrado perfeito.
Uma possível demonstração . Desde que a + b + c = 0, 

 c = - (a + b)  [ 1 ]
 c2 = a2 + 2ab + b2   [ 2 ]
a2 + b2 + c2 + 2 (ab + c (a+b) ) = 0   [ 3]
Nestas condições:
Elevando ambos membros de [3] ao quadrado e substituindo [ 1] encontramos: 
 a2 + b2 + c2 = 2 (a2 +ab+ b2 )  [ 4]
Elevando ambos membros de [4] ao quadrado e substituindo [ 2] encontramos:
a4 + b 4 + c4 + 2 [a2b2 + (a2 + b2) (a2 + 2ab + b2)]=4[a4 +3a2b2 +2 a3 b+2 ab3 +b4]
Simplificando e agrupando de modo conveniente, obtém-se sucessivamente:
a4 + b 4 + c4 = 2[(a4 + a3b + a2b2 ) + (a3 b + a2b2 + a b3 ) + (a2b2 + a b3 + b4 )]
a4 + b 4 + c4 = 2[a2 (a2 + a b + b2 ) + ab(a2 + a b + b2 ) + b2 (a2 + a b + b2 )]
Dai, 
2a4 +2b 4 +2 c4 = 4[a2 (a2 + a b + b2 ) + ab(a2 + a b + b2 ) + b2 (a2 + a b + b2 )]
 = [ 2 (a2 + a b + b2 ) ]2 ,
Portanto, para a, b e c inteiros tais que: a + b + c = 0, segue-se que
2a4 +2b 4 +2 c4 =[2 (a2 + a b + b2 ) ]2 , isto é, que o número 2(a4 + b 4 +c4 )é um quadrado perfeito, o que finaliza a demonstração.

Do amigo PONCE
Aproveitando este Email ,gostaria a desejar a todos os membros desta lista um feliz natal e ummaravilhoso 2006.






De:
[EMAIL PROTECTED]




Para:
obm-l@mat.puc-rio.br




Cópia:





Data:
Fri, 16 Dec 2005 16:00:12 -0300 (ART)




Assunto:
[obm-l] questões olim internacional
 valeu para aqueles que me ajudaram resolvendo as 2 questões anteriores .
 
 agora quem puder ajuda nessa eu agradeço !
 http://img29.imagevenue.com/img.php?loc=loc298image=64fe3_fagner3.JPG


Yahoo! doce lar. Faça do Yahoo! sua homepage. 


[]a, L.PONCE.


Re: [obm-l] questões olim internacional

2005-12-21 Por tôpico mentebrilhante brilhante
eu não tô conseguindo entende parti '' aqui eu não sei de cabeça '' quem pode ajuda Ricardo Bittencourt [EMAIL PROTECTED] escreveu:  mentebrilhante brilhante wrote: valeu para aqueles que me ajudaram resolvendo as 2 questões anteriores . agora quem puder ajuda nessa eu agradeço ! http://img29.imagevenue.com/img.php?loc=loc298image=64fe3_fagner3.JPG Os inteiros a,b,c satisfazem a+b+c=0, prove que 2a^4+2b^4+2c^4é um quadrado perfeito. É só fazer na unha, c=-a-b, então2a^4+2b^4+2c^4=2a^4+2b^4+2(-a-b)^4=2a^4+2b^4+2(a^4+4a^3b+6a^2b^2+4ab^3+b^4)=4a^4+8a^3b+12a^2b^2+8ab^3+4b^4=4(a^4+2a^3b+3a^2b^2+2ab^3+b^4)=Aqui eu não sei de cabeça, mas tenho a impressãoque é 4(a^2+b^2+C)^2, expandindo dá:4(a^4+b^4+C^2+2a^2C+2b^2C+2a^2b^2)!
E depois,
 subtraindo:C^2+2a^2C+2b^2C=2a^3b+a^2b^2+2ab^3,de onde você vê que C=ab, logo2a^4+2b^4+2c^4 = (2*(a^2+ab+b^2))^2Ricardo Bittencourt http://www.mundobizarro.tk[EMAIL PROTECTED] "kimitatino kitiwa subete CATS ga itadaita"-- União contra o forward - crie suas proprias piadas --=Instruções para entrar na lista, sair da lista e usar a lista emhttp://www.mat.puc-rio.br/~nicolau/olimp/obm-l.html=
		 
Yahoo! doce lar. Faça do Yahoo! sua homepage.

[obm-l] questões olim internacional

2005-12-16 Por tôpico mentebrilhante brilhante
valeu para aqueles que me ajudaram resolvendo as 2 questões anteriores .agora quem puder ajuda nessa eu agradeço !  http://img29.imagevenue.com/img.php?loc=loc298image=64fe3_fagner3.JPG
		 
Yahoo! doce lar. Faça do Yahoo! sua homepage.

Re: [obm-l] questões olim internacional

2005-12-16 Por tôpico Ricardo Bittencourt

mentebrilhante brilhante wrote:

valeu  para aqueles  que me ajudaram  resolvendo as  2 questões anteriores .
agora  quem puder ajuda nessa  eu agradeço !
http://img29.imagevenue.com/img.php?loc=loc298image=64fe3_fagner3.JPG 


Os inteiros a,b,c satisfazem a+b+c=0, prove que 2a^4+2b^4+2c^4
é um quadrado perfeito. É só fazer na unha, c=-a-b, então

2a^4+2b^4+2c^4=
2a^4+2b^4+2(-a-b)^4=
2a^4+2b^4+2(a^4+4a^3b+6a^2b^2+4ab^3+b^4)=
4a^4+8a^3b+12a^2b^2+8ab^3+4b^4=
4(a^4+2a^3b+3a^2b^2+2ab^3+b^4)=

Aqui eu não sei de cabeça, mas tenho a impressão
que é 4(a^2+b^2+C)^2, expandindo dá:

4(a^4+b^4+C^2+2a^2C+2b^2C+2a^2b^2)

E depois, subtraindo:
C^2+2a^2C+2b^2C=2a^3b+a^2b^2+2ab^3,

de onde você vê que C=ab, logo

2a^4+2b^4+2c^4 = (2*(a^2+ab+b^2))^2


Ricardo Bittencourt   http://www.mundobizarro.tk
[EMAIL PROTECTED]  kimitatino kitiwa subete CATS ga itadaita
-- União contra o forward - crie suas proprias piadas --
=
Instruções para entrar na lista, sair da lista e usar a lista em
http://www.mat.puc-rio.br/~nicolau/olimp/obm-l.html
=


Re: [obm-l] questões de olim internacional

2005-12-14 Por tôpico Marcos Martinelli
 Legal!


[obm-l] questões de olim internacional

2005-12-13 Por tôpico mentebrilhante brilhante
preciso de ajuda nessas duas questões http://img24.imagevenue.com/img.php?loc=loc99image=5b711_fagner2.JPG
		 
Yahoo! doce lar. Faça do Yahoo! sua homepage.

Re: [obm-l] questões de olim internacional

2005-12-13 Por tôpico Marcos Martinelli



Na questão 74, faça y=x^2-3x-2 e obtenha o seguinte sistema de equações:

.y=x^2-3x-2
.x=y^2-3y-2
E agora subtraia as duas equações.




[obm-l] Re:[Spam] [obm-l] questões!!

2005-11-14 Por tôpico fgb1
Essa sua pergunta é muito subjetiva. Vc quer questões contextualizadas, questões difíceis, elas são para concursos... enfim. Seja mais claro.








De:
[EMAIL PROTECTED]




Para:
obm-l@mat.puc-rio.br




Cópia:





Data:
Mon, 14 Nov 2005 00:18:18 -0200




Assunto:
[Spam] [obm-l] questões!!
 alguem sabe onde eu posso encontrar questões boas, ótimas de 5º a 8º série ??
 e também livros ??
 
 =
 Instruções para entrar na lista, sair da lista e usar a lista em
 http://www.mat.puc-rio.br/~nicolau/olimp/obm-l.html
 =
 
 E-mail classificado pelo Identificador de Spam Inteligente Terra.
 Para alterar a categoria classificada, visite
 http://mail.terra.com.br/protected_email/imail/imail.cgi?+_u=fgb1_l=1,1131936105.663520.855.mueru.terra.com.br,2842,20031127114101,20031127114101
 
 Esta mensagem foi verificada pelo E-mail Protegido Terra.
 Scan engine: McAfee VirusScan / Atualizado em 11/11/2005 / Versão: 4.4.00/4626
 Proteja o seu e-mail Terra: http://mail.terra.com.br/
 


Fábio Bernardo
[EMAIL PROTECTED]
Tel. 2676-6854


  1   2   3   >